Anda di halaman 1dari 152

Chandan Kumar Sah Page 1 of 152 MBA 2ND Semester

Chandan Kumar Sah Page 2 of 152 MBA 2ND Semester


Assignment Set – 1

Q1. What are the components of systems productivity? Explain how CAD and CIM help
in improving productivity.
Ans: Productivity is generally expressed as the ratio of outputs to inputs. Enhancement of
Productivity is achieved by either reducing the inputs for the same output or increasing
the output by using the same input. Productivity is a measure of the efficiency of the
system and looks at the economies achieved during the processes. The processes are:
People, Machines, Facilitating goods. Quality circles are very efficient in incorporating
low cost and non-intrusive methods of improving productivity and quality throughout the
organization.
CAD: CAD stands of Computer Aided Design. It is an electronic system for designing
new parts or products or altering existing ones, replacing drafting computers and graphics
software. Designer can create drawings can create drawings and view them from any
angle on a display monitor. Images of different components can be seen as ‘assembled’;
sections taken and relative positions can be checked to great accuracies. Views can be
made from different angles and positions, so that the visualization process of the designed
component/product helps the designer to suggest alternatives to the customer and the
production department. Using the design data stored in the computer’s memory,
manufacturing engineers and other users can quickly obtain printouts of plans and
specifications for a part of product. The software can generate the volume, wrights of
components as also other engineering parameters like centre of gravity, deflections under
estimated loads, and various other assemblies. The database can be prepared, updated
continuously and their access to executive. Analysts use CAD to store, retrieve, and
classify data about various parts.
CIM: CIM stands for Computer Integrated manufacturing. Integration occurs when a
broad range of manufacturing and supporting activities are linked. CIM is complete
automation of a manufacturing plant, with all processes functions under computer control
and digital information tying them together. The three major functions in manufacturing
are production, design, and management function. Production function converts resources
into products. The design function transforms customer specifications into design. Finally
the management functions plan and control production activities. The three computer aids

Chandan Kumar Sah Page 3 of 152 MBA 2ND Semester


in CIM are: i) Computer aid to the production function, ii) Computer aid to the design
function, iii) Computer aid to the management function. When it comes to production,
CIMs are included in different areas of production process such as in engineering design,
production planning, shop control, order processing, material control, distribution and
many other areas. CIM process helps in rapid production and also reduces indirect costs.
CIM uses computers to control the entire production process. This integration allows the
processes to exchange information with each other and thus they become capable of
initiating actions. CIM helps in avoiding accumulation of materials resulting in better
throughput and better utilization of space. Bar coded labels that accompany materials
contain instructions for processing them which are read by sensing devices and display
the status on monitors. This information is available to all concerned personnel
responsible in planning, marketing and other activities so that they will be aware of the
status of any order. Thus, CIM is an important aspect of technology in manufacturing.

Q2. What do you understand by ‘industry best practice’? Briefly explain different types of
Benchmarking.
Ans: Each industry would have developed over years or decades, during this period the materials
and process of production changes from time to time as all products or services are meant
to serve needs of the customers, they undergo continuous changes both in shapes and
features. The companies that were are the front innovates to stay in business as new
entrants would be adopting the latest techniques that the pioneers had taken decades to
establish. So, the various firms in any industry would end up adopting almost similar
methods of getting an output required. Such practices would get refined to a great extent
giving rise to what we call industry best practices. This trend to get stabilized or changed
owing to the development or new equipments which are designed. Industry best practices
open up the field for benchmarking by companies which need to improve their
performance.
Benchmarking is used to understand how these got into the system and what circumstances
brought them about. It is a learning process, with a view to find out whether some of the
reasons have changed and to bring in new processes for improvement. The metrics that
could be used are a) Number of pieces per hour, b) Cost per unit, c) Number of breakdowns
per week, d) Customer alienation during a week, e) Return on investment, f) Number of

Chandan Kumar Sah Page 4 of 152 MBA 2ND Semester


returns from customers in a month, g) Inventory turnover, h) Many others. The figures
obtained from the above determine the efficiency of the organization. The following are the
types of benchmarking considered by various firms:
i) Process Benchmarking – Business Process
ii) Financial Benchmarking
iii) Performance Benchmarking
iv) Product Benchmarking
v) Strategic Benchmarking
vi) Functional Benchmarking
Planning, Analysis, Integration, and Action are the four steps recognized in the process of
benchmarking. The select criteria are compared with the performance parameters of the
company which is considered the best in the industry. Targets are set and activities are
conducted to reach them. 1) Planning determines the process, service or the product to be
benchmarked on which metrics are assigned for collection of data. 2) Analyzed data gives
inputs for comparison with the target company’s performance on the parameter benchmark
on which data was collected. Measuring gaps helps in identifying the process which should
be improved for reaching the benchmark. 3) Integration: Resources are required across all
functions to achieve the target needs. Integration involves putting together resources like
people, equipments, and communication, so that, progress is unhindered and all activities
reach their logical conclusions without loss of initiative or time. 4) Action: When changes
are needed, actions have to be planned according to the steps earlier started. Teams are
provided with necessary leadership, authority, and supporting facilities to enable them to
complete all activities within the time frame set for the purpose.

Q3. List out the various automated systems for transfer of materials in the production
plant. What do you understand by Line Balancing? Explain with an example.
Ans: Basically, automation system comes to reduce labour power and time in the production.
Here we can see the evolution systems with some examples. The goods requited by society
were produces in small quantities by craftsman who would know the need of the
community and produced them by their own hands with simple tools. The apprentices or by
another craftsman, who would make them to meet the requests made. The parts and
components used to make these machines had to be replaced when they wore making parts

Chandan Kumar Sah Page 5 of 152 MBA 2ND Semester


so that interchangeability was achieved made setting up standards and specifications
important for meeting. The craftsmen gave way to engineers, workers, superiors and
inspectors. Division of labour became necessary to achieve efficiencies and the jobs that
became specialized. Competition has necessitated improved quality, reduced sates and
better services to the customer. Automation systems cost huge sums of money and
therefore a deep analysis of the various factors has to be done. For services, automation
usually means labour saving devices in education, long distance learning technology helps
in supplementing class room instruction. The facilitating goods that are used are website
and videos. Automation in the banking sector has resulted in ATMs which same the banks
a huge amount customer satisfaction. Automation is ideas when the service provided or the
product manufactured is highly standardized. Some extent of automation can be designed
even with customization i. e. product or service a meant to produce or deliver low volume
specific to a requirement. The advantages of automation are it has low variability and will
be more consistent on a repetitive basis. The machines have sensing and control devices
that enable them to operate automobile. The simplest of them called machine attachments
replace human effort. They guide, locate, move and achieve revise position by means of
came, optical sensing. Load sensing mechanisms and activate the controls to remove
human intention. Robots are higher in the order of automation as they perform a variety of
tasks. They are designed to move movements according to programmers written into the
computer that inside them. With the help of automation, inspection of component can be
done 100% ensures highest quality identification and movement of materials are helped by
bar codes which are read and fed into the system for monitoring quantity, location,
movement etc. They help the automated systems to sort information and provide
information for effecting any changes necessary. To make effective use of automated
machines, we need to have the movement of materials from and to different stations as
stores, automated. Automated storage and Retrieval systems (ASRS) receive orders for
materials from anywhere in the production area, collect materials and deliver materials to
the workstations. Computers and information systems are used for placing orders for
materials, giving commands and adjusting inventory records which show the location and
quantity of materials needed. Automated guided vehicle systems (AGVS) are pallet trucks
and unit load carriers follow embedded guide wires or paint strips to reach destinations as

Chandan Kumar Sah Page 6 of 152 MBA 2ND Semester


programmed. In an automobile showroom we can see all the work automatically with latest
machine.
Line Balancing: Production lines have a number of work centers in a particular sequence
so that the material that gets processed has to move further without encountering any
bottlenecks. The quantities produced the rate of production at each center, the number of
operations and the total production required are factors taken into account. The purpose of
balancing is to see that shortages occur between work centers and minimum inventory gets
created. We use the principles of JIT and lean Manufacturing to achieve these. Linear
programming, Dynamic programming and other mathematical models are used to study
material flow problems.

Q4. Explain the different types of Quality Control Tools with examples? How do Crosby’s
absolutes of quality differ from Deming’s principles?
Ans: The different types of Quality Control Tools are Flow Chart, Check Sheet, Histogram,
Pareto Analysis, Scatter Diagram, Control Chart, and Cause and Effect Diagram.
a) Flow Chart: Flow chart is a visual representation of process showing the various steps.
It helps in locating the points at which a problem exists or an improvement is possible.
Detailed data can be collected, analyzed, and methods for correction can be developed
using flow charts. Criteria and consequences that go with decisions are amenable to
evaluation for purposes of assessing quality. The flow chart helps in pin-pointing the
exact points at which errors have crept in.
b) Check Sheet: Check sheets are used to record the number of defects, types of defects,
locations at which they are occurring, times at which they are occurring, and workmen
by whom they are occurring. The sheet keeps a record of the frequencies of occurrence
with reference to possible defect causing parameters. It helps to implement a corrective
procedure at the point where the frequencies are more.
c) Histogram: Histograms are graphical representations of distribution of data. They are
generally used to record huge volumes of data about a process. They reveal whether the
pattern of distribution has a single peak, or many peaks and also the extent of variation
around the peak value. This helps in identifying whether the problem is serious. When
used in conjunction with comparable parameters, the visual patterns help us to identify
the problem which should be attended to.

Chandan Kumar Sah Page 7 of 152 MBA 2ND Semester


d) Pareto Analysis: Pareto analysis is a tool for classifying problem areas according to the
degree of importance and attending to the most important. Pareto principle, also called
80-20 rule, states that 80 percent of the problems that we encounter arise out of 20
percent of items. If we find that, in a day we have 156 assemblies having problems and
there are 9 possible causes, it is observed that 80 percent of them, that is 125 of them
have been caused by just 2 or 3 of them. It will be easy to focus on these 2 or 3 and
reduce the number of defects to a great extent. When the cause of these defects has
been attended, we will observe that some other defect becomes predominantly observed
and if the process is continued, we are marching toward zero defects.
e) Scatter Diagram: Scatter Diagram is used when we have two variables and want to
know the degree of relationship between them. We can determine if there is cause and
effect relationship between the variables and the degree of extent over a range of values
of the variables. Sometimes, we can observe that there is no relationship, in which we
can change one parameter being sure that it has no effect on the other parameter.
f) Control Charts: Control charts are used to verify whether a process is under control.
Variables, when they remain within a range, will render the product and maintain the
specifications. This is called the quality of conformance. The range of permitted
deviations is determined by design parameters. Samples are taken and the mean and
range of the variable of each sample is recorded. The mean of the means of the samples
gives the control lines.
g) Cause and Effect Diagram: Cause and effect diagram represents all the possible causes
which lead to a defect on quality characteristics. These are arranged in such a way that
different branches representing causes connect the stem in the direction of the
discovery of the problem. When each of them is investigated thoroughly we will be
able to pin-point some factors which cause the problem. When we observe that we have
excessive defects from a machine, we try to identify all possible sources of the causes
of defects.

Deming Wheel:-Deming’s approach is summarized in his 14 points.

1. Constancy of purpose for continuous improvement

2. Adopt the TQM philosophy for economic purposes

Chandan Kumar Sah Page 8 of 152 MBA 2ND Semester


3. Do not depend on inspection to deliver quality

4. Do not award any business based on price alone

5. Improve the system of production and service constantly

6. Conduct meaningful training on the job

7. Adopt modern methods of supervision and leadership

8. Remove fear from the minds of everyone connected with the organisation

9. Remove barriers between departments and people

10. Do not exhort, repeat slogans and put up posters.

11. Do not set up numerical quotas and work standards

12. Give pride of workmanship to the workmen

13. Education and training to be given vigorously


14. State and exhibit top management’s commitment for quality and productivity

Using the above principles, Deming gave a four step approach to ensure a purposeful
journey of TQM. The slope is shown to indicate that if efforts are let up the programme
will roll back.
Plan – means that a problem is identified, processes are determined and relevant theories
are checked out.
Do – means that the plan is implemented on a trial basis. All inputs are correctly measured
and recorded.
Check – means that the trials taken according to the plan are in accordance with the
expected results.
Act – When all the above steps are satisfactory regular production is started so that quality
outcomes are assured

Crosby’s Absolutes of Quality


Like Deming, he also lays emphasis on top management commitment and responsibility for
designing the system so that defects are not inevitable. He urged that there be no restriction

Chandan Kumar Sah Page 9 of 152 MBA 2ND Semester


on spending for achieving quality. In the long run, maintaining quality is more economical
rather than compromising on its achievement.
His absolutes can be listed as under.
(i) Quality is conformance to requirements – not ‘goodness’.
(ii) Prevention, not appraisal, is the path to quality.
(iii) Quality is measured as the price paid for non-conformance and as indexes.
(iv) Quality originates in all factions – not quality department. There are no quality
problems people, design, process create problems.
Crosby also has given 14 points similar to those of Deming. His approach emphasizes on
measurement of quality, increasing awareness, corrective action, error cause removal and
continuously reinforcing the system, so that advantages derived are not lost over time. He
desires that the quality management regimen should improve the overall health of the
organization and prescribed a vaccine.
The Ingredients are:
1. Integrity:- Honesty and commitment to produce everything right first time, every time.
2. Communication:- Flow of information between departments, suppliers, customers –
helps in identifying opportunities.
3. Systems and operations:- These should bring in a quality environment – so that nobody
is comfortable with anything less than the best.

Q5 Define project cycle, project management, and scope of project. List the various
project management knowledge areas? What are the reasons for failure of a project?
Ans: Definitions
a) Project Cycle: A project cycle consists of the various activities of operations, resources,
and the limitations imposed on them.
b) Project Management: Project management is the application of knowledge, skills,
tools, and techniques to project activities to meet project requirements.
c) Scope of Project: Scope of project refers to the various parameters that affect the
project in its planning, formulation, and executions.
Various Project Management Knowledge areas are:
• Project Integration management, cost management, communications management
• Project scope management, quality management, risk management

Chandan Kumar Sah Page 10 of 152 MBA 2ND Semester


• Project time management, human management, procurement management
Reasons for failure of a project is as under:
• Incidence of Project failure: It may be due to
a) Projects being initiated at random at all levels
b) Project objective not in line with business objective
c) Project management not observed
d) Project manager with no prior experience in the related project
e) Non-dedicated team
f) Lack of complete support from clients
• Factors contributing to project success not emphasized:
a) Project objective in alignment with business objective
b) Working within the framework of project management methodology
c) Effective scoping, planning, estimation, executing, controls and reviews, closure
intertwined with quality
d) Proactive approach towards project bottlenecks
e) Communication and managing expectations effectively with clients, team members,
and stake holders
f) Prior experience of project manager in a similar project
• Overview of Information and Communication Technologies projects:
a) Involve information and communications technologies such as the World Wide
Web, e-mail, fiber-optics, and satellites
b) Enable societies to produce, access, adapt and apply information in great amounts,
more rapidly and at reduced costs
c) Offer enormous opportunities for enhancing business and economic viability
• Common problems encountered during projects:
a) No prioritization of project activity from an organizational position
b) One or more of the stages in the project mishandled
c) Less qualified/non-dedicated manpower
d) Absence of smooth flow of communication between the involved parties

Q6 Explain the various phases in project management life cycle. Explain the necessity
and objectives of SCM.

Chandan Kumar Sah Page 11 of 152 MBA 2ND Semester


Ans: The Various phases in Project management life cycle are:
a) Analysis and Evolution phase: This is the initial phase of any project. In this phase
information is collected from the customer pertaining to the project and the
requirements are analyzed. The entire project has to be planned and it should be
done in a strategic manner. The project manager conducts the analysis of the
problem and submits a detailed report to the top management. The report of
Project Manager Analyst should consists of the details of the project
justification, details on what the problem is, method of solving the problem, list
of the objectives to be achieved, Estimation of project budget, success rate of
completing the project. The report must also contain information and the project
feasibility, and the risks involved in the project. Project management life cycle is
the integrated part of management. It is attached with project responsibility or
failure of a project.

The important tasks of this phase are as follows:


Specification Requirements Analysis: It has to be conducted to determine the
essential requirements of a project in order to achieve the target.
Feasibility study: To analyze whether the project is technically, economically
and practically feasible to be undertaken.
Trade off analysis: To understand and examine the various alternatives which
could be considered for solving the problem.
Estimation: To estimate the project cost, effort requires for the project and
functionality of various process in the project.
System design: Choose a general design that can fulfill the requirements.
Project evolution: Evaluate the project in terms of expected profit, cost and risks
involved.
b) Marketing Phase: A project proposal is prepared by a group of people including
the project manager. This proposal has to contain the strategies adopted to
market the product to the customers.

Chandan Kumar Sah Page 12 of 152 MBA 2ND Semester


c) Design Phase: This phase involves the study of inputs and outputs of the various
project stages.
d) Execution Phase: In this phase the project manager and the team members work
on the project objectives as per the plan. At every stage during the execution
reports are prepared.
e) Control - Inspecting, testing and delivery phase: The project team works under
the guidance of the project manager. The project manager has to ensure that the
team working under him is implementing the project designs accurately, the
project manager has to ensure ways of managing the customer and marketing he
future work, as well as ways to perform quality control work.
f) Closure and post completion analysis phase: Upon satisfactory completion
and delivery of the intended product or service the staff performance has to be
evaluated. The project manager has to document the lessons from the project.
Prepare the reports on project feedback analysis followed by the project
execution report. The phase which involve in the above are: The preparation
stage involves the preparation and approval of project outline, project plan and
project budget. The next stage involves selecting and briefing the project team
about the proposals followed by discussions on the roles and responsibility of the
project member and the organization.

SCM: SCM is the abbreviation of supply chain Management. It is considered by


many express worldwide as the ultimate solution towards efficient enterprise
management. Now, we explain the necessity and objectives of SCM-
SCM is required by and enterprise as a tool to enhance management effectiveness
with a following organizational objective:
• Reduction of inventory
• Enhancement of participation level and empowerment level

Chandan Kumar Sah Page 13 of 152 MBA 2ND Semester


• Increase in functional effectiveness of existing systems like Enterprise Resources
Planning, Accounting Software, and Documentation such as Financial reports/
Statements/ ISO 9000 Documents
• Effective integration of multiple systems like ERP, communication systems,
documentation system and security
• Design R&D systems etc.
• Better utilization of resources like men, material, equipment and money.
• Optimization of money flow cycle within the organization as well as from external
agencies.
• Enhancement of value of products, operations and services and consequently,
enhancements of profitability.
• Enhancement of satisfaction level of customer and clients, supporting institutions,
statutory control agencies, supporting institutions, statutory control agencies,
suppliers and vendors, employees and executives.
• Enhancement of flexibility in the organization to help in easy implementation of
schemes involving modernization, expansion and divestment, merges and
acquisitions.
• Enhancement of coverage and accuracy of management information systems.

With the objectives of SCM its implementation are required. Implementation is in


the form of various functional blocks of an organization interpenetrated through
which a smooth flow of the product development is possible. A relatively new SCM
option involves web based software with a browser interface. Several electronic
market places for buying and selling goods and materials.

Chandan Kumar Sah Page 14 of 152 MBA 2ND Semester


Assignment Set – 2

Q1. Take an example of any product or service industry and explain the factors
considered while taking the decision on plant location.
Ans: Selection of plant location for establishing a chemical industry:-
The geographical location of the plant contributes a lot to the success of any chemical
business venture .Utmost care and judgment is required for selecting the plant site, and
many different factors must be considered while selecting the plant site. The plant site
should be ideally located where the cost of production and distribution can be at a
minimum level .Also there has to be a good scope for plant expansion and a conductive
environment, safe living conditions for easy plant operation. But other factors, such as safe
living conditions for plant personnel as well as the surrounding community are also
important. The major factors in the selection of chemical plant sites are:-
(1) raw materials, (2) markets, (3) energy supply, (4) climate, (5) transportation facilities,
and (6)water supply. For a preliminary survey, the first four factors should be considered.
On the basis of raw materials availability, market survey, energy supply, and climate,
acceptable locations can usually be reduced to one or two general geographical regions.

In the second step, the effects of transportation facilities and water supply are taken into
account. This permits reduction of the possible plant location to few general target areas.
These areas can be reduced further by considering all the factors that have an influence on
plant location

As a third step, a detailed analysis of the remaining sites can be made. Exact data on items
such as freight rates, labor conditions, tax rates, price of land, and general local conditions
can be obtained. The various sites can be inspected and appraised on the basis of all the
factors influencing the final decision. The final decision on selecting the plant site should
take into consideration all the factors that can affect the ultimate success of the overall
plant operation.

The choice of the final site should be based on a detailed survey of various geographical
areas, and ultimately, on the advantages and disadvantages of available real estate. An

Chandan Kumar Sah Page 15 of 152 MBA 2ND Semester


initial outline regarding the plant location should be obtained before a design project
reaches the detailed estimate stage, and a firm location should be established upon
completion of the detailed estimate design The factors that must be evaluated in a plant
location study indicate the need for a vast amount of information.

The following factors should be considered in selecting a plant site:


1. Raw materials availability: The source of raw materials is one of the most important
factors influencing the selection of a plant site .This is particularly true if large number of
raw materials source permits considerable reduction in transport and storage charges.
Attention should be given to the purchased price of the raw materials, distance from the
source of the supply, freight or transportation expenses, availability and reliability of
supply ,purity of the raw materials, and storage requirements.
2. Energy availability: Power and steam requirements are high in most of the chemical
plants, and fuel is ordinarily required to supply these utilities .Power and fuel can be
combined as one major factor in the choice of a plant site. If the plant requires large
quantities of coal or oil, location near a source of fuel supply may be essential for economic
operation. The local cost of power can help determine whether power should be purchased
or self generated.
3. Meteorological data: If the plant is located in a cold climate, costs may be increased by
the necessity for construction of protective shelters around the process equipment, and
special cooling towers or air-conditioning equipment may be required if the prevailing
temperatures are high. Excessive humidity or extremes of hot or cold weather can have
serious effect on the economic operation of the plant, and these factors should be examined
when selecting a site.
4. Market study: The location of markets or distribution centers affects the cost of product
distribution and the time required for shipping .Proximity to the major markets is an
important consideration in the selection of a plant site, because the buyer usually finds it
advantageous to purchase from near by sources. It should be noted that markets are needed
for by products as well as for major final products.
5. Transportation facilities: Water, railroads, and highways are common means of
transportation used by major industrial concerns. The kind and quantity of products and
raw materials determine the most suitable type of transportation facilities. Careful attention

Chandan Kumar Sah Page 16 of 152 MBA 2ND Semester


should be given to local freight rates and existing railroad lines. The proximity to railroad
centers and the possibility of canal, river, lake or ocean transport must be considered.
Motor trucking facilities are widely used and can serve as a useful supplement to rail and
water facilities. If possible, the plant site should have access to all three types of
transportation and, certainly, at least two types should be available. There is usually a need
for convenient air and rail transportation facilities between the plant and the company head
quarters, and effective transportation facilities for the plant personnel are necessary.
6. Water supply: The process industries use large quantities of water for cooling, washing,
steam generation, and as a raw material in process. Hence, the plant must be located where
a dependable supply of water is available. A large river or lake is preferable, although deep
wells or artesian wells may be satisfactory if the amount of water required is not too great.
The level of the existing water table can be checked by consulting the state geological
survey ,and information on the constancy of the water table and the year round capacity of
local rivers or lakes should be obtained .If the water supply shows seasonal fluctuations, it
may be desirable to construct a reservoir or to drill several standby wells. The temperature,
mineral content, silt or sand content, bacteriological content, and cost for supply and
purification must also be considered when choosing the water supply.
7. Waste disposal: In recent years, many legal restrictions have been placed on the methods
for disposing of waste materials from the process industries. The site selected for a plant
should have adequate capacity and facilities for correct waste disposal. In choosing a plant
site, the permissible tolerance levels for various methods of waste disposal should be
considered carefully, and attention should be given to potential requirements for additional
waste-treatment facilities.
8. Labor supply: The type and supply of labor available in the vicinity of a proposed plant
site must be examined .Consideration should be given to prevailing pay scales ,restrictions
on number of hours worked per week, competing industries that can cause dissatisfaction
or high turnover rates among the workers, and variations in the skill and productivity of the
workers.
9. Taxation and legal restrictions: State and local tax rates on property income,
unemployment insurance and similar items vary from one location to another. Similarly,
local regulations on zoning, building codes, nuisance aspects, and transportation facilities
can have a major influence on the final choice of a plant site. In fact, zoning difficulties and

Chandan Kumar Sah Page 17 of 152 MBA 2ND Semester


obtaining the many required permits can often be much more important in terms of cost and
time delays than many of the factors discussed in the preceding sections.
10. Site characteristics: The characteristics of the land at a proposed plant site should be
examined carefully. The topography of land and the soil structure must be considered, since
either or both may have a pronounced effect on construction costs. The cost of the land is
important, as well as local building costs and living conditions. Future changes may make
it desirable or necessary to expand the plant facilities. Therefore, even though no
immediate expansion is planned, a new plant should be constructed at a location where
additional space is available.
11. Safety and Environmental measures: Many industrial plants are located along rivers or
near large bodies of water, and there are risks of flood or hurricane damage. Before
selecting a plant site, the regional history of natural events of this type should be examined
and the consequences of such occurrences considered. Protection from losses by fire is
another important factor for selecting a plant location. In case of a major fire, assistance
from the fire departments should be available. Fire hazards in the surrounding area of plant
site must not be overlooked.
12. Community factors: The nature and facilities of a community can have an effect on the
location of the plant. If minimum number of facilities for satisfactory living of plant
personnel do not exist, it becomes a burden for the plant to subsidize such facilities.
Cultural facilities of the community are important to sound growth. Facilities such as
religious centers, libraries, schools, civic theatres, concert associations, and other similar
groups do much to make a community progressive. The efficiency, character, and history
of both state and local governments should be evaluated. The existence of low taxes is not
in itself a favorable situation unless the community is already well developed and relatively
free of debt.

Q2. What is Business Process? Explain with an example as to why a business process is to
be modelled.
Ans: A Business process is a process designed to achieve a particular business objective. It refers
to the techniques and activities used as part of Business Process Management discipline.
Management process gets exhibited in corporate governance and strategic management.
The operational component of Management consists of workflow across the functions

Chandan Kumar Sah Page 18 of 152 MBA 2ND Semester


which realize the objectives of the business. Business Process is a total response that a
business undertakes utilizing the resources and delivering the outputs that create a value for
the customer. The business process has a goal, uses specific inputs, delivers specific
outputs, collects resources, performs a number of activities in some order, and creates value
for the customer.
Business Process Modeling refers to a set of activities undertaken to optimize the business
process. The reasons for optimizing the business process is to improve the performance of
the process, deliver better value for the customer, maximize the earnings of the
organization, keep its head above competition form outsiders.

Q3. What is Value Engineering? Explain briefly its origins and objectives.
Ans: Value Engineering is a methodology by which we try to find substitutes for a product or an
operation. The concept of value engineering originated during the Second World War, it
was developed by the General Electric Corporations. Value Engineering has gained
popularity due to its potential for gaining high Returns on Investment. This methodology is
widely used in business re-engineering, government projects, automakers, transportation
and distribution, industrial equipment, construction, assembling and machining processes,
health care and environmental engineering, and many others. Value engineering process
calls for a deep study of a product and the purpose for which it is used, such as, the raw
materials used; the processes of transformation; the equipment needed, and many others.
This applies to all aspects of the product. The different aspects of value engineering can be
encapsulated into a sequence of steps known as a ‘Job Plan’. Value Engineering in
organizations helps to identify: a) The problem or situation that needs to be
changed/improved, b) All that is good about the existing situation, c) The improvements
required in the situation, d) The functions to be performed, e) The ways of performing each
function, f) The best ways among the selected function, g) The person who executes the
function.

Q4. Discuss the difference between work study and motion study. Explain with an
example.
Ans: Work Study: Work Study is being conducted when analysis of work methods is conducted
during the period when a job is done on a machine or equipment. It helps in designing the

Chandan Kumar Sah Page 19 of 152 MBA 2ND Semester


optimum work method and standardization of the work method, it enables the methods
engineer to search for better methods for higher utilization of man and machine and
accomplishment of higher productivity. It gives an opportunity to the workmen to learn the
process of study thus making them able to offer suggestions for improved methods. This
encourages workmen participation and they can be permitted to make changes and report
the advantages that can be derived from those. This course is in alignment with the
principle of continuous improvement and helps the organization in the long run. It
comprises of work measurement and method study. Work measurement focuses on the time
element of work, while method study focuses on the methods deployed and development of
better methods.
Methods Study: Method study focus is on studying the method currently being used and
developing a new method of performing the task in a better way. Operation Flow Chats,
Motion Charts, Flow Process charts, which are the elements of the task, are studied to find
the purpose of each activity, the sequence in which they are done, and the effect of these on
the work. The study may help in changing some of them and even eliminate some of them
to effect improvements. The new method should result in saving of time, reduced motions,
and simpler activities.

Q5. Time taken by three machines on five jobs in a factory is tabulated below in table
below. Find out the optimal sequence to be followed to minimize the idle time taken
by the jobs on the machines.
Job Machine 1 Machine 2 Machine 3
(M1) (M2) (M3)
A 6 8 7
B 4 5 3
C 5 5 7
D 3 4 6
E 4 3 4

Ans: Steps to be taken to solve the algorithm are as follows:


1) Take column M1 and M3, ignoring M2. Get the sequence by applying Johnson’s rule.
2) Next combine M1 and M2 – Make it as one machine MX and combine M2 and M3 and
call it MY. Get the sequence.

Chandan Kumar Sah Page 20 of 152 MBA 2ND Semester


3) Calculate the total time taken to process all jobs A, B, C, D and E. Choose whichever
given the total time.
4) According to the steps mentioned above the job order sequence of machines 1 and 2,
that is MA can be derived with the help of the following table.
Sequence of MA
Job Machine 1 Machine 3
(M1) (M3)
A 6 7
B 4 3
C 5 7
D 3 6
E 4 4

Using Johnson’s algorithm, the job order sequence obtained is:


MA = D E C A B
The time taken (TA) by sequence MA is now calculated as shown in table below.
Job Processing Time
Machine 1 Machine 2 Machine 3
Time in Time out Time in Time out Time in Time out
D 0 3 3 7 7 13
E 3 7 7 10 10 14
C 7 12 12 17 17 24
A 12 18 18 26 26 33
B 18 22 22 27 27 30

The time taken (TA) to complete the sequence obtained above is 30.
The job order sequence of machine (1,2) and (2,3), that is MB can be derived with the help
of the following table

Chandan Kumar Sah Page 21 of 152 MBA 2ND Semester


Sequence of MB
Job M1 + M2 M2 + M3
A 14 15
B 9 8
C 10 12
D 7 10
E 7 7

Using Johnson’s algorithm, the job order sequence obtained is:


MB = E D C A B
The time taken (TB) by sequence MB is now calculated as shown in table below.
Job Processing Time
Machine 1 Machine 2 Machine 3
Time in Time out Time in Time out Time in Time out
E 0 4 4 7 7 11
D 4 7 7 11 11 17
C 7 12 12 17 17 24
A 12 18 18 26 26 33
B 18 22 22 27 27 30

The time taken (TB) to complete the sequence obtained above is 30.
Thus, we get values : TA = 30, TB = 30
From the two values: TA and and TB both is the same. Therefore, the optimal sequence for
the given 5 jobs under the 3 machines is both MA & MB.
MA = D E C A B
MB = E D C A B

Q6. List the seven principles of SCM. Discuss Bullwhip effect, its effects and
countermeasures.
Ans: Seven principles of SCM are:
1. Group customer by needs:- Effective SCM groups, customer by distinct service needs,
regardless of industry and then tailors services to this particular segment.
2. Customize the logistic network:- In designing their logistics network, companies need
to focus on the service requirement and profit of the customer segments identified.
3. Listen to signals of market demand and plan accordingly:- Sales and operations
planners must monitor the entire supply chain to detect early warning signals of

Chandan Kumar Sah Page 22 of 152 MBA 2ND Semester


changing customer demand and needs. This demand driven approach leads to more
consistent forecast and optimal resource allocation.
4. Differentiate the product closer to the customer:- Companies today no longer can
afford to stockpile inventory to compensate for possible forecasting errors. Instead, they
need to postpone product differentiation in the manufacturing process closer to actual
consumer demand. This strategy allows the supply chain to respond quickly and cost
effectively to change in customer needs.
5. Strategically manage the sources of supply:- By working closely with their key
suppliers to reduce the overall costs of owning materials and services; SCM maximizes
profit margins both for themselves and their suppliers.
6. Develop a supply chain wide technology strategy:- As one of the cornerstones of
successful SCM, information technology must be able to support multiple levels of
decision making. It also should afford a clear view and ability to measure the flow of
products, services and information.
7. Adopt channel spanning chain performance measures:- Excellent supply chain
performance measurement system do more than just monitor internal functions. They
apply performance criteria to every link in the supply chain - criteria that embrace both
service and financial metrics.

Bullwhip Effect:- An unmanaged supply chain is not inherently stable. Demand variability
increases as one move up the supply chain away from the retail customer, and small
changes in consumer demand can result in large variations in orders placed upstream.
Eventually, the network can oscillate in very large swings as each organization in the
supply chain seeks to solve the problem from its own perspective. This phenomenon is
known as the bullwhip effect and has been observed across most industries, resulting in
increased cost and poorer service.

Chandan Kumar Sah Page 23 of 152 MBA 2ND Semester


Causes of the Bullwhip Effect:

Sources of variability can be demand variability, quality problems, strikes, plant fires, etc.
Variability coupled with time delays in the transmission of information up the supply chain
and time delays in manufacturing and shipping goods down the supply chain create the
bullwhip effect. The following all can contribute to the bullwhip effect:

• Overreaction to backlogs
• Neglecting to order in an attempt to reduce inventory
• No communication up and down the supply chain
• No coordination up and down the supply chain
• Delay times for information and material flow
• Order batching - larger orders result in more variance. Order batching occurs in an effort
to reduce ordering costs, to take advantage of transportation economics such as full
truck load economies, and to benefit from sales incentives. Promotions often result in
forward buying to benefit more from the lower prices.
• Shortage gaming: customers order more than they need during a period of short supply,
hoping that the partial shipments they receive will be sufficient.
• Demand forecast inaccuracies: everybody in the chain adds a certain percentage to the
demand estimates. The result is no visibility of true customer demand.
• Free return policies

Countermeasures to the Bullwhip Effect

While the bullwhip effect is a common problem, many leading companies have been able
to apply countermeasures to overcome it. Here are some of these solutions:

• Countermeasures to order batching - High order cost is countered with


Electronic Data Interchange (EDI) and computer aided ordering (CAO). Full truck load
economics are countered with third-party logistics and assorted truckloads. Random or
correlated ordering is countered with regular delivery appointments. More frequent
ordering results in smaller orders and smaller variance. However, when an entity orders
more often, it will not see a reduction in its own demand variance - the reduction is
seen by the upstream entities. Also, when an entity orders more frequently, its required

Chandan Kumar Sah Page 24 of 152 MBA 2ND Semester


safety stock may increase or decrease; see the standard loss function in the Inventory
Management section.
• Countermeasures to shortage gaming - Proportional rationing schemes are
countered by allocating units based on past sales. Ignorance of supply chain conditions
can be addressed by sharing capacity and supply information. Unrestricted ordering
capability can be addressed by reducing the order size flexibility and implementing
capacity reservations. For example, one can reserve a fixed quantity for a given year
and specify the quantity of each order shortly before it is needed, as long as the sum of
the order quantities equals to the reserved quantity.
• Countermeasures to fluctuating prices - High-low pricing can be replaced with
every day low prices (EDLP). Special purchase contracts can be implemented in order
to specify ordering at regular intervals to better synchronize delivery and purchase.
• Countermeasures to demand forecast inaccuracies - Lack of demand visibility
can be addressed by providing access to point of sale (POS) data. Single control of
replenishment or Vendor Managed Inventory (VMI) can overcome exaggerated
demand forecasts. Long lead times should be reduced where economically
advantageous.
• Free return policies are not addressed easily. Often, such policies simply must be
prohibited or limited.

Chandan Kumar Sah Page 25 of 152 MBA 2ND Semester


Chandan Kumar Sah Page 26 of 152 MBA 2ND Semester
Assignment Set – 1

Q1. Write the short notes on


1. Financial management
2. Financial Planning
3. Capital Structure
4. Cost of Capital
5. Trading on equity.
Ans: 1. Financial Management: Financial Management is the art and science of managing
money. Regulatory and economic environments have undergone drastic changes due to
liberalization and globalization of Indian economy. Indian financial managers have
transformed themselves from licensed raj managers to well-informed dynamic proactive
managers capable of taking decisions of complex nature. Traditionally, Financial
Management was considered a branch of knowledge with focus on the procurement of
funds. Instruments of financing, formation, merger and restructuring of firms and legal and
institutional frame work occupied the prime place in this traditional approach. The modern
approach transformed the field of study from the traditional narrow approach to the most
analytical nature. The core of modern approach evolved around the procurement of the
least cost funds and its effective utilization for maximization of share holders’ wealth.

2. Financial Planning: Financial planning is a process by which funds required for each
course of action is decided. A financial plan has to consider capital structure, capital
expenditure and cash flow. Decisions on the composition of debt and equity must be taken.
Financial Planning indicates a) the quantum of funds required to execute business plans, b)
composition of debt and equity, keeping in view the risk profile of the existing business,
new business to be taken up and the dynamics of capital market conditions, c) formulation
of policies, giving effect to the financial plans under consideration.

3. Capital Structure: Capital structure refers to the mix of long-term finances used by the
firm. It is the financial plan of the company. The capital structure should add value to the
firm. Financial mix decisions are investment decisions and have no impact on the operating
earnings of the firm. Such decisions influence the firm’s value through the earnings

Chandan Kumar Sah Page 27 of 152 MBA 2ND Semester


available to the shareholders. The value of a firm is dependent on its expected future
earnings and the required rate of return. The objective of any company is to have an ideal
mix of permanent sources of funds in a manner that it will maximize the company’s market
price. The proper mix of funds is referred to as optimal capital structure. The capital
structure decisions include debt-equity mix and dividend decisions. Bothe these have an
effect on the EPS.

4. Cost of Capital: Cost of capital is the minimum required rate of return needed to justify
the use of capital. A company obtains resources from various sources – issue of debentures,
availing term loans from banks and financial institutions, issue of preference and equity
shares or it may even withhold a portion or complete profits earned to be utilized for future
activities.

5. Trading on equity: Trading equities involves more than stock trading. Equity trading in
the public markets can involve many different securities, requiring diverse strategies and
trading skills. Skilled traders will know the related trading strategies that complement their
objectives. Trading equity refers to the universe of stocks, options and preferred stock in
public markets. Equity trading specifically eliminates debt trading. Trading equities usually
takes place in public markets, both domestic and overseas. Equity trades can take place at
almost any time of the day or night. Trading securities not on a listed exchange involves
cost inefficiencies due to lack of liquidity. It is possible to trade equities through the futures
markets.

Q2. a. Write the feature of interim dividend and also write the factors influencing dividend
policy?
b. What is reorder level?
Ans: a) Interim Dividend: A dividend payment made before a company's AGM and final
financial statements. This declared dividend usually accompanies the company's interim
financial statements. A dividend which is declared and distributed before the company's
annual earnings have been calculated; often distributed quarterly. Usually, board of
directors of company declares dividend in annual general meeting after finding the real net
profit position. If boards of directors give dividend for current year before closing of that

Chandan Kumar Sah Page 28 of 152 MBA 2ND Semester


year, then it is called interim dividend. This dividend is declared between two annual
general meetings. Before declaring interim dividend, board of directors should estimate the
net profit which will be in future. They should also estimate the amount of reserves which
will deduct from net profit in profit and loss appropriation account. If they think that it is
sufficient for operating of business after declaring such dividend. They can issue but after
completing the year, if profits are less than estimates, then they have to pay the amount of
declared dividend. For this, they will have to take loan. Therefore, it is the duty of directors
to deliberate with financial consultant before taking this decision. Accounting treatment of
interim dividend in final accounts of company:
First Case: Interim dividend is shown both in profit and loss appropriation account and
balance sheet, if it is outside the trial balance in given question.
(a) It will go to debit side of profit and loss appropriation account
(b) It will also go to current liabilities head in liabilities side.
Second Case: Interim dividend is shown only in profit and loss appropriation account, if it
is shown in trial balance.
(a) It will go only to debit side of profit and loss appropriation account. If in final
declaration is given outside of trial balance and this will be proposed dividend and interim
dividend in trial balance will be deducted for writing proposed dividend in profit and loss
appropriation account and balance sheet of company, because if we will not deducted
interim dividend, then it will be double deducted from net profit that is wrong and error
shows when we will match balance sheets assets with liabilities. Usually, board of directors
of company declares dividend in annual general meeting after finding the real net profit
position. If boards of directors give dividend for current year before closing of that year,
then it is called interim dividend. This dividend is declared between two annual general
meetings. Before declaring interim dividend, board of directors should estimate the net
profit which will be in future. They should also estimate the amount of reserves which will
deduct from net profit in profit and loss appropriation account. If they think that it is
sufficient for operating of business after declaring such dividend. They can issue but after
completing the year, if profits are less than estimates, then they have to pay the amount of
declared dividend. For this, they will have to take loan. Therefore, it is the duty of directors
to deliberate with financial consultant before taking this decision.

Chandan Kumar Sah Page 29 of 152 MBA 2ND Semester


A number of considerations affect the dividend policy of company. The major factors are
1. Stability of Earnings: The nature of business has an important bearing on the dividend
policy. Industrial units having stability of earnings may formulate a more consistent
dividend policy than those having an uneven flow of incomes because they can predict
easily their savings and earnings. Usually, enterprises dealing in necessities suffer less from
oscillating earnings than those dealing in luxuries or fancy goods.
2. Age of corporation: Age of the corporation counts much in deciding the dividend policy.
A newly established company may require much of its earnings for expansion and plant
improvement and may adopt a rigid dividend policy while, on the other hand, an older
company can formulate a clear cut and more consistent policy regarding dividend.
3. Liquidity of Funds: Availability of cash and sound financial position is also an
important factor in dividend decisions. A dividend represents a cash outflow, the greater
the funds and the liquidity of the firm the better the ability to pay dividend. The liquidity of

a firm depends very much on the investment and financial decisions of the firm which in
turn determines the rate of expansion and the manner of financing. If cash position is weak,
stock dividend will be distributed and if cash position is good, company can distribute the
cash dividend.
4. Extent of share Distribution: Nature of ownership also affects the dividend decisions. A
closely held company is likely to get the assent of the shareholders for the suspension of
dividend or for following a conservative dividend policy. On the other hand, a company
having a good number of shareholders widely distributed and forming low or medium
income group, would face a great difficulty in securing such assent because they will
emphasis to distribute higher dividend.
5. Needs for Additional Capital: Companies retain a part of their profits for strengthening
their financial position. The income may be conserved for meeting the increased
requirements of working capital or of future expansion. Small companies usually find
difficulties in raising finance for their needs of increased working capital for expansion
programmers. They having no other alternative, use their ploughed back profits. Thus, such
Companies distribute dividend at low rates and retain a big part of profits.
6. Trade Cycles: Business cycles also exercise influence upon dividend Policy. Dividend
policy is adjusted according to the business oscillations. During the boom, prudent

Chandan Kumar Sah Page 30 of 152 MBA 2ND Semester


management creates food reserves for contingencies which follow the inflationary period.
Higher rates of dividend can be used as a tool for marketing the securities in an otherwise
depressed market. The financial solvency can be proved and maintained by the companies
in dull years if the adequate reserves have been built up.
7. Government Policies: The earnings capacity of the enterprise is widely affected by the
change in fiscal, industrial, labour, control and other government policies. Sometimes
government restricts the distribution of dividend beyond a certain percentage in a particular
industry or in all spheres of business activity as was done in emergency. The dividend
policy has to be modified or formulated accordingly in those enterprises.
8. Taxation Policy: High taxation reduces the earnings of he companies and consequently
the rate of dividend is lowered down. Sometimes government levies dividend-tax of
distribution of dividend beyond a certain limit. It also affects the capital formation. N India,
dividends beyond 10 % of paid-up capital are subject to dividend tax at 7.5 %.
9. Legal Requirements: In deciding on the dividend, the directors take the legal
requirements too into consideration. In order to protect the interests of creditors outsiders,
the companies Act 1956 prescribes certain guidelines in respect of the distribution and
payment of dividend. Moreover, a company is required to provide for depreciation on its
fixed and tangible assets before declaring dividend on shares. It proposes that Dividend
should not be distributed out of capita, in any case. Likewise, contractual obligation should
also be fulfilled, for example, payment of dividend on preference shares in priority over
ordinary dividend.
10. Past dividend Rates: While formulating the Dividend Policy, the directors must keep in
mind the dividend paid in past years. The current rate should be around the average past
rat. If it has been abnormally increased the shares will be subjected to speculation. In a new
concern, the company should consider the dividend policy of the rival organization.
11. Ability to Borrow: Well established and large firms have better access to the capital
market than the new Companies and may borrow funds from the external sources if there
arises any need. Such Companies may have a better dividend pay-out ratio. Whereas
smaller firms have to depend on their internal sources and therefore they will have to build
up good reserves by reducing the dividend pay out ratio for meeting any obligation
requiring heavy funds.

Chandan Kumar Sah Page 31 of 152 MBA 2ND Semester


12. Policy of Control: Policy of control is another determining factor is so far as dividends
are concerned. If the directors want to have control on company, they would not like to add
new shareholders and therefore, declare a dividend at low rate. Because by adding new
shareholders they fear dilution of control and diversion of policies and programmes of the
existing management. So they prefer to meet the needs through retained earning. If the
directors do not bother about the control of affairs they will follow a liberal dividend
policy. Thus control is an influencing factor in framing the dividend policy.
13. Repayments of Loan: A company having loan indebtedness are vowed to a high rate of
retention earnings, unless one other arrangements are made for the redemption of debt on
maturity. It will naturally lower down the rate of dividend. Sometimes, the lenders (mostly
institutional lenders) put restrictions on the dividend distribution still such time their loan is
outstanding. Formal loan contracts generally provide a certain standard of liquidity and
solvency to be maintained. Management is bound to hour such restrictions and to limit the
rate of dividend payout.
14. Time for Payment of Dividend: When should the dividend be paid is another
consideration. Payment of dividend means outflow of cash. It is, therefore, desirable to
distribute dividend at a time when is least needed by the company because there are peak
times as well as lean periods of expenditure. Wise management should plan the payment of
dividend in such a manner that there is no cash outflow at a time when the undertaking is
already in need of urgent finances.
15. Regularity and stability in Dividend Payment: Dividends should be paid regularly
because each investor is interested in the regular payment of dividend. The management
should, inspite of regular payment of dividend, consider that the rate of dividend should be
all the most constant. For this purpose sometimes companies maintain dividend
equalization Fund.

b) Reorder Level: The reorder level is that inventory level at which an order should be
placed to replenish the inventory. To arrive at the reorder level under certainty, Lead time
and Average usage are two key. Lead time refers to the average time required to replenish
the inventory after placing orders for inventory. Reorder level = Lead time X Average
usage. Under certainty, reorder point refers to that inventory level which will meet the
consumption needs during the lead time.

Chandan Kumar Sah Page 32 of 152 MBA 2ND Semester


Q3. Sales Rs. 4,00,000 less returns Rs. 10,000, Cost of Goods Sold Rs. 3,00,000,
Administration and selling expenses Rs. 20,000, Interest on loans Rs. 5,000, Income
Tax Rs. 10,000, Preference dividend Rs. 15,000, Equity Share Capital Rs. 1,00,000 @
Rs. 10 per share. Find EPS.
Ans: Sales = Rs. 4,00,000
Sales Returns = Rs. 10,000
Cost of Goods Sold = Rs. 3,00,000
Administration and selling expenses = Rs. 20,000
Interest on loans = Rs. 5,000
Income Tax = Rs. 10,000
Preference Dividend = Rs. 15,000
Equity Share Capital @ Rs. 10 per share = Rs. 1,00,000
No. of Equity Shares = Equity Share Capital / Rate per share
= 1,00,000 / 10
= 10,000 shares
Gross Profit = Sales – Sales Returns – Cost of Goods Sold
= 4,00,000 – 10,000 – 3,00,000
= 90,000
EBIT = Gross Profit – Administrative and selling Expenses
= 90,000 – 20,000
= 70,000
Net Earnings before Pref. Dividend = EBIT – Interest on loans – Income Tax
= 70,000 – 5,000 – 10,000
= 55,000
Net Earnings = Net earning before Pref. Dividend – Pref. Dividend
= 55,000 – 15,000
= 40,000
EPS = Net Earnings / No. of Shares
= 40,000 / 10,000
= Rs. 4

Chandan Kumar Sah Page 33 of 152 MBA 2ND Semester


Q4. What are the techniques of evaluation of investment?
Ans: Techniques of evaluation of investment:
Three steps are involved in the evaluation of an investment:
•Estimation of cash flows
•Estimation of the required rate of return (the opportunity cost of capital)
•Application of a decision rule for making the choice.
The first two steps, discussed in the subsequent chapters, are assumed as given. Thus, our
discussion in this chapter is confined to the third step. Specifically, we focus on the merits
and demerits of various decision rules.
Investment decision rule
The investment decision rules may be referred to as capital budgeting techniques, or
investment criteria. A sound appraisal technique should be used to measure the economic
worth of an investment project. The essential property of a sound technique is that it should
maximize the shareholders’ wealth. The following other characteristics should also be
possessed by a sound investment evaluation criterion.
• It should consider all cash flows to determine the true profitability of the project.
• It should provide for an objective and unambiguous way of separating good projects form
bad projects.
• It should help ranking of projects according to their true profitability.
• It should recognize the fact that bigger cash flows true profitability.
• It should recognize the fact that bigger cash flows are preferable to smaller once and early
cash flows are preferable to later ones.
• It should help top choose among mutually exclusive projects that project which
maximizes the shareholders’ wealth.
• It should be a criterion which is applicable to any conceivable investment project
independent of other.
These conditions will be clarified as we discuss the features of various investment criteria
in the following pages.
Evaluation criteria
A number of investments criteria (or capital budgeting techniques) are in use in proactive.
They may be grouped in the following two categories:
1. Discounted cash flow (DCF) criteria

Chandan Kumar Sah Page 34 of 152 MBA 2ND Semester


• Net present value (NPV)
• Internal rate of return (IIR)
• Profitability index (PI)
2. Non-discounted cash flow criteria
• Payback period (PB)
• Discounted payback period
• Accounting rate of return (ARR).
Discounted payback is a variation of the payback method. It involves discounted cash
flows, but as we shall see later, it is not a true measure of investment profitability. We will
show in the following pages that the net present value criterion is the most valid technique
of maximizing the shareholders wealth.

Q5. What are the problems associated with inadequate working capital?
Ans: Working Capital is defined as the excess of current assets over current liabilities and
provisions. It is that portion of asset of a business which is used frequently in current
operations and in the operating cycle of the firm. Inadequacy of working capital is the
leading cause of many business failures. A financial manager, therefore, spends a large part
of his time in managing working capital. There are two important elements to be
considered under the working capital management: i) Decisions on the amount of current
assets to be held by a firm for efficient operations of its business, ii) Decisions on financing
working capital requirement. The need for proper management of working capital
management is even more important in the modern era of information technology. Working
capital may be regarded as the life blood of business. Working capital is of major
importance to internal and external analysis because of its close relationship with the
current day-to-day operations of a business. Every business needs funds for two purposes.
Long term funds are required to create production facilities through purchase of fixed
assets such as plants, machineries, lands, buildings & etc. Short term funds are required for
the purchase of raw materials, payment of wages, and other day-to-day expenses. . It is
other wise known as revolving or circulating capital. It is nothing but the difference
between current assets and current liabilities. i.e. Working Capital = Current Asset –
Current Liability.

Chandan Kumar Sah Page 35 of 152 MBA 2ND Semester


Businesses use capital for construction, renovation, furniture, software, equipment, or
machinery. It is also commonly used to purchase inventory, or to make payroll. Capital is
also used often by businesses to put a down payment down on a piece of commercial real
estate. Working capital is essential for any business to succeed. It is becoming increasingly
important to have access to more working capital when we need it.

Importance of Adequate Working Capital: A business firm must maintain an adequate


level of working capital in order to run its business smoothly. It is worthy to note that both
excessive and inadequate working capital positions are harmful. Working capital is just like
the heart of business. If it becomes weak, the business can hardly prosper and survive. No
business can run successfully without an adequate amount of working capital.

Danger of inadequate working capital: When working capital is inadequate, a firm faces
the following problems. Fixed Assets cannot efficiently and effectively be utilized on
account of lack of sufficient working capital. Low liquidity position may lead to liquidation
of firm. When a firm is unable to meets its debts at maturity, there is an unsound position.
Credit worthiness of the firm may be damaged because of lack of liquidity. Thus it will lose
its reputation. There by, a firm may not be able to get credit facilities. It may not be able to
take advantages of cash discount.

Disadvantages of Redundant or Excessive Working Capital:


1. Excessive Working Capital means ideal funds which earn no profits for the business and
hence the business cannot earn a proper rate of return on its investments.
2. When there is a redundant working capital, it may lead to unnecessary purchasing and
accumulation of inventories causing more chances of theft, waste and losses.
3. Excessive working capital implies excessive debtors and defective credit policy which
may cause higher incidence of bad debts.
4. It may result into overall inefficiency in the organization.
5. When there is excessive working capital, relations with banks and other financial
institutions may not be maintained.
6. Due to low rate of return on investments, the value of shares may also fall.
7. The redundant working capital gives rise to speculative transactions.

Chandan Kumar Sah Page 36 of 152 MBA 2ND Semester


Disadvantages or Dangers of Inadequate Working Capital:
1. A concern which has inadequate working capital cannot pay its short-term liabilities
in time. Thus, it will lose its reputation and shall not be able to get good credit facilities.
2. It cannot buy its requirements in bulk and cannot avail of discounts, etc.
3. It becomes difficult for the firm to exploit favorable market conditions and undertake
profitable projects due to lack of working capital.
4. The firm cannot pay day-to-day expenses of its operations and its creates inefficiencies,
increases costs and reduces the profits of the business.
5. It becomes impossible to utilize efficiently the fixed assets due to non-availability of
liquid funds.
6. The rate of return on investments also falls with the shortage of working capital.

Disadvantages or Dangers of Inadequate or Short Working Capital :

• Can’t pay off its short-term liabilities in time.


• Economies of scale are not possible.
• Difficult for the firm to exploit favorable market situations
• Day-to-day liquidity worsens
• Improper utilization the fixed assets and ROA/ROI falls sharply

Q6. What is leverage? Compare and Contrast between operating Leverage and financial
leverage.
Ans: Leverage is the influence of power to achieve something. The use of an asset or source of
funds for which the company has to pay a fixed cost or fixed return is termed as leverage.
Leverage is the influence of an independent financial variable on a dependent variable. It
studies how dependent variable responds to a particular change in independent variable.

Operating leverage arises due to the presence of fixed operating expenses in the firm’s
income flows. The operating leverage is the firm’s ability to use fixed operating costs to
increase the effects of changes in sales on its earnings before interest and taxes. Operating
leverage occurs any time a firm has fixed costs. The percentage change in profits with a
change in volume of sales in more than the percentage change in volume.

Chandan Kumar Sah Page 37 of 152 MBA 2ND Semester


Financial leverage as opposed to operating leverage relates to the financing activities of a
firm and measures the effect of earnings before interest and tax on earnings per share of the
company. Financial leverage refers to the mix of debt and equity in the capital structure of
the firm. This results from the presence of fixed financial charges in the company’s income
stream. Such expenses have nothing to do with the firm’s performance and earnings and
should be paid off regardless of the amount of earnings before income and tax. It is the
firm’s ability to use fixed financial charges to increase the effects of changes in EBIT on
the EPS. It is the use of funds obtained at fixed costs which increase the returns on
shareholders.

Chandan Kumar Sah Page 38 of 152 MBA 2ND Semester


Assignment Set – 2

Q1. Discuss the three board areas of Financial Decision Making.


Ans: Finance function is the most important function of a business. Finance is, closely,
connected with production, marketing and other activities. In the absence of finance, all
these activities come to a halt. In fact, only with finance, a business activity can be
commenced, continued and expanded. Finance exists everywhere, be it production,
marketing, human resource development or undertaking research activity. Understanding
the universality and importance of finance, finance manager is associated, in modern
business, in all activities as no activity can exist without funds. Financial Decisions or
Finance Functions are closely inter-connected. All decisions mostly involve finance. When
a decision involves finance, it is a financial decision in a business firm. In all the following
financial areas of decision-making, the role of finance manager is vital. We can classify the
finance functions or financial decisions into four major groups:
(A) Investment Decision or Long-term Asset mix decision
(B) Finance Decision or Capital mix decision
(C) Dividend Decision or Profit allocation decision
(A) Investment Decision : Investment decisions relate to selection of assets in which funds
are to be invested by the firm. Investment alternatives are numerous. Resources are scarce
and limited. They have to be rationed and discretely used. Investment decisions allocate
and ration the resources among the competing investment alternatives or opportunities. The
effort is to find out the projects, which are acceptable. Investment decisions relate to the
total amount of assets to be held and their composition in the form of fixed and current
assets. Both the factors influence the risk the organization is exposed to. The more
important aspect is how the investors perceive the risk. The investment decisions result in
purchase of assets. Assets can be classified, under two broad categories:
(i) Long-term investment decisions – Long-term assets
(ii) Short-term investment decisions – Short-term assets
Long-term Investment Decisions: The long-term capital decisions are referred to as capital
budgeting decisions, which relate to fixed assets. The fixed assets are long term, in nature.
Basically, fixed assets create earnings to the firm. They give benefit in future. It is difficult
to measure the benefits as future is uncertain. The investment decision is important not only

Chandan Kumar Sah Page 39 of 152 MBA 2ND Semester


for setting up new units but also for expansion of existing units. Decisions related to them
are, generally, irreversible. Often, reversal of decisions results in substantial loss. When a
brand new car is sold, even after a day of its purchase, still, buyer treats the vehicle as a
second-hand car. The transaction, invariably, results in heavy loss for a short period of
owning. So, the finance manager has to evaluate profitability of every investment proposal,
carefully, before funds are committed to them.
Short-term Investment Decisions: The short-term investment decisions are, generally,
referred as working capital management. The finance manger has to allocate among cash
and cash equivalents, receivables and inventories. Though these current assets do not,
directly, contribute to the earnings, their existence is necessary for proper, efficient and
optimum utilization of fixed assets.
(B) Finance Decision: Once investment decision is made, the next step is how to raise
finance for the concerned investment. Finance decision is concerned with the mix or
composition of the sources of raising the funds required by the firm. In other words, it is
related to the pattern of financing. In finance decision, the finance manager is required to
determine the proportion of equity and debt, which is known as capital structure. There are
two main sources of funds, shareholders’ funds (variable in the form of dividend) and
borrowed funds (fixed interest bearing).These sources have their own peculiar
characteristics. The key distinction lies in the fixed commitment. Borrowed funds are to be
paid interest, irrespective of the profitability of the firm. Interest has to be paid, even if the
firm incurs loss and this permanent obligation is not there with the funds raised from the
shareholders. The borrowed funds are relatively cheaper compared to shareholders’ funds,
however they carry risk. This risk is known as financial risk i.e. Risk of insolvency due to
non-payment of interest or non-repayment of borrowed capital.
On the other hand, the shareholders’ funds are permanent source to the firm. The
shareholders’ funds could be from equity shareholders or preference shareholders. Equity
share capital is not repayable and does not have fixed commitment in the form of dividend.
However, preference share capital has a fixed commitment, in the form of dividend and is
redeemable, if they are redeemable preference shares. Barring a few exceptions, every firm
tries to employ both borrowed funds and shareholders’ funds to finance its activities. The
employment of these funds, in combination, is known as financial leverage. Financial
leverage provides profitability, but carries risk. Without risk, there is no return. This is the

Chandan Kumar Sah Page 40 of 152 MBA 2ND Semester


case in every walk of life! When the return on capital employed (equity and borrowed
funds) is greater than the rate of interest paid on the debt, shareholders’ return get
magnified or increased. In period of inflation, this would be advantageous while it is a
disadvantage or curse in times of recession.
Example:
Total investment: Rs. 1,00,000
Return 15%.
Composition of investment:
Equity Rs. 60,000
Debt @ 7% interest Rs. 40,000
Return on investment
@ 15% Rs. 15,000
Interest on Debt Rs. 2,800
7% on Rs.40,000
Earnings available to
Equity shareholders Rs. 12,200
Return on equity (ignoring tax) is 20%, which is at the expense of debt as they get 7%
interest only.
In the normal course, equity would get a return of 15%. But they are enjoying 20% due to
financing by a combination of debt and equity. This area would be discussed in detail while
dealing with Leverages, in the later chapter. The finance manager follows that combination
of raising funds which is optimal mix of debt and equity. The optimal mix minimizes the
risk and maximizes the wealth of shareholders.
(C) Dividend Decision: Dividend decision is concerned with the amount of profits to be
distributed and retained in the firm.
Dividend: The term ‘dividend’ relates to the portion of profit, which is distributed to
shareholders of the company. It is a reward or compensation to them for their investment
made in the firm. The dividend can be declared from the current profits or accumulated
profits. Which course should be followed – dividend or retention? Normally, companies
distribute certain amount in the form of dividend, in a stable manner, to meet the
expectations of shareholders and balance is retained within the organization for expansion.
If dividend is not distributed, there would be great dissatisfaction to the shareholders. Non-

Chandan Kumar Sah Page 41 of 152 MBA 2ND Semester


declaration of dividend affects the market price of equity shares, severely. One significant
element in the dividend decision is, therefore, the dividend payout ratio i.e. what proportion
of dividend is to be paid to the shareholders. The dividend decision depends on the
preference of the equity shareholders and investment opportunities, available within the
firm. A higher rate of dividend, beyond the market expectations, increases the market price
of shares. However, it leaves a small amount in the form of retained earnings for
expansion. The business that reinvests less will tend to grow slower. The other alternative
is to raise funds in the market for expansion. It is not a desirable decision to retain all the
profits for expansion, without distributing any amount in the form of dividend.
There is no ready-made answer, how much is to be distributed and what portion is to be
retained. Retention of profit is related to
• Reinvestment opportunities available to the firm.
• Alternative rate of return available to equity shareholders, if they invest themselves.

Q2. What is the future value of an annuity and state the formulae for future value of an
annuity.
Ans: Annuity refers to the periodic flows of equal amounts. These flows can be either termed as
receipts or payments. The future value of a regular annuity for a period of n years at i rate
of interest can be summed up as FVAn = A {(1+i) n – 1}/ i
Where, FVAn = Accumulation at the end of n years
i = Rate of interest
n = Time horizon or no. of years
A = Amount invested at the end of every year for n years
The expression (1+i) n – 1)/i is called the Future Value Interest Factor for Annuity.

Q3. The equity stock of ABC Ltd. is currently selling for Rs. 30 per share. The dividend
expected next year is Rs. 2.00, the investors required rate of return on this stock is 15
per cent. If the constant growth model applies to ABC Ltd., What is the expected
growth rate?
Ans: If the constant growth model applies to ABC Ltd. Then
Po = D1/(Ke – g)
Where, Po = Current market price of the share

Chandan Kumar Sah Page 42 of 152 MBA 2ND Semester


D1 = Expected dividend after one year
Ke = Required rate of return on the equity share
g = Growth rate
Here in the case of ABC Ltd.
Po = Rs. 30
D1 = Rs. 2
Ke = 15%
g = ?
therefore using the formulae Po = D1/(Ke – g)
Ke – g = D1/Po
.15 – g = 2 / 30
g = .15 – 2/30
= (4.5 – 2 ) / 30
= 2.5 / 30
= .083333
= 8.333 %

Therefore, the expected growth rate is 8.33 %.

Q4. State the assumptions underlying the CAPM model and MM model.
Ans: CAPM model establishes a relationship between the required rate of return of a security
and its systematic risks expressed as “β”. According to this model,
Ke = Rf + β (Rm - Rf )
Where Ke is the rate of return of share,
Rf is the risk free rate of return,
β is the beta of security,
Rm is return on market portfolio
The assumptions underlying CAPM model are:
• Investors are risk-averse.
• Investors make their investment decisions on a single-period horizon.

Chandan Kumar Sah Page 43 of 152 MBA 2ND Semester


• Transaction costs are low and therefore can be ignored. This translates to assets
being bought and sold in any quantity desired. The only considerations that matter are
the price and amount of money at the investor’s disposal.
• All investors agree on the nature of return and risk associated with each
investment.

MM model criticize that the cost of equity remains unaffected by leverage up to a


reasonable limit and K0 remains constant at all degrees of leverage. They state that the
relationship between leverage and cost of capital is elucidated as in net operating income
approach. The assumptions underlying the MM model are:
• Perfect capital markets: Securities can be freely traded, that is, investors are free
to buy and sell securities, there are no hindrances on the borrowings, no presence of
transaction costs, securities are infinitely divisible, availability of all required
information at all times.
• Investors behave rationally: They choose the combination of risk and return
which is most advantageous to them.
• Homogeneity of investors’ risk perception: All investors have the same perception
of business risk and returns.
• Taxes: There is no corporate or personal income tax.
• Dividend pay-out is 100%: The firms do not retain earnings for future activities.

Q5. Write the cash flow analysis?


Ans: Cash Flow Analysis:
(1) Estimate your annual gross income as the first step in preparing a cash flow analysis.
Allow for subtractions if your business is not operating at full potential. For instance,
if you own an apartment complex, you add the amount of rent for each month, but
subtract an estimated amount for unforeseen vacancies. The longer you are in
business, the easier it will be to predict operating losses.
(2) Add any other income you receive and you will arrive at your "effective gross
income." This is a reliable business accounting figure that represents your entire
annual projected gross income. Write this number down for future figuring.

Chandan Kumar Sah Page 44 of 152 MBA 2ND Semester


(3) Compile a list of the expenses you incur in order to operate your business. Separate
this by category. Think about the purchases of big equipment you make. If you have a
painting business, you would write down the expenses you pay annually for paint
sprayers, rollers, brushes and drop cloths.
(4) Write down all your office expenses, utility charges, advertising expenses and other
fees you pay for equipment repairs and maintenance. Everything you need to
purchase in the operation of your business counts.
(5) Remember to include professional fees and taxes in your cash flow analysis. If you
have an accountant, his fee goes here--so does insurance policy expense, worker's
compensation payments, unemployment insurance fees and taxes charged on your
equipment or building.
(6) Add your business accounting expense together and double check your chart of
accounts to make sure you got them all. This number represents the total amount of
expenses necessary to operate your business. Write it down beneath the effective
income figure.
(7) Figure out your debt service. Calculate the amount of payments you will make to the
bank for loans, mortgages or other financing. Add these together and write the
number down beneath the total expenses figure.
(8) Subtract the total expenses and the total debt service figures from the effective annual
income number. This is your cash flow analysis for the year.

Q6. The following two projects A and B requires an investment of Rs. 2,00,000 each. The
income returns after tax for these projects are as follows:
Year Project A Project B
1 Rs. 80,000 Rs. 20,000
2 Rs. 80,000 Rs. 40,000
3 Rs. 40,000 Rs. 40,000
4 Rs. 20,000 Rs. 40,000
5 Rs. 60,000
6 Rs. 60,000
Using the following criteria determine which of the projects is preferable.
Ans:

Chandan Kumar Sah Page 45 of 152 MBA 2ND Semester


Project A

year Income PVIF@10% PVCI


1 80000 0.909 72720
2 80000 0.826 66080
3 40000 0.751 30040
4 20000 0.683 13660
PVCI 182500

PVCI - NPV
182500 200000 = -17500

Project B

year Income PVIF@10% PVCI


1 20000 0.909 18180
2 40000 0.826 33040
3 40000 0.751 30040
4 40000 0.683 27320
5 60000 0.621 37260
6 60000 0.564 33840
PVCI 179680

PVCI - NPV
179680 200000 = -20320

As Project A is preferable option as it has minimal losses.

Chandan Kumar Sah Page 46 of 152 MBA 2ND Semester


Chandan Kumar Sah Page 47 of 152 MBA 2ND Semester
Assignment Set – 1

Q1. a. Explain the different micro-environmental forces with examples.


b. Mention the different ad appeals with suitable examples.
Ans: a) The different micro-environment forces are:
i) Company: Safe Express, a leader in the supply chain management solution wants to
hold its number one position in the US $ 90 billion Indian logistics market. The
company plans to expand its service areas in the coming months. To meet the targets
of the marketing plan, other departments of safe express also expanding their horizon.
The company is coming out with logistics parks in different cities; plans to hold seven
million square feet of warehousing capacity in the next three years and invest Rs. 10
billion in three years to meet those targets. The above example shows that the
company’s marketing plan should be supported by the other functional departments
also.
ii) Intermediaries: Intermediaries are firms which distribute and sell the goods of the
company to the consumer. Marketing intermediaries play an important role in the
distribution, selling and promoting the goods and services to customer are some of the
major functions carried out by the middlemen. Retailers, wholesalers, agents, brokers,
jobbers and carry forward agents are few of the intermediaries.
iii) Publics: Publics are microenvironment groups, which help a company to generate the
financial resources, creating the image, examining the companies’ policy and
developing the attitude towards the product. We can identify six types of publics a)
Financial publics, b) Media publics, c) Regulation Agency, d) Citizen action groups,
e) General publics, f) Internal publics.
iv) Competitors: The Company should monitor its immediate competitors as its sale will
be affected by the nature and intensity of the competitors. The sale of Coca cola will
be affected by Pepsi cols, or Britannia cheese by Amul cheese. Michael Porter, the
author of Competitive Advantage of Nations suggested that, in addition to direct
competition, companies should also consider competition from substitutes. In
addition to existing competitors, the potential competitors should also be anticipated.
Competition may arise from small firms with low overheads producing duplicates,
Firms which diversify into certain products by merely being in the particular industry

Chandan Kumar Sah Page 48 of 152 MBA 2ND Semester


for e.g. pepsi entered the snacks sector competing with pure snacks producer like
Haldiram, Firms which expand in the same vertical for e.g. Godrej which
manufactured office furniture and steel cupboards went on to the entire range or home
furniture thereby giving competition to pure home furniture makers.
v) Customers: A company may sell their products directly to the customer or use
marketing intermediaries to reach them. Direct or indirect marketing depends on what
type of markets company serves. Generally we can divide the market into Consumer
market, Business market, Reseller market, Government market and International
market. MRF, a tyre company sells its product directly to consumer i.e. operates in
consumer market. It operates in business markets by selling tyres to companies like
Maruti Udyog Limited. MRF also sells tyres to BMTC and KSRTC, transport
organizations of Karnataka government. If MRF buys the old tyres, retreads it and
sells it to the consumer at a profit them company is operating in the reseller market.
vi) Suppliers: Suppliers are the first link in the entire supply chain of the company.
Hence any problems or cost escalation in this stage will have direct effect on the
company. Many companies adopted supplier relation management system to manage
them well; Suppliers are a source of competition to firms today. For a large retail
store like Reliance Retail or Big Bazar the suppliers play the most significant role in
both cost and time. Timely supplies reduce stocking of goods and blocking of space,
at the same time meet customer requirements.

b) The different ad appeals are


Audio-Visual Media Advertising: In this advertising technique, the advertisers use the very
popular audio and visual media to promote a product. It is the most widely used media that
can effectively influence the masses. Television and radio have always been used to
achieve a mass appeal.
1) Bandwagon: This advertising appeal aims to persuade people to do a certain thing
because the masses are doing it. It is a human tendency to do as the masses do. Bandwagon
technique of advertising captures exactly this psychology of human beings to induce them
to use a certain product or service.
2) Black and White Fallacy: In this advertising appeal, only two choices are presented
before the audiences, thus compelling them to buy the product being advertised.

Chandan Kumar Sah Page 49 of 152 MBA 2ND Semester


3) Card Stacking: This advertising appeal involves the display of a comparative study
between two competing products. The facts in favor of the product to be advertised are
selected. They are put forth in comparison with those of a competing product to make the
product appear better than its competitors.
4) Classified Advertising: This type of advertising makes use of newspapers and
periodicals to make public appeals about the products or services to be advertised. Know
about goint global with classified advertising.

Corporate Advertising: Corporate advertising is an advertising appeal wherein corporate


logos and company message are publicized on a large scale. Hot air balloons are commonly
used in advertising a product. Some companies propose to place their logos on booster
rockets and space stations to achieve a wide publicity of the company.
1) Covert Advertising: This is the practice of achieving an indirect publicity of the
product by advertising it through movies and TV shows. TV actors and characters in
movies are often shown using certain products. Covert advertising is an indirect way of
advertising a product by featuring it in films and television shows.
2) Demonizing the Enemy: In this advertising appeal, the advertisers make the people
with an opposing point of view appear unacceptable. The people bearing certain ideas are
made to appear among the disliked individuals of society. This form of advertising involves
the idea of encouraging an idea by discouraging the ideas contrary to it. It is like proving a
theorem by disproving its inverse.
3) Direct Order: When the advertisers appeal the masses by showing them the steps to
take, in order to opt for a particular product or service, they are said to be using direct
order. This advertising appeal often communicates the steps to choose a certain product or
service and presents them to the audiences in a simplistic manner.
4) Disinformation: This technique involves a purposeful dissemination of false
information. In the context of military, this technique is used to mislead the enemy. It
commonly includes forging of documents and the spread of rumors.

Email Advertising: This is a relatively new advertising appeal that makes use of emails to
advertise products. Advertisements are sent through emails, thus bringing out
communication with a wide range of audiences.

Chandan Kumar Sah Page 50 of 152 MBA 2ND Semester


Emotional Words: This advertising appeal makes use of positive words to generate
positive feelings in the minds of the people about a certain product. The advertisers often
use the words like 'luxury', 'comfort' and 'satisfaction' to create positive vibes among the
masses to attract them towards the product being advertised.
1) Euphoria: The use of positive events characterizes this advertising appeal. Declaration
of a great discount or sale on a holiday and making luxury items available at affordable
prices are often used to aim mass appeal.
2) Flag-waving: The advertising appeal that makes use of the patriotic flavor to publicize a
product is known as flag-waving. In case of flag-waving, the advertisers try to justify
certain actions on the grounds of patriotism. The masses are persuaded to choose a
particular product because doing so will be an exhibit of patriotism.
3) Glittering Generalities: In this form of advertising, advertisers make use of logical
fallacies. They use appealing words without giving any concrete idea about what is being
advertised.
4) Half Truth: In this advertising appeal, the advertisers use deceptive statements to
publicize their product. They often use double-meaning words or statements to convey
their message to the masses.
5) Interactive Advertising: This advertising appeal makes use of the interactive media to
reach out to the target audiences. Advertisers often hold exhibitions or trade-shows and
offer rewards in the forms of discounts and free gifts to increase the sale of their product.
Distribution of free samples of a newly launched product and publicity of a product
through interactive means are some examples of interactive advertising. Have you
participated in contests intended to promote products? Have you come across websites
offering free products to users? The free stuffs are popularly known as freebies. Know
more about Freebies.

Internet Advertising: It is a relatively recent form of advertising appeal. Internet has


become one of the most influential media of the modern times. Websites attract thousands
of users everyday and expose them to the advertisements on the websites.

Chandan Kumar Sah Page 51 of 152 MBA 2ND Semester


1) Labeling: With an intent to increase or diminish the perceived quality of a product, the
advertisers use labeling or categorization. Categorizing a product under a group of
associated entities makes it appealing or unappealing to the masses.
2) Name-calling: This advertising appeal makes use of direct or indirect attack on the
products in competition with the product being advertised. Direct name-calling involves
making a direct attack on the opponent while indirect name-calling makes use of sarcasm
to demean the products of the competitors.

Outdoor Advertising: Outdoor advertising is a popular advertising appeal that uses


different tools to attract the customers outdoors. Billboards, kiosks and tradeshows are
some of the commonly used means of outdoor advertising. Fairs, exhibitions and billboards
that draw in the passersby are often used in outdoor advertising.
1) Performance-based Advertising: In performance-based advertising appeal, the
advertisers pay only for the results. The advertising agency assumes the entire risk and
hence ensures that the advertisement is pitched well.
2) Plain Folks: This advertising appeal aims at attracting the masses by using common
people to advertise a product. Bombastic words may not always appeal the common folks.
They can rather be attracted by communicating with them in their language. The use of
homey words, as they are called, and purposeful errors while speaking to give a natural feel
to the speech, is characteristic to this advertising appeal.
3) Print Media Advertising: The print media is one of the most effective means of
advertising. Many advertising appeals make use of the print media to reach out to the
masses. Media like newspapers, brochures, manuals and magazines are used for publicizing
the products.
4) Public Service Advertising: This advertising technique is used to convey socially
relevant messages to the masses. Social messages on issues like poverty, inequality, AIDS
awareness and environmental issues such as global warming, pollution and deforestation
are conveyed by the means of appeals to the public through public service advertising.
5) Quotes Out of Context: This advertising appeal makes use of popular quotes. The
advertisers using this advertising appeal alter the widely known quotes to change their
meaning. This technique is used in political documentaries.

Chandan Kumar Sah Page 52 of 152 MBA 2ND Semester


Relationship Marketing: This form of advertising focuses on the retention of customers
and customer satisfaction. Advertisers appeal to the target audiences with information that
suits their requirements and interests.
1) Repetition: This advertising appeal uses the technique of repeating the product name
several times during an advertisement. Jingles are often used in this advertising technique
to linger the product name in the minds of the masses.
2) Scientific Evidence: This technique attempts to appeal the masses to use the advertised
product, by providing the audiences with survey results. The advertisers often use statistical
evidences and market surveys to publicize their product.
3) Shockvertising: This advertising appeal makes use of shocking images or scenes to
advertise a product. The name is derived as a combination of the two words, ‘shocking’
and ‘advertising’.
4) Slogans: Slogans, as we all know, are striking phrases used to convey important
information of the product to be advertised in an interesting manner. There are lot many
famous advertising slogans that we are familiar with.
5) Snob Appeal: Snob appeal is an exact reverse of the bandwagon technique. In this
advertising appeal, people are induced to buy a certain product so that they can stand out of
the crowd. It is often indicated that buying the product will make them look different from
the rest. It is often indicated that the product is not affordable for the common masses by
attaching a 'sense of exclusivity' to such products.
6) Stereotyping: This advertising appeal is also known as name-calling or labeling and
attempts to categorize the advertised object under the class of the entities, which the masses
fear.

Subliminal Advertising: This advertising appeal makes use of subliminal messages, which
are intended to be subconsciously perceived. Subliminal signals go undetected by the
human eye. However, they are perceived at a subconscious level. Subliminal appeal often
makes use of hidden messages and optical illusions.
1) Surrogate Advertising: In cases where advertising of a particular product is made
illegal, the product companies come up with other products with the same brand name.
Advertising the legal products with the same brand name reminds the audiences of their
legally banned products as well.

Chandan Kumar Sah Page 53 of 152 MBA 2ND Semester


2) Testimonial: People tend to relate to their favorite figures in the glamor industry. People
attracted to their idols often tend to adopt what their idols do. They want to emulate the
people they relate to. Testimonial makes use of this human tendency by using the words of
an expert to recommend their products. Celebrities are used as ambassadors for products in
order to promote the sale.
3) Transfer: This advertising appeal is implemented in two ways. In a positive transfer, a
product is made to associate with a respected individual of society. However, in case of a
negative transfer, the advertisers state an analogy between a product and a disliked figure
in society.
4) Unstated Assumption: When the idea behind the product or service being advertised is
repeatedly implied, it is known as an unstated assumption. In case of using this advertising
appeal, the advertisers do not state the concepts explicitly. They rather imply their ideas in
various ways.

Viral Advertising: It can take the form of word-of-mouth publicity or of Internet


advertising. The aim of the advertiser is to market the product on a very large scale. This
advertising appeal intends to achieve a speedy publicity of a product similar to the
spreading of a pathological or a computer virus!
1) Word-of-Mouth Advertising: It can turn out being a very effective advertising appeal.
It can achieve phenomenal success to an extent where a brand is equated to a common
noun. Vaseline can be sited as one of the excellent examples of word-of-mouth publicity,
where the company name, ‘Vaseline’ became synonymous with the product name,
‘petroleum jelly’.

This was an overview of the different advertising appeals that advertisers around the world
use to market their products and services. Many of the popularly used products and
services of today have gained popularity thanks to the advertising techniques that were
implemented for their publicity. Advertising appeals used by a company are important
determinants of its success.

Q2. What are the different market entry strategies if a company wants to enter
international markets?

Chandan Kumar Sah Page 54 of 152 MBA 2ND Semester


Ans: International Market Entry Strategies: Organizations that plan to go for international
marketing should know the answers for some basic questions like –
a. In how many countries would the company like to operate?
b. What are the types of countries it plans to enter?
That’s why companies evaluate each country against the market size, market growth, and
cost of doing business, competitive advantage and risk level. Once the market is found to
be attractive, companies should decide how to enter this market. Companies can enter the
international market by adopting any one of the following strategies. They are:-
a. Exporting
b. Licensing
c. Contract manufacturing
d. Management contract
e. Joint ownership
f. Direct investment
Exporting is the technique of selling the goods produced in the domestic country in a
foreign country with some modifications. For example, Gokaldas textiles export the cloth
to different countries from India. Exporting may be indirect or direct. In case of indirect
exporting, company works with independent international marketing intermediaries. This is
cost effective and less risky too. Direct exporting is the technique in which organization
exports the goods on its own by taking all the risks. Maruti Udyog Limited, India’s leading
car manufacturer exports its cars on its own. Company can also set up overseas branches to
sell their products. Adani Exports, another leading exporter from India has international
office in Singapore.
Licensing: According to Philip Kotler, licensing is a method of entering a foreign market
in which the company enters into an agreement with a license in the foreign market,
offering the right to use a manufacturing process, trademark, patent, or other item of value
for a fee or royalty. For example, Torrent Pharmaceuticals has license to sell the
cardiovascular drugs of Chinese manufacturer Tasly. Licensing may cause some problems
to the parent company. Licensee may violate the agreement and can use the technology of
the parent company.
Contract manufacturing: Company enters the international market with a tie up between
manufacturer to produce the product or the service. For example, Gigabyte Technology has

Chandan Kumar Sah Page 55 of 152 MBA 2ND Semester


contract manufacturing agreement with D- link India to produce and sell their mother
boards. Another significant manufacturer is TVS Electronics; it produces key boards in its
own name as well as for other companies too.
Management contracting: In this case, a company enters the international market by
providing the know how of the product to the domestic manufacturer. The capital,
marketing and other activities are carried out by the local manufacturer, hence it is less
risky too.
Joint ownership: A form of joint venture in which an international company invests
equally with a domestic manufacturer. Therefore it also has equal right in the controlling
operations. For example, Barbara, a lingerie manufacturer has joint venture with Gokaldas
Images in India.
Direct Investment: In this method of international market entry, Company invests in
manufacturing or assembling. The company may enjoy the low cost advantages of that
country. Many manufacturing firms invested directly in the Chinese market to get its low
cost advantage. Some governments provide incentives and tax benefits to the company
which manufactures the product in their country. There is government restriction in some
countries to opt only for direct investment, as it produces the jobs to the local people. This
mode also depends on the country attractiveness. It may become risky if the market
matures or unstable government exists.

Q3. a. State the meaning of Product life cycle and explain the different stages involved in
it.
b. Define Customer Relationship Management.

Ans: a) Product life cycle (PLC): PLC means a product has to go though the various stages
since its inception and till it completely fades out from the market. Like human beings,
products also have their own life-cycle. From birth to death human beings pass
through various stages e.g. birth, growth, maturity, decline and death. A similar life-
cycle is seen in the case of products. The product life cycle goes through multiple
phases, involves many professional disciplines, and requires many skills, tools and
processes. Product life cycle (PLC) has to do with the life of a product in the market

Chandan Kumar Sah Page 56 of 152 MBA 2ND Semester


with respect to business/commercial costs and sales measures. To say that a product
has a life cycle is to assert four things:

• that products have a limited life,


• product sales pass through distinct stages, each posing different challenges,
opportunities, and problems to the seller,
• profits rise and fall at different stages of product life cycle, and
• products require different marketing, financial, manufacturing, purchasing, and
human resource strategies in each life cycle stage.

There are five stages in a product's life cycle:

1. Product development stage: In this stage company identifies the viable idea
and develops it. Even if sales in this stage are nil it requires huge research and
development budget. Therefore company incurs losses at this stage. For
Example, TATA Docomo before entering the cellular services market had done
research and found that calls were charged for minutes rather than seconds.
2. Introduction stage: Company introduces the product into the market. As
the product is new to the market, consumer awareness is usually very low.
Here company adopts heavy sales promotion and product awareness programs.
The cost of product is very high and sales are very low. At this juncture the
company high price to the customers. For example, TATA Docomo has
entered into cellular services initially through the Billboards.
3. Growth Stage: Company gets experience over the period and now tries to
get the maximum market share. Sales reduces the price of the product and
offers varieties and values in it. It focuses on building better distribution
network and pushes the product through it. Therefore company needs less sales
promotion. There will be increase in Competition and the company is forced to
keep a tab on this competitors. For example, TATA Docomo has entered into
the growth stage by aggressively advertising on Television and other mediums
and at the same time giving competition to the existing players.

Chandan Kumar Sah Page 57 of 152 MBA 2ND Semester


4. Maturity Stage: In this stage, the product has already established itself in
the market. These are the characteristics of this stage - a) Peak Sales, b) Low
cost per customer, c) High profits, d) Competition based pricing, e)
Communicating the product differentiation to consumers, f) Improving supply
chain efficiency, g) Defend the market share, h) Industry experience
consolidation. For example, Airtel in its advertising is clearly stating its
subscriber base as 1,00,000 indicating that it has entered into a mature stage.
5. Decline stage: In this stage, product sales and profit decline. Company
should phase out weak items from their product mix and may even lower the
prices of the existing products. The advertisement budget of the company also
comes down and the company may struggle to meet its costs. For example,
VCR’s have been replaced with DVD players and so VCR entered into the
decline stage and is almost out of the market.

b) Customer relationship management: It is a broadly recognized, widely-


implemented strategy for managing and nurturing a company’s interactions with
clients and sales prospects. It involves using technology to organize, automate, and
synchronize business processes—principally sales activities, but also those for
marketing, customer service, and technical support. The overall goals are to find,
attract, and win new clients, nurture and retain those the company already has,
entice former clients back into the fold, and reduce the costs of marketing and client
service. Once simply a label for a category of software tools, today, it generally
denotes a company-wide business strategy embracing all client-facing departments
and even beyond. When an implementation is effective, people, processes, and
technology work in synergy to increase profitability, and reduce operational costs.

Knowledge Potential
Business Objectives Type of Benefit
requirements technology
Direct Marketing Acquiring new clients using Client profiles e-mail distribution or
InterNet technologies. mailing services.

Chandan Kumar Sah Page 58 of 152 MBA 2ND Semester


Reduced cost of marketing.
Matching software
of client profile to
Cross selling by using IT Client profiles.
client profile.
to identify products Increasing business with Strategies for
Data mining.
similar to those purchased existing clients. providing new
Building client
earlier services.
history.
Mailing services.
Quicker ways to
Increasing business with Personalized
Service customization match services to
existing clients. interfaces.
client needs.
Contact center.
Reduced cost of responding Building enquiry
Web based call
Improved response to to clients. profiles.
center
client requests. Client retention through Standard responses
Personalized client
better service. to queris.
workspaces
Current status of
Reduced cost of travel client service.
Personalized client
Improving field service Client retention through Potential new
interfaces.
better service. directions given the
client profile.
Increase relationship with
client by suggesting services Identifying unusual
Event identification Messahge analysus.
to meet new client events.
circumstances.

Q4. a. You are a sales manager in ABC firm. You have taken some interviews and
shortlisted a few candidates. How will you select the right candidate for the sales
job?
b. As a consumer, what are the steps you will undertake before you decide to buy a
car?

Chandan Kumar Sah Page 59 of 152 MBA 2ND Semester


Ans: a) Being as sales manager in ABC firm the following steps I will take to select the right
candidate for the sales job:
1. Necessary Information about the applicant is required to be considered for
appointment such as name, sex, qualification, age, experience, health, social
activities, address, references etc.
2. Screening is a process by which applications are to be screened out from further
consideration, on the basis of unsuitability. The remaining applications are formally
considered for appointment, subject to further formalities.
3. The qualities are checked with care and caution by the referees. If the opinions are
favorable, the applications pass on to the next stage; and in case the referee gives
unfavorable comment, the application is rejected at this stage. Personal contact is
necessary and it is better, because people are straight forward in tongue better than in
pen.
4. Through the personal interview, I can understand the positive and negative qualities
of the applicant, with reference to the job duties.
5. Test is an additional tool, with which the applicants are further tested to determine
their suitability for the job.
6. The important thing about any person, apart from all qualities and eligibility, is that
he/she must be physically fit for the job.
7. The selected applicant is probably, called for a final interview and his/her suitability
is measured through different tests, physical reports etc. Once confirmed,
appointment letter will be given to the person and the job will be explained to
him/her, along with all relevant details, which are required to perform the duties
efficiently.

Chandan Kumar Sah Page 60 of 152 MBA 2ND Semester


b) The steps undertaken before deciding to buy a car-
1. Need recognition: customer posses two type of stimuli’ at this juncture. One is driven by
the internal stimuli and another is external stimuli. The examples of internal stimuli are
customer’s desire, attitude or perception and external stimuli are advertising etc. From both
stimuli customers understand the need for the product. Here marketer should understand
what customers needs have that drew customers towards the product and should highlight
those in the communication strategy.
2. Information search: In this stage customer wants to find out the information about the
product, place, price and point of purchase. Customer collects the information from
different sources like
a. Personal sources: Family, friends and neighbors
b. Commercial sources: Advertising, sales people, dealers, packaging and displays.
c. Public sources: mass media and consumer rating agencies.
d. Experiential sources: Demonstration, examining the product.
In this stage marketer should give detailed information about the product. The
communication should highlight the attributes and advantages of the product in this stage
so that he created the positive image about the product.
3. Evaluation of alternatives: After collecting the information, consumers arrive at some
conclusion about the product. In this stage he will compare different brands on set
parameters which he or she thinks required in the product. The evaluation process varies
from person to person. In general Indian consumer evaluate on the following parameters
a. Price
b. Features
c. Availability
d. Quality
e. Durability
At this stage marketer should provide comparative advertisements to evaluate the different
brands. The advertisement should be different for different segments and highlight the
attribute according to the segment.
4. Purchase decision: In this stage consumer buy the most preferred brand. In India
affordability plays an important role at this stage. Organizations’ bring many varieties of
the products to cater to the needs of customers.

Chandan Kumar Sah Page 61 of 152 MBA 2ND Semester


5. Post purchase behavior: After purchasing the product the consumer will experience
some level of satisfaction and dissatisfaction. The consumer will also engage in post
purchase actions and product uses of interest to the marketer. The marketer’s job does not
end when the product is bought but continues into the post purchase period. Customer
would like to see the performance of the product as he perceived before purchase. If the
performance of the product is not as he expected then he develops dissatisfactions.
Marketer should keep an eye on how consumer uses and disposes the product. In some
durable goods Indian consumer want resale value also. Many automobile brands that were
not able to get resale value lost their market positions.

Q5. a. What are the features of Business markets? How is it different from consumer
markets?
b. List out the 5 important requisites of an effective segmentation by giving suitable
examples.

Ans: a) Business Markets: The business market includes organizational buying, institutional
buying and government buying. The buying may involve materials as final consumption
and materials for intermediate consumption. The features of Business Markets are : i) Few
but bulk Buyers, ii) Geographical concentration of buyers, iii) Variable demand, iv)
Inelastic demand, v) Systematic purchasing, vi) Multiple buying influences, vii)
Reciprocation, viii) Lease agreements. Marketing in businesses is different to marketing to
consumers, although many of the techniques, such as branding, can be transferred
successfully. In particular, business-to-business markets (B2B) are more focused on
customer relationships than is normally true for consumer markets.

The difference between Business markets and consumer markets is as under:

Consumer Market Business Market


i) Every customer has equal value and i) There are a small number of big
represents a small % of revenue customers that account for a large % of
revenue
ii) Sales are made remotely, the ii) Sales are made personally, the
manufacturer doesn't meet the customer manufacturer gets to know the customer
iii) Products are the same for all iii) Products are customized for different

Chandan Kumar Sah Page 62 of 152 MBA 2ND Semester


customers. The service element is low customers. Service is highly valued
iv) Purchases are made for personal use - iv) Purchases are made for others to use -
image is important for its own sake image is important where it adds value to
customers
v) The purchaser is normally the user v) The purchaser is normally an integrator;
someone down the supply chain is the user.
vi) Costs are restricted to purchase costs vi) Purchase costs may be a small part of
the total costs of use
vii) The purchase event is not subject to vii) The purchase event is conducted
tender and negotiation professionally and includes tender and
negotiation.
viii) The exchange is one of transaction. viii) The exchange is often one of strategic
There is no long-time view (financial intent. There is the potential for long term
services differ) value

b) The five important requisites of an effective segmentation are:


1. Measurable and Obtainable: The size, profile and other relevant characteristics of
the segment must be measurable and obtainable in terms of data. If the information is
not obtainable, no segmentation can be carried out. For example, Census of India
provides the data on migration and education level, but does not specify how may of
the migrated employees are educated and if educated how many are in white collared
jobs.
2. Substantial: The segment should be large enough to be profitable. For consumer
markets, the small segment might disproportionably increase the cost and hence
products are priced too high. For example, when the cellular services started in India
cost of the incoming calls and outgoing calls were charged at Rs. 12/minute. As the
number of subscribers grew, incoming calls became free. Further growth of
subscribers resulted in lowering tariffs for outgoing calls to the lowest level in the
world.
3. Accessible: The segment should be assessable through existing network of people at
an affordable cost. For example, Majority of the rural population is still not able to
access the internet due to the high cost and non-availability of connections and
bandwith.

Chandan Kumar Sah Page 63 of 152 MBA 2ND Semester


4. Differentiable: The segments should be different from each other and may require
different 4Ps and programs. For example, Life Insurance Corporation of India needs
separate marketing programs to sell their insurance plans, unit plans, pension plans
and group schemes.
5. Actionable: The segments which a company wishes to pursue must be actionable in
the sense that there should be sufficient finance, personnel, and capability to take
them all.

Q6. Explain briefly what are the several processes involved in new product development.
Ans: New products are essential for existing firms to keep the momentum and for new firms they
provide the differentiation. New product doesn’t mean that it is absolutely new to the
world. The several processes involved in new product development are:
Stage 1 – Idea generation: New product idea can be generated either from the internal
sources or external sources. The internal sources include employees of the organization and
data collected from the market. The external source includes customers, competitors and
supply chain members.
State 2 – Idea screening: Organization may have various ideas but it should find out which
of these ideas can be translated into concepts. In an interview to Times of India, Mr. Ratan
Tata, chairman TATA group discussed how his idea saw many changes from the basic
version. He told that he wanted to develop car with scooter engine, plastic doors etc. But
when he unveiled the car, there were many changes in the product. This shows that initial
idea will be changed on the basis of market requirements.
State 3 – Concept development: the main feature or the specific desire that it caters to or
the basic appeal of the product is created or designed in the concept development. Concepts
used for Tata Nano care are :- Concept I: Low-end ‘rural car,’ probably without doors or
windows and with plastic curtains that rolled down, a four-wheel version of the auto-
rckshaw, Concept II: A car made by engineering plastics and new materials, and new
technology like aerospace adhesives instead of welding, Concept III: Indigenous, in-house
car, which meets all the environment standards.
Stage 4 - Concept testing: At this stage concept is tested with the group of target
customers. If any changes are required in the concept or the message it will be done during

Chandan Kumar Sah Page 64 of 152 MBA 2ND Semester


this stage. Also the effectiveness is tested on a minor scale. If the concept meets the
specific requirements, then it will be accepted.
Stage 5 Marketing strategy development: The marketing strategy development involves
three parts. The first part focuses on target market, sales, market share and profit goals.
TATA’s initial business plan consisted sales of 2 lakhs cars per annum. The second part
involves product price, distribution and marketing budget strategies. TATA’s fixed Rs. 1
lakhs as the car price, and finding self employed persons who work like agent to distribute
the cars. The final part contains marketing mix strategy and profit goals.
Stage 6 Business analysis: It is the analysis of sales, costs and profits estimated for a new
product and to find out whether these align with the company mission and objectives.
Stage 7 Product development: during this stage, product is made to undergo further
improvements, new features or improvised versions are added to the product. There is also
scope for innovation and using the latest technology into the product.
Stage 8 Test marketing: is the most crucial stage for the testing product’s performance and
its future in the market. There are certain cases where product has failed in the test
marketing and had to be withdrawn. a) the product is introduced into the realistic market, b)
The 4Ps’ of marketing are tested, c) The cost of test marketing varies with the type of
product.
Stage 9 Commercialization: In this stage product is completely placed in the open market
and aggressive communication program accompanied with promotion activities is carried
out to support it.

Chandan Kumar Sah Page 65 of 152 MBA 2ND Semester


Assignment Set – 2

Q1. Highlight the importance of Distribution channel and marketing intermediaries in


carrying out the marketing function.
Ans: The delivery of goods and services from producers to their ultimate consumers or users
includes many different activities. These different activities are known as marketing
functions. Different thinkers have described these functions in different ways. Some of the
most important functions of marketing are briefly discussed below:
1. Marketing Research and Information Management: Marketers need to take decisions
scientifically. Marketing research function is concerned with gathering, analyzing and
interpreting data in a systematic and scientific manner. The types of market information
could be analysis of market size and characteristics, consumer tastes and preferences and
changes in them from time to time, channels of distribution and communication and their
effectiveness, economic, social, political and technological environment and changes
therein. A company can procure such information from specialized market research
agencies, government or can decide to collect themselves.
2. Advertising and Sales Promotion: Advertising is a mass media tool used to inform,
persuade or remind customers about products or services. It is an impersonal form of
communication targeted at a chosen group through paid space or time. Sales Promotion is a
short-term incentive given to customers or intermediaries to promote sales. It supplements
advertising and personal selling and can be used at the time of launching a new product or
even during its maturity period.
3. Product Planning and Management: A Marketer should identify the needs and wants of
consumers, develop suitable products / services and make them available. Marketer is also
required to maintain the product and its variations in size, weight, package and price range
according to the changing needs and requirements of his customers. Information available
through Market Research helps product management in taking appropriate decisions while
planning the marketing efforts.
4. Selling: This function of marketing is concerned with transferring of products to the
customer. An important part of this function is organizing sales force and managing their
activities. Sales force management includes recruitment, training, supervision,
compensation and evaluation of salesmen. They need to be assigned targets and territories

Chandan Kumar Sah Page 66 of 152 MBA 2ND Semester


where they can operate. The salesmen interact with prospective purchasers face-to-face in
order to sell the goods. The purchaser may be end customer or an intermediary, such as a
retailer or a dealer.
5. Physical Distribution: Moving and handling of products from factory to consumers
come under this function. Order processing, inventory, management, warehousing and
transportation are the key activities in the physical distribution system.
6. Pricing: This is perhaps the most important decision taken by marketer, as it is the only
revenue fetching function and success and failure of the product may depend upon this
decision. Therefore, the decision regarding how much to charge should be taken such that
the price is acceptable to the prospective buyers and at the same time fetches profits for the
company. While deciding on the price, the factors to be considered are competition,
competitive prices, company’s marketing policy, government policy, and the buying
capacity of target market etc.

Importance of marketing intermediaries:


These are firms which distribute and sell the goods of the company to the consumer.
Marketing intermediaries play an important role in the distribution, selling and promoting
the goods and services. Stocking and delivering, bulk breaking, and selling the goods and
services to customer are some of the major functions carried out by the middlemen.
Retailers, wholesalers, agents, brokers, jobbers and carry forward agents are few of the
intermediaries. Retailers are final link between the company and the customers. Their role
in the marketing of product is increasing every day.

Q2. a. Explain the different product mix pricing strategies.


b. Give a note on marketing concepts.
Ans: a) Product mix pricing strategies: The different product mix pricing strategies are:
1. Product Line pricing: Strategy of setting the price for entire product line.
Marketer differentiates the price according to the range of products, i.e.
suppose the company is having three products in low, middle and high end
segment and prices the three products say at Rs 10 Rs 20 and Rs 30
respectively. The three levels of differentiation create three price points in
the mind of consumer. The task of marketer is to establish the perceived

Chandan Kumar Sah Page 67 of 152 MBA 2ND Semester


quality among the three segments. If the customers do not find much
difference between the three brands, he/she may opt for low end products.
2. Optional Product pricing: this strategy is used to set the price of optional or
accessory products along with a main product. Maruti Suzuki will not add above
accessories to its product Swift but all these are optional. Customer has to pay
different prices as mentioned in the picture for different products. Organizations
separate these products from main product so that customer should not
perceive products are costly. Once the customer comes to the show room,
organization explains the advantages of buying these accessory products.
3. Captive product pricing: Setting a price for a product that must be used along
with a main product. For example, Gillette sells low priced razors but make
money on the replacement cartridges.
4. By-product pricing: It is determining the price for by-products in order to make
the main product s price more attractive. For example, L.T. Overseas,
manufacturers of Dawaat basmati rice, found that processing of rice results in
two by-products i.e. rice husk and rice brain oil. If the company sells husk
and brain oil to other consumers, then company is adopting by-product
pricing.
5. Product bundle pricing: It is offering companies several products together
as a bundle at the reduced price. This strategy helps companies to generate
more volume, get rid of the unused products and attract the price conscious
consumer. This also helps in locking the customer from purchasing the
competitors products. For example, Anchor toothpaste and brush are offered
together at lower prices.

b) The Marketing Concepts: The Marketing Concepts proposes that a company’s task is
to create, communicate and deliver a better value proposition through its marketing
offer, in comparison to its competitors; to its target segment and that this customer
oriented approach only can lead to success in the market place. Marketing function is
seen as one of the most important functions in the organization. Many marketers put the

Chandan Kumar Sah Page 68 of 152 MBA 2ND Semester


customers at the centre of the company and argue in favor of such a customer
orientation, where all functions work together to respond, serve and satisfy the
customer.

Q3. a. What are the features of business markets?


b. Write a short note on product line and product mix.

Ans: a) Business Markets: The business market includes organizational buying, institutional
buying and government buying. The buying may involve materials as final consumption
and materials for intermediate consumption. The features of Business Markets are :

i) Few but bulk Buyers: The no. of buyers is few but they buy in large quantity. For
example, major airlines buy the necessary equipments from the aircraft
manufacturers.

ii) Geographical concentration of buyers: Buyers are geographically concentrated.


For example, shipping industries are located on the east and west coasts of India
than in any other places.

iii) Variable demand: The nature of demand is fluctuation because the demand is
basically a derived one. Based on the requirements of the consumer markets,
organizations buy the goods and make the finished goods available in the market
for final consumption. For example, mobiles are being used by a large population
and so cellular companies have to meet this rising demand.

iv) Inelastic demand: The demand is also inelastic because organizations cannot
make rapid changes in the production structure and so prices remain constant in
the short-term. For example, Shoe manufacturers will not buy much leather if the
price of leather is less neither will they buy less leather if the price increases.

v) Systematic purchasing: The purchasing activity is directly between the buyer and
supplier organization which means there are no or every few middlemen involved.
Purchasing activity is usually undertaken by purchase departments based on a
proper structure and through various mechanism like having purchase requisitions

Chandan Kumar Sah Page 69 of 152 MBA 2ND Semester


from other sections, inviting tenders and sending invoices from the suppliers,
purchasing agreements or contracts with the key suppliers, renewing agreements
etc. For example, Reliance Fresh has regular contracts with the agricultural
producers for smooth supply of fresh fruits and vegetables.

vi) Multiple buying influences: There will be several parties involved in deciding
about the purchases because organizations will have several departments and units
functioning under it with different requirements. For example, purchase
department in a Hospital must be aware about the specific requirements in the
clinical wards, operation theaters, labs, etc.

vii) Reciprocation: This means that when an organization buys goods from another
organization then the supplier organization also might need certain other goods
that are produced by the buyer organization. For example, a stationery supplier
will supply the necessary stationeries to the paper manufacturer who in turn
provides papers to the supplier.

viii) Lease agreements: Most organizations take on lease the expensive equipments
required by them rather than buy it. So, in this way, they reduce cost, get better
service and the lessor or one who provides the equipments will also profit from
the rent or lease charges. For example, TATA provides the transport trucks to
other organizations on lease.

b) Product Line: The product line identifies profitable and unprofitable products and helps
in allocation of resources according to that. The product line understanding helps the
marketer to take line extension, line pruning and line filling strategies of the company.

Product Mix: A company’s product mix has four different dimensions. They are product
mix width, product mix length, product mix depth and product mix consistency. Product
mix width: The total number of product lines that company offers to the consumers.
Product mix length: The total number of items that company carries within its product line.
Product line depth: The number of versions offered of each product in the line. Product mix
consistency: If company’s product lines usage, production and marketing are related, then
product mix is consistent, else it is unrelated.

Chandan Kumar Sah Page 70 of 152 MBA 2ND Semester


Q4. a. Select any deodorant brand and evaluate its positioning strengths or weakness in
terms of attributes, benefits, values, brand name and brand equity.
b. You are a research expert in the field of marketing footwear products. What are
the various research approaches you would consider before making a consumer
survey regarding footwear?
Ans: a) Brand Name:
A name, term, design, symbol, or any other feature that identifies one seller’s good or
service as distinct from those of other sellers’. The legal term for brand is trademark. A
brand may identify one item, a family of items, or all items of that seller. If used for the
firm as a whole, the preferred term is trade name

Brand name: Axe

Axe was launched in France in 1983 by Unilever. It was inspired by another of Unilever's
brands, Impulse. Unilever were keen to capitalize on Axe's French success and the rest of
Europe from 1985 onwards, later introducing the other products in the range. Unilever
were unable to use the name Axe in the United Kingdom and Ireland due to trademark
problems so it was launched as Lynx. Although Axe's lead product is the fragranced
aerosol deodorant body spray, other formats of the brand exist. Within underarm care the
following are available: deodorant aerosol body spray, deodorant stick, deodorant roll-on,
anti-perspirant aerosol spray (called Axe Dry), and anti-perspirant stick (also called Axe
Dry). The attribute of the brand that customer associates with his/ her belief. A person may
associate the brand for power, strength or protectiveness. For example, a customer may
associate Axe brand not just for perfumes but also any accessory associated with perfumes
such as Shampoos etc. So, for him, Axe represents perfume.

Brand Equity

Brand equity is set of assets linked to a brand‘s name and symbol that adds value to the
product or service and/or that firm’s customer.
Components of brand equity:
1. Brand loyalty: From its launch (Axe), the yearly fragrance variant has played a key part
in the success of the brand by offering something new each year.

Chandan Kumar Sah Page 71 of 152 MBA 2ND Semester


2. Brand awareness: The type of fragrance (Axe) variants have evolved over time. From
1983 until about 1989, the variant names were descriptions of the fragrances and included
Musk, Spice, Amber, Marine, and Oriental.
3. Perceived quality
4. Brand associations: Axe also launches limited edition variants from time to time that
may be on sale for a few months or over a year

b) Below are the various research approaches that need to be considered before making a
consumer survey regarding footwear products:
1) Observational Research – Fresh data can be collected by observing the situation
and the people in the situation.
2) Focus Group Research is a method of discussion in which a team of eight to
twelve persons invited for a group discussion in presence of a skilled moderator to
discuss a product, service, a firm or any marketing related activity. The proceedings are
observed and recorded on videotape and subsequently analyzed to understand consumer
attitudes, beliefs and behavior.
3) Survey Research – This is the most common of the approaches wherein surveys
are undertaken with the help of a questionnaire to learn about people’s knowledge,
beliefs and preferences.
4) Behavioral Research – Customer’s actual behavior in terms of actual purchases
reflect their preferences and are more reliable than responses provided in surveys which
are memory based.
5) Experimental Research – The most scientific method of research is experimental
research which tries to capture cause and affect relationships.

Q5. a. What advice would you give a company that has facing bad publicity? What steps
would you tell the company to improve its reputation?
b. As a brand manager, what are the ways in which you will select a brand name for
your product – watches and how will you position it in the market?
Ans: a) Bad Publicity: The following are the ways in which organizations can avoid or minimize
the effects of bas publicity:

Chandan Kumar Sah Page 72 of 152 MBA 2ND Semester


1. Providing people with the accurate information and giving clarifications in needed
either through press release, media interviews, websites, public messages, advertising
etc.
2. Company’s top management or spokesperson can give a public statement or comment
in the various media.
3. Improvising Public Relations and designing good publicity message to erase the
effects of bad publicity.
4. Continuing to provide quality products and services to the consumers.
5. Involving in community work or environmental protection campaigns or any such
activity for a good cause.

b) As a brand manager, the following are the ways in which I will select a brand name for
our product – Watches:
1. It should suggest something about the product benefits and qualities;
2. It should be easy to pronounce, recognize, and remember;
3. The brand name should be distinctive;
4. IT should be extendable;
5. The name should be easily translated into a foreign language;
6. It should be capable of registration and legal protection.

Q6. a. What is MIS? What are its benefits?


b. How is rural marketing different from urban market?

Ans: a) MIS: MIS stands for Marketing Information Systems which provides the marketing
department information about buyer wants, preferences, behavior and also about
competition. They are able to do this by setting up systems and marketing related research
methods to collect this valuable information which is ultimately used to make marketing
decisions. A MIS is a set of procedures to collect, analyze and distribute accurate, prompt
and appropriate information to different levels of marketing decision makers.
The benefits of MIS is as under:
i) It allows marketing managers to carry out their analysis, planning implementation and
control responsibilities more effectively.

Chandan Kumar Sah Page 73 of 152 MBA 2ND Semester


ii) It ensures effective tapping of marketing opportunities and enables the company to
develop effective safeguard against emerging marketing threats.
iii) It provides marketing intelligence to the firm and helps in early spotting of changing
trends.
iv) It helps the firm adapt its products and services to the needs and tastes of the
customers.
v) By providing quality marketing information to the decision maker, MIS helps in
improving the quality of decision making.

b) The different between rural markets and urban markets are as under:
1. Accessibility and mobility: This applies both for the supplier and the consumer.
The movement of the people is restricted by the lack of surface roads and the
mode of transport. There are restrictions by way of visibility during night.
2. Average income level of consumers: The average wage earners are characterized
by lower per capita income and disposable income in comparison to the urban.
3. Geographical distances: The living quarters are separated more than they are in
the urban areas. The cluster of villages is also segregated by distances.
4. Literacy level: On an average the literacy level in the rural sector is lower in
comparison to the urban sector.
There could be several other issues which are specific to the rural sector. These may
force marketers to take a different approach for the entire marketing process or at least
some of them as against the urban sector.

Chandan Kumar Sah Page 74 of 152 MBA 2ND Semester


Chandan Kumar Sah Page 75 of 152 MBA 2ND Semester
Assignment Set – 1

Q1. What is MIS? Define the characteristics of MIS? What are the basic Functions of
MIS? Give some Disadvantage of MIS?
Ans: Management information system: MIS stands for Management Information System is a
system or process that provides information needed to manage organizations effectively.
Management information systems are regarded to be a subset of the overall internal
controls procedures in a business, which cover the application of people, documents,
technologies, and procedures used by management accountants to solve business problems
such as costing a product, service or a business-wide strategy. Management information
systems are distinct from regular information systems in that they are used to analyze other
information systems applied in operational activities in the organization. Academically, the
term is commonly used to refer to the group of information management methods tied to
the automation or support of human decision making, e.g. Decision Support Systems,
Expert Systems, and Executive information systems. Organized approach to the study of
information needs of a management at every level in making operational, tactical, and
strategic decisions. Its objective is to design and implement man-machine procedures,
processes, and routines that provide suitably detailed reports in an accurate, consistent, and
timely manner. Modern, computerized systems continuously gather relevant data, both
from inside and outside the organization. This data is then processed, integrated, and stored
in a centralized database (or data warehouse) where it is constantly updated and made
available to all who have the authority to access it, in a form that suits their purpose.

Characteristics of MIS:
1. Management oriented: The system is designed form the top to work downwards. It
does not mean that the system is designed to provide information directly to the top
management. Other levels of management are also provided with relevant
information.
2. Management directed: Management orientation of MIS, it is necessary that
management should continuously make reviews. For example, in the marketing
information system, the management must determine what sales information is
necessary to improve its control over marketing operations.

Chandan Kumar Sah Page 76 of 152 MBA 2ND Semester


3. Integrated: The word 'integration' means that system has to cover of all the functional
areas of an organization so as to produce more meaningful management information,
with a view to achieving the objectives of the organization. It has to consider various
sub-Systems, their objectives, information needs, and recognize the independence,
that these sub-systems have amongst themselves, so that common areas of
information are identified and processed without repetition and overlapping. For
example, in the development of an effective production scheduling system, a proper
balance amongst the following factors is desired:
I. Set up costs
ii. Overtime
iii. Manpower
iv. Production capacity
v. Inventory level
vi. Money available
vii. Customer service.
4. Common data flows: The integration concept of MIS, common data flow concept
avoids repetition and overlapping in data collection and storage, combining similar
functions, and simplifying operations wherever possible. For example, in the
marketing operations, orders received for goods become the basis billing of goods
ordered, setting up of the accounts receivable, initiating production activity, sales
analysis and forecasting etc.
5. Heavy element: A management information system cannot be established overnight.
It takes almost 2 to 4 years to establish it successfully in an organization. Hence,
long-term planning is required for MIS development in order to fulfill the future
needs and objectives of the organization. The designer of an information system
should therefore ensure that it would not become obsolete before it actually gets into
operation. An example of such a feature of MIS may be seen in a transportation
system where a highway is designed not to handle today's traffic requirements but to
handle the traffic requirements five to ten years.
6. Flexibility and ease of use: While building an MIS system all types of possible
means, which may occur in future, are added to make it flexible. A feature that often
goes with flexibility is the ease of use. The MIS should be able to incorporate all

Chandan Kumar Sah Page 77 of 152 MBA 2ND Semester


those features that make it readily accessible to wide range of users with easy
usability.
The basic functions of MIS are:
1. Data Processing: Gathering, storage, transmission, processing and getting output of the
data. Making the data into information is a major task.
2. Prediction: Prediction is based on the historical data by applying the prior knowledge
methodology by using modern mathematics, statistics or simulation. Prior knowledge
varies on the application and with different departments.
3. Planning: Planning reports are produced based on the enterprise restriction on the
companies and helps in planning each functional department to work reasonably.
4. Control: MIS helps in monitoring the operations and inspects the plans. It consists of
differences between operation and plan with respect to data belonging to different
functional department. It controls the timely action of the plans and analyzes the
reasons for the differences between the operations and plan.
5. Assistance: It stores the related problems and frequently used information to apply
them for relative economic benefits. Through this it can derive instant answers of the
related problem.
6. Database: This is the most important functions of MIS. All the information is needs a
storage space which can be assessed without causing any anomalies in the data.
Integrated Database avoids the duplication of data and thereby reduces redundancy and
hence consistency will be increased.
7. The major functions of MIS lies in application of the above functions to support the
managers and the executives in the organization in decision making.

Disadvantages of MIS are as under:


1. MIS is highly sensitive: MIS is very helpful in maintaining logging information of an
authorized user. This needs to monitor constantly.
2. Quality of outputs is governed by quality of inputs.
3. MIS budgeting: There is difficulty in maintaining indirect cost and overheads.
Capturing the actual cost needs to have an accrual system having true costs of outputs
which is extremely difficult. It has been difficult to establish definite findings.
4. MIS is not flexible to update itself for the changes.

Chandan Kumar Sah Page 78 of 152 MBA 2ND Semester


5. The changes in the decision of top level management decrease its effectiveness.
6. Information accountability is based on the qualitative factors and the factors like
morality, confidence or attitude will not have any base.

Q2. Explain Knowledge based system? Explain DSS and OLAP with example?
Ans: Knowledge based System: KBS are the systems based on Knowledge base. Knowledge
base is the database maintained for knowledge management which provides the means of
data collections, organization and retrieval of knowledge. The knowledge management
manages the domain where it creates and enables organization for adoption of insights and
experiences. There are two types of knowledge bases; a) Machine readable knowledge
bases, and b) Human readable knowledge bases. KBS refers to a system of data and
information used for decision market. The system is automated to work on the knowledge
based data and information required in a particular domain of management activity. The
processing is done based on the past decisions taken under suitable conditions. Decision
making is based on the fact that the condition is similar to the past situation hence the
decision is also is similar.

DSS: DSS stands for Decision Support Systems. DSS is an interactive computer
based system designed to help the decision makers to use all l the resources
available and make use in the decision making. In management many a time
problems arise out of situations for which simple solution may not be possible. To
solve such problems you may have to use complex theories. The models that would
be required to solve such problems may have to be identified. DSS requires a lot of
managerial abilities and managers judgment. You may gather and present the
following information by using decision support application: a) Accessing all of
your current information assets, including legacy and relational data sources,
cubes, data warehouses, and data marts, b) Comparative sales figures between one
week and the next, c) Projected revenue figures based on new product sales
assumptions, d) The consequences of different decision alternatives, given past
experience in a context that is described. We can summarize the benefits of DSS
into following:

Chandan Kumar Sah Page 79 of 152 MBA 2ND Semester


• Improves personal efficiency
• Expedites problem solving
• Facilitates interpersonal communication
• Promotes learning or training
• Increases organizational control
• Generates new evidence in support of a decision
• Creates a competitive advantage over competition
• Encourages exploration and discovery on the part of the decision maker
• Reveals new approaches to thinking about the problem space

OLAP: OLAP stands for Online Analytical Processing. OLAP refers to a system in
which there are predefined multiple instances of various modules used in business
applications. Any input to such a system results in verification of the facts with
respect to the available instances. A nearest match is found analytically and the
results displayed from the database. The output is sent only after thorough
verification of the input facts fed to the system. The system goes through a series of
multiple checks of the various parameters used in business decision making. OLAP
is also referred to as a multi dimensional analytical model. Many big companies use
OLAP to get good returns in business. The querying process of the OLAP is very
strong. It helps the management take decisions like which month would be
appropriate to launch a product in the market, what should be the production
quantity to maximize the returns, what should be the stocking policy in order to
minimize the wastage etc. A model of OLAP may be well represented in the form
of a 3D box. There are six faces of the box. Each adjoining faces with common
vertex may be considered to represent the various parameter of the business
situation under consideration. E.g.: Region, Sales & demand, Product etc.

Q3. What are Value Chain Analysis & describe its significance in MIS? Explain what is
meant of BPR? What is its significance? How Data warehousing & Data Mining is
useful in terms of MIS?

Chandan Kumar Sah Page 80 of 152 MBA 2ND Semester


Ans: Value Chain Analysis: Value-addition production, marketing delivery, and servicing of the
product. These activities are connected in a chain. Support activities include those
providing purchased inputs, technology, human resources, or overall infrastructure
functions to support eh primary activities. It is possible to reduce the transaction cost by
proper coordination of all the activities. It should be possible to gather better information
for various controls and also replace the same by less costlier activities. It will also be
possible to reduce the overall time required to complete an activity. Therefore coordination
is very important to achieve competitive advantage. For this it is necessary to manage the
value chain as a system rather than as separate parts. An enterprise’s value chain for
competing in a particular industry is embedded in a large stream of activities. What Porter
termed as value system may be referred to as the industry value-chain. This chain consists
of mainly the suppliers and distribution channels. Any activity of an organization is
subjected to one or more of the following - New Technologies, Shifting buyer needs,
Variation in industry segmentation, Changes in the costs, Changes in government
regulations.

BPR: BPR stands for Business Process Re-engineering. BPR is used to gain an
understanding the process of business and to understand the process to make it better and
re-designing and thereby improving the system. BPR is mainly used for change in the work
process. Latest software is used and accordingly the business procedures are modified, so
that documents are worked upon more easily and efficiently. This is known as workflow
management.
Business process are a group of activities performed by various departments, various
organizations or between individuals that is mainly used for transactions in business. There
may be people who do this transaction or tools. We all do them at one point or another
either as a supplier or customer. You will really appreciate the need of process
improvement or change in the organizations conduct with business if you have ever waited
in the queue for a longer time to purchase 1 kilo of rice from Public Distribution Shop
(PDS-ration shop). The process is called the check-out process. It is called process because
uniform standard system has been maintained to undertake such a task. The system starts
with forming a queue, receiving the needed item from the shop, getting it billed, payment
which involves billing, paying amount and receiving the receipt of purchase and the

Chandan Kumar Sah Page 81 of 152 MBA 2ND Semester


process ends up with the exit from the store. It is the transaction between customer and
supplier. BPR is a major innovation changing the way organizations conduct their business.
Such changes are often necessary for profitability or even survival. BPR is employed when
major IT projects such as ERP are undertaken; Reengineering involves changes in
structure, organizational culture and processes. Many concepts of NPR changes
organizational structure. Team based organization, mass customization; empowerment and
telecommuting are some of the examples.

Data Mining: Data mining is primarily used as a part of information system today, the
companies with strong consumer focus- retail, financial, communication, and marketing
organizations. It enables these companies to determine relationships among “internal”
factors such as price, product positioning, or staff skills, and “external” factors such as
economic indicators, competition, and customer demographics. And, it enables them to
determine the impact on sales, customer satisfaction, and corporate profits. Finally, it
enables them to “drill down” into summary information to view detail transactional data.
With data mining, a retailer could use point-of-sale records of customer purchases to send
targeted promotions based on an individual’s purchase history. Bu mining demographic
data from comment or warranty cards, the retailer could develop products and promotions
to appeal to specific customer segments. Data mining is a collaborative tool which
comprises of database systems, statistics, machine learning, visualization and information
science. Based on the data mining approach used, different techniques from the other
discipline can be used such as neural networks, artificial intelligence, fuzzy logic,
knowledge representation, high performance computing and inductive logic programming.
Data mining refers to extracting or mining knowledge from large amount of data. There
may be other terms which refer data mining such as knowledge mining, knowledge
extraction, data/pattern analysis, data archeology, and data dredging.

Data Warehousing: Data Warehousing is defined as collection of database which is


referred as relational database for the purpose of querying and analysis rather than just
transaction processing. Data warehouse is usually maintained to store heuristic data for
future use. Data warehousing is usually used to generate reports. Integration and separation
of data are the two basic features need to be kept in mind while creating a date

Chandan Kumar Sah Page 82 of 152 MBA 2ND Semester


warehousing. The main output from data warehouse systems are; either tabular listings
(queries) with minimal formatting or highly formatted “formal” reports on business
activities. This becomes a convenient way to handle the information being generated by
various processes. Data warehouse is an archive of information collected from wide
multiple sources, stored under a unified scheme, at a single site. This data is stored for a
king time permitting the user an access to archived data for years. The data stored under a
unified scheme, at a single site. This data is stored for a long time permitting the user an
access to archived data for years. The data stored and the subsequent report generated out
of a querying process enables decision making quickly. This concept is useful for big
companies having plenty of data on their business processes.

Q4. Explain DFD & Data Dictionary? Explain in detail how the information requirement
is determined for an organization?
Ans: DFD: DFD stands for Data flow diagrams which represent the logical flow of data within
the system. DFD do not explain how the processes convert the input data into output. They
do not explain how the processing takes place. DFD uses few symbols like circles and
rectangles connected by arrows to represent data flows. DFD can easily illustrate
relationships among data, flows, external entities and stores. DFD can also be drawn in
increasing levels of detail, starting with a summary high level view and proceeding of more
detailed lower level views. The number of guidelines should be used in constructing DFD.
a) Choose meaningful names for the symbols on the diagram; b) Number the processes
consistently. The numbers do not imply the sequence; c) Avoid over complex DFD; d)
Make sure the diagrams are balanced.

Data Dictionary: The data dictionary is used to create and store definitions of data,
location, format for storage and other characteristics. The data dictionary can be used to
retrieve the definition of data that has already been used in an application. The data
dictionary also stores some of the description of data structures, such as entities, attributes
and relationships. It can also have software to update itself and to produce reports on its
contents and to answer some of the queries.

Chandan Kumar Sah Page 83 of 152 MBA 2ND Semester


The information requirement’s determination for an organization: The some purpose of
the MIS is to produce such information which will reduce uncertainty risk in a given
situation. There are four methods of determining the information requirements. They are:
1. Asking or Interviewing: In this method a designer of the MIS puts questions or
converses with the user of the information and determines the information
requirements. Putting the questions is an art and it should be used properly to seek
information. When the user has to select one answer from a finite set of answers a
closed question should be asked. But an open question is put, when the user has no
precise knowledge but has an ability to determine all answers to select one out of
them? In open questions, the answers may not be immediate but can be obtained by
surveying the domain knowledge of the user. When multiple users or several
decision-makers in similar functions or positions are involved, a brain storming
session is performed to cover all possible answers to the questions. When several
users are involved, group consensus can be sought to get the most feasible set of
answers. The experts or experienced users are asked to give their best answers – this
approach is called the Delphi method.
2. Determining from the Existing System: In a number of cases the existing system,
when has been evolved after a number of years, and has been designed out of
experience gives straightaway the requirement of information. In any situations,
systems from other companies can give additional information requirements. The
fund of knowledge is available from the textbooks, handbooks, research studies
which can determine the information requirement. Irrespective of the type of
organization and business, ninety per cent of the information requirement is common
and the balance ten per cent may be typical to the organization or the business, which
needs to be determined separately. The managers in the operations and the middle
management use the existing systems as a reference for determining the information
requirements. This method is adopted when the rules and decision methods are
outside the purview of the decision-maker.
3. Analysing the Critical Success Factors: Every business organization performs
successfully on efficient management of certain critical success factors. Other factors
are important and play a support role in the functioning of the organization. Many
times a function is singularly critical to the successful functioning of a business

Chandan Kumar Sah Page 84 of 152 MBA 2ND Semester


organization. In a consumer industry, marketing and service becomes the critical
function. The information requirements of such organizations largely relate to these
critical factors. The analysis of these functions or factors will determine the
information requirements.
4. Experimentation and Modelling: When there is total uncertainty, the designer
and the user of the information resort to this method for determining the information
requirement. The experimentation would decide the methodology for handling the
complex situation. If the method is finalized, the information needs are determined as
they have been evolved through the experimentation. Test marketing of a product is
an approach of the experimentation to decide the correct marketing strategy.
Sometimes models are used for deciding the initial information needs and they are
modified during the implementation stage. The information requirements determined
through such methods undergo a qualitative change as the users get the benefit of
learning and experience and the needs may undergo a change or get replaced
completely.

Q5. What is ERP? Explain its existence before and its future after? What are the
advantages & Disadvantages of ERP? What is Artificial Intelligence? How is it
different from Neural Networks?
Ans: ERP: ERP stands for Enterprise Resource Planning. The ERP is typically reserved for
larger, more broadly based applications. The introduction of an ERP system to replace two
or more independent applications eliminates the need for external interfaces previously
required between systems, and provides additional benefits that range from standardization
and lower maintenance to easier and/or greater reporting capabilities. Examples of modules
in an ERP which formerly would have been standalone applications include:
Manufacturing, Supply Chain, Financials, Customer Relationship Management, Human
Resources, Warehouse Management and Decision Support System.
Prior to the concept of ERP systems, departments within an organization would have their
own computer systems. The HR computer system would typically contain information on
the department, reporting structure, and personal details of employees. The payroll
department would typically calculate and store paycheck information. The financial
department would typically store financial transactions for the organization. Each system

Chandan Kumar Sah Page 85 of 152 MBA 2ND Semester


would have to rely on a set of common data to communicate with each other. For the HRIS
to send salary information to the payroll system, an employee number would need to be
assigned and remain static between the two systems to accurately identify an employee.
The financial system was not interested in the employee-level data, but only in the payouts
made by the payroll systems, such as the tax payments to various authorities, payments for
employee benefits to providers, and so on. This provided complications. For instance, a
person could not be paid in the payroll system without an employee number.
ERP software, among other things, combined the data of formerly separate applications.
This made the worry of keeping numbers in synchronization across multiple systems
disappears. It standardized and reduced the number of software specialties required within
large organizations.

Advantages and Disadvantages of ERP:


Advantages: In the absence of an ERP system, a large manufacturer may find itself with
many software applications that do not talk to each other and do not effectively interface.
Tasks that need to interface with one another may involve: a) A Total integrated system, b)
The ability to streamline different processes and workflows, c) The ability to easily share
data across various departments in an organization, d) Improved efficiency and
productivity levels, e) Better tracking and forecasting, f) Lower costs. G) Improved
customer service. Change how a product is made, in the engineering details, and that is
how it will now be made. Effective dates can be used to control when the switch over will
occur from an old version to the next one, both the date some ingredients go into effect, and
date that some are discontinued. Part of the change can include labeling to identify version
numbers. Some security features are included within an ERP system to protect against both
outsider crime, such as industrial espionage, and insider crime, such as embezzlement. A
data tampering scenario might involve a disgruntled employee intentionally modifying
prices to below the breakeven point in order to attempt to take down the company, or other
sabotage. ERP systems typically provide functionally for implementing internal controls to
prevent actions of this kind. ERP vendors are also moving toward better integration with
other kinds of information security tools.
Disadvantages: Many problems organizations have with ERP systems are due to
inadequate investment in ongoing training for involved personnel, including those

Chandan Kumar Sah Page 86 of 152 MBA 2ND Semester


implementing and testing changes, as well as a lack of corporate policy protecting the
integrity of the data in the ERP systems and how it is used. While advantages usually
outweigh disadvantages for most organizations implementing an ERP system, here are
some of the most common obstacles experienced: Usually many obstacles can be prevented
if adequate investment is made and adequate training is involved, however, success does
depend on skills and the experience of the workforce to quickly adapt to the new system.
A) Customization in many situation is limited, b) The need to reengineer business
processes, c) ERP systems can be cost prohibitive to install and run, d) Technical support
can be shoddy, e) ERP’s may be too rigid for specific organizations that are either new or
want to move in a new direction in the near future.

Artificial Intelligence: Artificial Intelligence is the science and technology based on


various functions to develop a system that can think and work like a human being. It can
reason, analyze, learn, conclude and solve problems. The systems which use this type of
intelligence are known as artificial intelligent system and their intelligence is referred to as
artificial intelligence. Artificial Intelligence can be classified into various branches like
Natural Language Processing, Speech Recognition, Automated Programming, Machine
Learning, Pattern Recognition and Probabilistic Networks.
Neural Networks: Neural Network is a system which tries to learn from the database and
the manager then decides what the right answer is. The entire neural network is realized in
the form of software. The software renders the computer as a problem solver. The neural
networks goes on building strong database for problem solving depending upon the
decisions taken by the manager in the form of response confirmation given to the system by
the manager. Neural networks can be used in various business applications like forecasting,
stock analysis, market analysis etc.
Difference between Artificial Intelligence and Neural Networks: Artificial intelligence is
afield of science and technology based on disciplines. The goal of Artificial Intelligence is
to develop computers that can simulate the ability to think, see, hear, walk, talk and feel.
Artificial Intelligence can be grouped under three major areas: 1) Cognitive Science, 2)
Robotics and 3) Natural Interfaces. Cognitive science focuses on researching on how the
human brain works and how humans think and learn. Applications in the cognitive science
area of Artificial Intelligence include the development of expert systems and other

Chandan Kumar Sah Page 87 of 152 MBA 2ND Semester


knowledge based systems that add a knowledge base and some reasoning capability to
information systems.
Whereas Neural Network software can learn by processing sample problems and their
solutions. As neural nets start to recognize patterns, they can begin to program themselves
to solve such problems on their own. Neural networks are computing systems modeled
after the human brain’s mesh like network of interconnected processing elements, called
neurons. The human brain is estimated to have over 100 billion neuron brain cells. The
neural networks are lot simpler in architecture. Like the brain, the interconnected
processors in a neural network operate in parallel and interact dynamically with each other.
This enables the network to operate and learn from the data it processes, similar to the
human brain. That is, it learns to recognize patterns and relationships in the data. Thus, the
neural networks will change the strengths of the interconnections between the processing
elements in response to changing patterns in the data it receives and results that occur.

Q6. Distinguish between closed decision making system & open decision making system?
What is ‘What – if’ analysis? Why is more time spend in problem analysis & problem
definition as compared to the time spends on decision analysis?

Ans: The decision-making systems can be classified in a number of ways. There are two types of
systems based on the manager’s knowledge about the environment. If the manager operates
in a known environment then it is a closed decision-making system. The conditions of the
closed decision making system are:- 1) The manager has a known set of decision
alternative and knows their outcomes fully in terms of value, if implemented; 2) The
manager has a model, a method or a rule whereby the decision alternatives can be
generated, tested, and ranked for selection; 3) The manager can choose one of them, based
on some goal or objective criterion. Few examples are a product mix problem, an
examination system to declare pass or fail, or an acceptance of the fixed deposits. If the
manager operated in an environment not known to him, then the decision making system is
termed as an open decision making system. The conditions of this system in contrast closed
decision making system are: 1) The manager does not know all the decision alternatives; 2)
The outcome of the decision is also not known fully. The knowledge of the outcome may
be a probabilistic one; 3) No method, rule or model is available to study and finalize one

Chandan Kumar Sah Page 88 of 152 MBA 2ND Semester


decision among the set of decision alternatives; 4) It is difficult to decide an objective or a
goal and, therefore, the manager resorts to that decision, where his aspirations or desires are
met best. The MIS tries to convert every open system to a closed decision making system
by providing information support for the best decision. The MIS gives the information
support, whereby the manager knows more and more about environment and the outcomes,
he is able to generate the decision alternatives, test them and select one of them.

What if analysis: Decisions are made using a model of the problem for developing various
solution alternatives ant testing them for best choice. The model is built with some
variables and relationship between variables. In reality, the considered values of variables
or relationship in the model may not hold good and therefore solution needs to be tested for
an outcome, if the considered values of variables or relationship change. This method of
analysis is called ‘what if analysis.’ ‘What if analysis’ creates confidence in decision-
making model by painting a picture of outcomes under different conditions?

More time spent in problem analysis & problem definition as compared to the time
spends on decision analysis:
The manager, being a human being, behaves in a peculiar way in a given situation. The
response of one manager may not be the same as that of the two other managers, as they
differ on the behavioural platform. Even though tools, methods and procedures are
evolved, the decision is many a times influenced by personal factors such as behaviour.
The managers differ in their approach towards decision-making in the organization, and,
therefore, they can be classified into two categories, viz., the achievement-oriented, i.e.,
looking for excellence and the task-oriented, i.e., looking for the completion of the task
somehow. The achievement-oriented manager will always opt for the best and, therefore,
will be enterprising in every aspect of the decision-making. He will endeavour to develop
all the possible alternatives. He would be scientific, and therefore, more rational. He would
weigh all the pros and cons properly and then conclude. The manager’s personal values
will definitely influence ultimately. Some of the managers show a nature of risk avoidance.
Their behaviour shows a distinct pattern indicating a conservative approach to decision-
making – a path of low risk or no risk. Further, even though decision-making tools are
available, the choice of the tools may differ depending on the motives of the manager. The

Chandan Kumar Sah Page 89 of 152 MBA 2ND Semester


motives are not apparent, and hence, are difficult to understand. A rational decision in the
normal course may turn out to be different on account of the motives of the manager. The
behaviour of the manager is also influenced by the position he holds in the organisation.
The behaviour is influenced by a fear and an anxiety that the personal image may be
tarnished and the career prospects in the organisation may be spoiled due to a defeat or a
failure. The managerial behaviour, therefore, is a complex mix of the personal values, the
atmosphere in the organisation, the motives and the motivation, and the resistance to
change. Such behaviour sometimes overrides normal decisions based on business and
economic principles. The interplay of different decision-making of all the managers in the
organisation shapes up the organisational decision-making. The rationale of the business
decision will largely depend upon the individuals, their positions in the organisation and
their inter-relationship with other managers. If two managers are placed in two decision-
making situations, and if their objectives are in conflict, the managers will arrive at a
decision objectively, satisfying individual goals. Many a times, they may make a conscious
decision, disregarding organisation’s objective to meet their personal goals and to satisfy
their personal values. If the manager is enterprising, he will make objectively rational
decisions. But if the manager is averse to taking risk, he will make a decision which will be
subjectively rational as he would act with limited knowledge and also be influenced by the
risk averseness. Thus, it is clear that if the attitudes and the motives are not consistent
across the organisation, the decision-making process slows down in the organisation.

Chandan Kumar Sah Page 90 of 152 MBA 2ND Semester


Assignment Set – 2

Q1. How hardware & software support in various MIS activities of the organization?
Explain the transaction stages from manual system to automated systems?
Ans: Hardware in the form of personal computers and peripherals like printers, fax machines,
copier, scanners etc are used in organization to support various MIS activities of the
organization. It support in various MIS activities of the organization in the following ways:
a) Speed:- A PC can process data at a very high speed. It can process millions of
instructions within fraction of seconds.
b) Storage:- APC can store large quantity of data in a small space. The storage system in a
PC is such that the information can be transferred from one place to another place in
electronic form.
c) Communication:- A PC on the network can offer great support as a communicator in
communicating information in the form of text and images.
d) Accuracy:- A PC is highly reliable in the sense that it could be used to perform
calculations continuously for hours with a great degree of accuracy. It is possible to
obtain mathematical results correct up to a great degree of accuracy.
e) Conferencing:- A PC with internet offers facility of video conferencing worldwide.
Business people across the globe travel a lot to meet their business activities. By video
conferencing inconvenience of traveling can be avoided.

The transaction stages from manual system to automated systems: The value of
Information is not present day discovery. We have always observed that the Information is
the asset of any organization. The existence of information is since the ‘Big bang’
happened and then on it went on. But the value of information is being used only after the
industrial revolution. Before, it was only in the record which we are using now in an
efficient way. The first information was binary, information is generated by interactions;
information is by interaction, as without comparison, without a context, without interaction,
there is nothing. Traditional information systems are said to contain data, which is then
processed. The processed data is called information. The processing of data takes place by
selecting the required fact and organizing it in a way to form meaningful information which
is used for some organizational needs. In Manual systems, a series of action takes which

Chandan Kumar Sah Page 91 of 152 MBA 2ND Semester


may be similar as well as different to processing in traditional systems. For instance, in
hospital information systems the patient details can be viewed by the administrator as well
as patient. But the views perceived by these are different. One may view it as a record to
take print and other may be the source of his ailment description. What is common to the
two systems is the idea of transformation. Transformation occurs when systems
participants are faced with cues from their environment, which may be data or situations,
and the participants then define and redefine what to do next, either processing data or
developing a situation, altering the system each time to transform it to a state closer to the
participants’ goal or objective. When a fact from either type of system is presented for
manipulation, a transformation can occur. Thus, transformation is common to both types of
systems. A transformation had to necessarily go through the following stages:-
a) Appraisal of the procedures
b) Types of documents
c) Storage systems
d) Formulations and coding
e) Verification and validation
f) Review
g) Documentation
After the industrial revolution slowly manual systems were transformed into digital form
by means of computer and related instruments.

Q2. Explain the various behavioral factors of management organization? As per Porter,
how can performance of individual corporations be determined?
Ans: Behavioral Factors: The implementation of computer based information systems in
general and MSS in particular is affected by the way people perceive these systems and by
how they behave in accepting them. User resistance is a major behavioral factor associated
with the adoption of new systems. The major behavioral factors are:-
a) Decision Styles:- Symbolic processing of Artificial Intelligence is heuristic; DSS and
ANN are analytic
b) Need for explanation:- ES provides explanation, ANN does not, DSS may provide
partial explanation. Explanation can reduce resistance to change

Chandan Kumar Sah Page 92 of 152 MBA 2ND Semester


c) Organizational climate:- Some organizations lead and support innovations and new
technologies whereas others wait and lag behind in making changes
d) Organizational expectations:- Over expectation can result in disappointments and
termination of innovation. Over expectation was observed in most early intelligent
systems.
e) Resistance to change:- Can be strong in MSS because the impacts may be significant.
Many can resist.
Performance: The performance of individual corporations is determined by the extent to
which they manage the following:
a) the bargaining power of suppliers;
b) the bargaining power of buyers;
c) the treat of new entrants;
d) the treat of substitute products; and
e) Rivalry among existing firms.

Q3. Compare various types of development aspect of Information System? Explain the
various stages of SDLC?
Ans: Types of development aspect of Information System:
a) Development and Implementation of MIS: Once the plan for MIS is made, the
development of the MIS, calls for determining the strategy of development. The
development strategy determines where to begin and in what sequence the development
can take place with the sole objective of assuring the information support. The designer
first develops systems independently and starts integrating them with other systems,
enlarging the system scope and meeting the varying information needs. Determining
the position of the system in the MIS is easy. The real problem is the degree of
structure, and formalization in the system and procedures which determine the timing
and during of development of the system. Higher the degree of structured-ness and
formalization, greater is the stabilization of the rules, the procedures, decision-making
and the understanding of the overall business activity. The development becomes a
method of approach with certainty in input process and outputs.
b) Prototype Approach: When the system is complex, the development strategy is
Prototyping of the System. Prototyping is a process of progressively ascertaining the

Chandan Kumar Sah Page 93 of 152 MBA 2ND Semester


information needs, developing methodology, trying it out on a smaller scale with
respect to the data and the complexity, ensuring that it satisfies the needs of the users,
and assess the problems of development and implementation. This process, therefore,
identifies the problem areas, inadequacies in the prototype vis-à-vis fulfillment of the
information needs. The designer then takes steps to remove the inadequacies. This may
call upon changing the prototype of the system, questioning the information needs,
streamlining the operational systems and procedures and move user interaction. In the
prototyping approach, the designer’s task becomes difficult, when there are multiple
users of the same system and the inputs they use are used by some other users as well.
c) Life Cycle Approach: There are many systems or sub-systems in the MIS which have a
life cycle, that is, they have birth and death. Their emergence may be sudden or may be
a part of the business need, and they are very much structured and rule-based. They
have 100% clarity of inputs and their sources, a definite set of outputs in terms of the
contents and formats. These details more or less remain static from the day the system
emerges and remains in that static mode for a long time. Minor modifications or
changes do occur but they are not significant in terms of handling either by the designer
or the user of the system. Such systems, therefore, have a life and they can be
developed in a systematic manner, and can be reviewed after a year or two, for
significant modification, if any. These systems have a fairly long duration of survival
and they contribute in a big way as sources of data to the Corporate MIS. Therefore,
their role is important and needs to be designed from the view point as an interface to
the Corporate MIS.
d) Implementation of the Management Information System: The implementation of the
system is a management process. It brings about organizational change; it affects
people and changes their work style. The process evokes a behavior response which
could be either favorable or unfavorable depending upon the strategy of system
implementation. In the process of implementation, the system designer acts as a change
agent or a catalyst. For a successful implementation he has to handle the human factors
carefully. The user of the system has a certain fear complex when a certain cultural
work change is occurring. The first and the foremost fear is about the security to the
person if the change-over from the old to new is not a smooth one. Care has to be taken
to assure the user that such fears are baseless and the responsibility, therefore, rests

Chandan Kumar Sah Page 94 of 152 MBA 2ND Semester


with the designer. The second fear is about the role played by the person in the
organisation and how the change affects him. On many occasions, the new role may
reduce his importance in the organisation, the work design may make the new job
impersonal, and a fear complex may get reinforced that the career prospects may be
affected.

SDLC: System development cycle stages are sometimes known as system study. System
concepts which are important in developing business information systems expedite
problem solving and improve the quality of decision making. The system analyst has to do
a lot in this connection. They are confronted with challenging task of creating new systems
and planning major changes in the organization. The system analyst gives a system
development project, meaning and direction. The various stages of SDLC are:
a) Feasibility study:- It is concerned with determining the cost effectiveness of various
alternatives in the designs of the information system and the priorities among the
various system components.
b) Requirements, collection and analysis:- It is concerned with understanding the mission
of the information systems, that is, the application areas of the system within the
enterprise and the problems that the system should solve.
c) Design:- It is concerned with the specification of the information systems structure.
There are two types of design: database design and application design. The database
design is the design of the database design and the application design is the design of
the application programs.
d) Prototyping:- A prototype is a simplified implementation that is produced in order to
verify in practice that the previous phases of the design were well conducted.
e) Implementation:- It is concerned with the programming of the final operational version
of the information system. Implementation alternatives are carefully varies and
compared.
f) Validation and testing:- It is the process of assuring that each phase of the
development process is of acceptable quality and is an accurate transformation from the
previous phase.

Chandan Kumar Sah Page 95 of 152 MBA 2ND Semester


Q4. Compare & Contrast E-enterprise business model with traditional business
organization model? Explain how in E-enterprise manager role & responsibilities are
changed? Explain how manager is a knowledge worker in E-enterprise?
Ans: The comparison between E-enterprise business model with tradition business organization
model are as under:
Traditional Organization Design E- enterprise
Top heavy organization structure Flat organization structure
Work & work place location at one place Separation of work from work place location
Manual & document-based work flows Paperless work flows
High administrative & management Low overheads
overheads
Inflexible, rigid and longer business Flexible agile and responsive process cycles
process cycles
Private business process systems for self Public business processes and systems for
use. They are barred for usage to use by customers, vendors and business
customers, vendors and business partners partners
Low usage of technology Use internet, wireless and network
technologies

In E enterprise, business is conducted electronically. Buyers and sellers through Internet


drive the market and Internet based web systems. Buying and selling is possible on
Internet. Books, CDs, computer, white goods and many such goods are bought and sold on
Internet. The new channel of business is well known as Ecommerce. On the same lines,
banking, insurance, healthcare are being managed through Internet E banking, E billing, E
audit, & use of Credit cards, Smart card, ATM, E money are the examples of the
Ecommerce application. The digital firm, which uses Internet and web technology and uses
E business And Ecommerce solutions, is a reality and is going to increase in number. MIS
for E business is different compared to conventional MIS design of an organization. The
role of MIS in E business organization is to deal with changes in global market and
enterprises. MIS produces more knowledge based products. Knowledge management
system is formally recognized as a part of MIS. It is effectively used for strategic planning
for survival and growth, increase in profit and productivity and so on. To achieve the said
benefits of E business organization, it is necessary to redesign the organization to realize
the benefits of digital firm. The organization structure should be lean and flat. Get rid of

Chandan Kumar Sah Page 96 of 152 MBA 2ND Semester


rigid established infrastructure such as branch office or zonal office. Allow people to work
from anywhere. Automate processes after reengineering the process to cut down process
cycle time. Make use of groupware technology on Internet platform for faster response
processing. Another challenge is to convert domestic process design to work for
international process, where integration of multinational information systems using
different communication standards, country specific accounting practices, and laws of
security are to be adhered strictly. Internet and networking technology has thrown another
challenge to enlarge the scope of Organization where customers and vendors become part
of the organization. This technology offers a solution to communicate, coordinate, and
collaborate with customers, vendors and business partners. This is just not a technical
change in business operations but a cultural change in the mindset of managers and
workers to look beyond the conventional organization. It means changing the organization
behavior to take competitive advantage of the E business technology.

The last but not the least important is the challenge to organize and implement information
architecture and information technology platforms, considering multiple locations and
multiple information needs arising due to global operations of the business into a
comprehensive MIS.

Q5. What do you understand by service level Agreements (SLAs)? Why are they needed?
What is the role of CIO in drafting these? Explain the various security hazards faced
by as IS?

Ans: A service level agreement (frequently abbreviated as SLA) is a part of a service contract
where the level of service is formally defined. In practice, the term SLA is sometimes used
to refer to the contracted delivery time (of the service) or performance. As an example,
internet service providers will commonly include service level agreements within the terms
of their contracts with customers to define the level(s) of service being sold in plain
language terms (typically the (SLA) will in this case have a technical definition in terms of
MTTF, MTTR, various data rates, etc.) A service level agreement (SLA) is a negotiated
agreement between two parties where one is the customer and the other is the service
provider. This can be a legally binding formal or informal "contract" (see internal

Chandan Kumar Sah Page 97 of 152 MBA 2ND Semester


department relationships). Contracts between the service provider and other third parties
are often (incorrectly) called SLAs — as the level of service has been set by the (principal)
customer, there can be no "agreement" between third parties (these agreements are simply a
"contract"). Operating Level Agreements or OLA(s), however, may be used by internal
groups to support SLA(s). The SLA records a common understanding about services,
priorities, responsibilities, guarantees, and warranties. Each area of service scope should
have the "level of service" defined. The SLA may specify the levels of availability,
serviceability, performance, operation, or other attributes of the service, such as billing.
The "level of service" can also be specified as "target" and "minimum," which allows
customers to be informed what to expect (the minimum), whilst providing a measurable
(average) target value that shows the level of organization performance. In some contracts,
penalties may be agreed upon in the case of non-compliance of the SLA (but see "internal"
customers below). It is important to note that the "agreement" relates to the services the
customer receives, and not how the service provider delivers that service. SLAs have been
used since late 1980s by fixed line telecom operators as part of their contracts with their
corporate customers. This practice has spread such that now it is common for a customer to
engage a service provider by including a service-level agreement in a wide range of service
contracts in practically all industries and markets. Internal departments (such as IT, HR,
and Real Estate) in larger organization have adopted the idea of using service-level
agreements with their "internal" customers — users in other departments within the same
organization. One benefit of this can be to enable the quality of service to be benchmarked
with that agreed to across multiple locations or between different business units. This
internal benchmarking can also be used to market test and provide a value comparison
between an in-house department and an external service provider. Service-level agreements
are, by their nature, "output" based — the result of the service as received by the customer
is the subject of the "agreement." The (expert) service provider can demonstrate their value
by organizing themselves with ingenuity, capability, and knowledge to deliver the service
required, perhaps in an innovative way. Organizations can also specify the way the service
is to be delivered, through a specification (a service-level specification) and using
subordinate "objectives" other than those related to the level of service. This type of
agreement is known as an "input" SLA. This latter type of requirement is becoming

Chandan Kumar Sah Page 98 of 152 MBA 2ND Semester


obsolete as organizations become more demanding and shift the delivery methodology risk
on to the service provider.

Role of CIO: CIO stands for Chief Information Officer. CIO’s are senior executives
responsible for all aspects of their companies’ information technology and systems. They
direct the use of IT to support the company’s goals. The CIO of an organization has to play
the following roles:-
a) Leader: The CIO is part of the senior executive group and the most senior manager
responsible for IT. As a leader, the Information Systems Manager is responsible for
supervising, hiring, training, and motivating information systems personal. The CIO
leads all usage of IT from a business perspective.
b) Spokesman: This role incorporates activities that require the CIO to extend
organizational contacts outside the department to other areas of the organization. The
spokesman’s role demands that the information systems manager acts as an information
disseminator and politician, ensuring that the information systems department is
properly connected to the top level of the organization and to the key decision makers
in other departments.
c) Monitor: The CIO must scan the external environment to keep up with technical
changes and competition. A CIO is proactive and knowledgeable about opportunities
for the use of IT in the respective industry.
d) Liaison: The CIO is responsible for developing a working relationship between IT and
the business units that fits the organization’s objectives, management style, and culture.
The information systems manager must communicate with the external environment
including exchanging information with the departments of suppliers, customers, buyers,
market analysts, and the media. To meet a distinct set of information systems
requirements and work in joint supplier-customer teams, the CIO should communicate3
information systems capabilities to meet the requirements of suppliers and customers.
e) Enterpreneur: The CIO plays an important role in influencing the overall strategy and
planning in the organization and translates these organizational strategies and plan into
definitive information systems actions. A CIO identifies business needs and develops
solutions that change business situations. A proactive CIO not only sets MIS planning
objectives consistent with the firm’s overall objectives, but, as an organizational change

Chandan Kumar Sah Page 99 of 152 MBA 2ND Semester


agent, is also able to influence the future strategic direction and opportunities of the
firm.
f) Resource Allocator: The information systems manager must decide how to allocate
human, financial, and information resources. As the information systems department
acts as the main custodian of corporate data, information resource allocation decisions
are becoming critical to the success of organizations. The CIO acts as a steward of
information and IT resources for the organization, e.g., the finance head is the steward
of financial resources.
Security Hazards faced by an IS: The following security hazards are to be faced by an IS:
a) Malfunctions: In this type of security hazard, all the components of a system are
involved. People, software and hardware errors cause the biggest problem. More
dangerous are the problems which are created by human beings due to the omission,
neglect and incompetence.
b) Fraud and unauthorized access: The hazard is due to dishonesty, cheating or deceit.
This can be done through – i) Infiltration and industrial espionage, ii) Tapping data
from communication lines, iii) Unauthorized browsing through lines by online
terminals, etc.
c) Power and communication failure: In some locations they are the most frequent
hazards than any other else because availability of both of them depends upon the
location. Sometimes communication channel are busy or noisy. There are power cuts
and sometimes high voltage surge destroys a sensitive component of the computer.
d) Fire hazard: It can happen because of electrical short circuits, flammable liquids etc.
e) Sabotage and riots: Sometimes the employees destroy the computer centre in case of
strike, lockout or there may be chances of riots in the area.
f)Natural Disasters: Natural disasters are not controllable. They are not frequent hazards
but if they happen they destroy the things or ruin them. Examples are earthquake,
floods, tornadoes and lightening.
g) General hazards: This category covers many more hazards which are not covered
anywhere are difficult to define and come spontaneously.

Q6. Case Study: Information system in a restaurant.

Chandan Kumar Sah Page 100 of 152 MBA 2ND Semester


Ans: Case Summary:

A waiter takes an order at a table, and then enters it online via one of the six terminals
located in the restaurant dining room. The order is routed to a printer in the appropriate
preparation area: the cold item printer if it is a salad, the hot-item printer if it is a hot
sandwich or the bar printer if it is a drink. A customer’s meal check-listing (bill) the items
ordered and the respective prices are automatically generated. This ordering system
eliminates the old three-carbon-copy guest check system as well as any problems caused by
a waiter’s handwriting. When the kitchen runs out of a food item, the cooks send out an
‘out of stock’ message, which will be displayed on the dining room terminals when waiters
try to order that item. This gives the waiters faster feedback, enabling them to give better
service to the customers. Other system features aid management in the planning and control
of their restaurant business. The system provides up-to-the-minute information on the food
items ordered and breaks out percentages showing sales of each item versus total sales.
This helps management plan menus according to customers’ tastes. The system also
compares the weekly sales totals versus food costs, allowing planning for tighter cost
controls. In addition, whenever an order is voided, the reasons for the void are keyed in.
This may help later in management decisions, especially if the voids consistently related to
food or service. Acceptance of the system by the users is exceptionally high since the
waiters and waitresses were involved in the selection and design process. All potential
users were asked to give their impressions and ideas about the various systems available
before one was chosen.
Questions to be analysed:
1. In the light of the system, describe the decisions to be made in the area of strategic
planning, managerial control and operational control? What information would you require
to make such decisions?
2. What would make the system a more complete MIS rather than just doing transaction
processing?
3. Explain the probable effects that making the system more formal would have on the
customers and the management.
Solution:
1. A management information system (MIS) is an organized combination of people,
hardware, communication networks and data sources that collects, transforms and

Chandan Kumar Sah Page 101 of 152 MBA 2ND Semester


distributes information in an organization. An MIS helps decision making by providing
timely, relevant and accurate information to managers. The physical components of an MIS
include hardware, software, database, personnel and procedures. Management information
is an important input for efficient performance of various managerial functions at different
organization levels. The information system facilitates decision making. Management
functions include planning, controlling and decision making. Decision making is the core
of management and aims at selecting the best alternative to achieve an objective. The
decisions may be strategic, tactical or technical. Strategic decisions are characterized by
uncertainty. They are future oriented and relate directly to planning activity. Tactical
decisions cover both planning and controlling. Technical decisions pertain to
implementation of specific tasks through appropriate technology. Sales region analysis,
cost analysis, annual budgeting, and relocation analysis are examples of decision-support
systems and management information systems. There are 3 areas in the organization. They
are strategic, managerial and operational control. Strategic decisions are characterized by
uncertainty. The decisions to be made in the area of strategic planning are future oriented
and relate directly to planning activity. Here basically planning for future that is budgets,
target markets, policies, objectives etc. is done. This is basically a top level where up-to-the
minute information on the food items ordered and breaks out percentages showing sales of
each item versus total sales is provided. The top level where strategic planning is done
compares the weekly sales totals versus food costs, allowing planning for tighter cost
controls. Executive support systems function at the strategic level, support unstructured
decision making, and use advanced graphics and communications. Examples of executive
support systems include sales trend forecasting, budget forecasting, operating plan
development, budget forecasting, profit planning, and manpower planning. The decisions
to be made in the area of managerial control are largely dependent upon the information
available to the decision makers. It is basically a middle level where planning of menus is
done and whenever an order is voided, the reasons for the void are keyed in which later
helps in management decisions, especially if the voids are related to food or service. The
managerial control that is middle level also gets customer feedback and is responsible for
customer satisfaction. The decisions to be made in the area of operational control pertain to
implementation of specific tasks through appropriate technology. This is basically a lower
level where the waiter takes the order and enters it online via one of the six terminals

Chandan Kumar Sah Page 102 of 152 MBA 2ND Semester


located in the restaurant dining room and the order is routed to a printer in the appropriate
preparation area. The item’s ordered list and the respective prices are automatically
generated. The cooks send ‘out of stock’ message when the kitchen runs out of a food item,
which is basically displayed on the dining room terminals when waiter tries to order that
item. This basically gives the waiters faster feedback, enabling them to give better service
to the customers. Transaction processing systems function at the operational level of the
organization. Examples of transaction processing systems include order tracking, order
processing, machine control, plant scheduling, compensation, and securities trading. The
information required to make such decision must be such that it highlights the trouble spots
and shows the interconnections with the other functions. It must summarize all information
relating to the span of control of the manager. The information required to make these
decisions can be strategic, tactical or operational information.

Advantages of an online computer system:


1. Eliminates carbon copies
2. Waiters’ handwriting issues
3. Out-of-stock message
4. Faster feedback helps waiters to service the customers

Advantages to management:
1. Sales figures and percentages item-wise
2. Helps in planning the menu
3. Cost accounting details

2. If the management provides sufficient incentive for efficiency and results to their
customers, it would make the system a more complete MIS and so the MIS should support
this culture by providing such information which will aid the promotion of efficiency in the
management services and operational system. It is also necessary to study the keys to
successful Executive Information System (EIS) development and operation. Decision
support systems would also make the system a complete MIS as it constitutes a class of
computer-based information systems including knowledge-based systems that support
decision-making activities. DSSs serve the management level of the organization and help

Chandan Kumar Sah Page 103 of 152 MBA 2ND Semester


to take decisions, which may be rapidly changing and not easily specified in advance.
Improving personal efficiency, expediting problem solving (speed up the progress of
problems solving in an organization), facilitating interpersonal communication, promoting
learning and training, increasing organizational control, generating new evidence in support
of a decision, creating a competitive advantage over competition, encouraging exploration
and discovery on the part of the decision maker, revealing new approaches to thinking
about the problem space and helping automate the managerial processes would make the
system a complete MIS rather than just doing transaction processing.

3. The management system should be an open system and MIS should be so designed that
it highlights the critical business, operational, technological and environmental changes to
the concerned level in the management, so that the action can be taken to correct the
situation. To make the system a success, knowledge will have to be formalized so that
machines worldwide have a shared and common understanding of the information
provided. The systems developed will have to be able to handle enormous amounts of
information very fast. An organization operates in an ever-increasing competitive, global
environment. Operating in a global environment requires an organization to focus on the
efficient execution of its processes, customer service, and speed to market. To accomplish
these goals, the organization must exchange valuable information across different
functions, levels, and business units. By making the system more formal, the organization
can more efficiently exchange information among its functional areas, business units,
suppliers, and customers. As the transactions are taking place every day, the system stores
all the data which can be used later on when the hotel is in need of some financial help
from financial institutes or banks. As the inventory is always entered into the system, any
frauds can be easily taken care of and if anything goes missing then it can be detected
through the system.

Chandan Kumar Sah Page 104 of 152 MBA 2ND Semester


Chandan Kumar Sah Page 105 of 152 MBA 2ND Semester
Assignment Set – 1

Q1. a. “Operation Techniques is a bunch of mathematical techniques.” Comment.


b. “Operation Research is an aid for the executive in making his decisions based on
scientific methods analysis”. Discuss the above statement in brief.
Ans: a) Operations Research is an interdisciplinary branch of applied mathematics and formal
science that uses methods such as mathematical modeling, statistics, and algorithms to
arrive at optimal or near optimal solutions to complex problems. It is typically concerned
with optimizing the maxima (profit, assembly line performance, crop yield, bandwidth, etc)
or minima (loss, risk, etc.) of some objective function. Operations research helps
management achieve its goals using scientific methods. The terms operations research and
management science are often used synonymously. When a distinction is drawn,
management science generally implies a closer relationship to the problems of business
management. The field of operations research is closely related to Industrial engineering.
Industrial engineers typically consider Operations Research (OR) techniques to be a major
part of their toolset. Some of the primary tools used by operations researchers are statistics,
optimization, probability theory, queuing theory, game theory, graph theory, decision
analysis, and simulation. Because of the computational nature of these fields, OR also has
ties to computer science, and operations researchers use custom-written and off-the-shelf
software. Operations research is distinguished by its frequent use to examine an entire
management information system, rather than concentrating only on specific elements
(though this is often done as well). An operations researcher faced with a new problem is
expected to determine which techniques are most appropriate given the nature of the
system, the goals for improvement, and constraints on time and computing power. For this
and other reasons, the human element of OR is vital. Like any other tools, OR techniques
cannot solve problems by themselves. Scope of operation Research. Examples of
applications in which operations research is currently used include:
• Critical path analysis or project planning: identifying those processes in a complex project
which affect the overall duration of the project
• Designing the layout of a factory for efficient flow of materials

Chandan Kumar Sah Page 106 of 152 MBA 2ND Semester


• Constructing a telecommunications network at low cost while still guaranteeing QoS
(quality of service) or QoS (Quality of Experience) if particular connections become very
busy or get damaged
• Road traffic management and 'one way' street allocations i.e. allocation problems.
• Determining the routes of school buses (or city buses) so that as few buses are needed as
possible
• designing the layout of a computer chip to reduce manufacturing time (therefore reducing
cost) Managing the flow of raw materials and products in a supply chain based on uncertain
demand for the finished products
• Efficient messaging and customer response tactics
• Robotizing or automating human-driven operations processes
• Globalizing operations processes in order to take advantage of cheaper materials, labor,
land or other productivity inputs Managing freight transportation and delivery systems
(Examples: LTL Shipping, intermodal freight transport)
• Scheduling:
○ Personnel staffing
○ Manufacturing steps
○ Project tasks
○ Network data traffic: these are known as queuing models or queuing systems.
○ sports events and their television coverage blending of raw materials in oil
refineries determining optimal prices, in many retail and B2B settings, within the
disciplines of pricing science. Operations research is also used extensively in government
where evidence-based policy is used.

b) Operation Research is a scientific method of providing executive departments with a


quantitative basis for decisions regarding the operations under their control. – Morse &
Kimball Operations research is a scientific approach to problem solving for executive
management. – H.M. Wagner Operations research is an aid for the executive in making
these decisions by providing him with the needed quantitative information based on the
scientific method of analysis. The mission of Operations Research is to serve the entire
Operations Research (OR) community, including practitioners, researchers, educators, and
students. Operations Research, as the flagship journal of our profession, strives to publish

Chandan Kumar Sah Page 107 of 152 MBA 2ND Semester


results that are truly insightful. Each issue of Operations Research attempts to provide a
balance of well-written articles that span the wide array of creative activities in OR. Thus,
the major criteria for acceptance of a paper in Operations Research are that the paper is
important to more than a small subset of the OR community, contains important insights,
and makes a substantial contribution to the field that will stand the test of time. Operational
research, also known as operations research, is an interdisciplinary branch of applied
mathematics and formal science that uses advanced analytical methods such as
mathematical modeling, statistical analysis, and mathematical optimization to arrive at
optimal or near-optimal solutions to complex decision-making problems. It is often
concerned with determining the maximum (of profit, performance, or yield) or minimum
(of loss, risk, or cost) of some real-world objective. Originating in military efforts before
World War II, its techniques have grown to concern problems in a variety of industries.
Operational research, also known as operations research, is an interdisciplinary branch of
applied mathematics and formal science that uses advanced analytical methods such as
mathematical modeling, statistical analysis, and mathematical optimization to arrive at
optimal or near-optimal solutions to complex decision-making problems. It is often
concerned with determining the maximum (of profit, performance, or yield) or minimum
(of loss, risk, or cost) of some real world objective. Originating in military efforts before
World War II, its techniques have grown to concern problems in a variety of industries.
Operational research encompasses a wide range of problem-solving techniques and
methods applied in the pursuit of improved decision-making and efficiency. Some of the
tools used by operational researchers are statistics, optimization, probability theory,
queuing theory, game theory, graph theory, decision analysis, mathematical modeling and
simulation. Because of the computational nature of these fields, OR also has strong ties to
computer science. Operational researchers faced with a new problem must determine which
of these techniques are most appropriate given the nature of the system, the goals for
improvement, and constraints on time and computing power. Work in operational research
and management science may be characterized as one of three categories: Fundamental or
foundational work takes place in three mathematical disciplines: probability, optimization,
and dynamical systems theory. Modeling work is concerned with the construction of
models, analyzing them mathematically, implementing them on computers, solving them
using software tools, and assessing their effectiveness with data. This level is mainly

Chandan Kumar Sah Page 108 of 152 MBA 2ND Semester


instrumental, and driven mainly by statistics and econometrics. Application work in
operational research, like other engineering and economics' disciplines, attempts to use
models to make a practical impact on real-world problems.

• The major sub disciplines in modern operational research, as identified by the journal
Operations Research, are:

• Computing and information technologies

• Decision analysis

• Environment, energy, and natural resources

• Financial engineering

• Manufacturing, service sciences, and supply chain management

• Policy modeling and public sector work

• Revenue management

• Simulation

• Stochastic models

• Transportation

Q2. Comment on the following statements:


a. Operation Research advocates a system approach and is concerned with
optimization.
b. Operation Research replaces management by personality.
Ans: a) Operation Research advocates a system approach and is concerned with optimization.
Operations research (O.R.) is the discipline of applying advanced analytical methods to
help make better decisions. By using techniques such as mathematical modeling to analyze
complex situations, operations research gives executives the power to make more effective
decisions and build more productive systems based on:
• More complete data

Chandan Kumar Sah Page 109 of 152 MBA 2ND Semester


• Consideration of all available options
• Careful predictions of outcomes and estimates of risk
• The latest decision tools and techniques
O.R. is unique. It's best of breed, employing highly developed methods practiced by
specially trained professionals. It’s powerful, using advanced tools and technologies to
provide analytical power that no ordinary software or spreadsheet can deliver out of the
box. And it’s tailored to you, because an O.R. professional offers you the ability to define
your specific challenge in ways that make the most of your data and uncover your most
beneficial options. To achieve these results, O.R. professionals draw upon the latest
analytical technologies, including:
• Simulation Giving you the ability to try out approaches and test ideas for improvement
• Optimization Narrowing your choices to the very best when there are virtually
innumerable feasible options and comparing them is difficult
• Probability and Statistics Helping you measure risk, mine data to find valuable
connections and insights, test conclusions, and make reliable forecasts
• O.R. has enhanced organizations and experiences all around us. From better scheduling of
airline crews to the design of waiting lines at Disney theme parks.

b) Operations research (OR) and management science are terms that are used
interchangeably to describe the discipline of applying quantitative techniques to make
decisions and solve problems. Many methods used in operations research were developed
during World War II to help take the guesswork out of missions such as deploying radar,
searching for enemy submarines, and getting supplies where they were most needed. The
prevalence of operations research in the Nation’s economy reflects the growing complexity
of managing large organizations that require the efficient use of materials, equipment, and
people. OR analysts determine the optimal means of coordinating these elements to achieve
specified goals by applying mathematical principles to organizational problems. They solve
problems in different ways and propose alternative solutions to management, which then
chooses the course of action that best meets their goals. In general, OR analysts are
concerned with issues such as strategy, forecasting, resource allocation, facilities layout,
inventory control, personnel schedules, and distribution systems. The duties of the
operations research analyst vary according to the structure and management philosophy of

Chandan Kumar Sah Page 110 of 152 MBA 2ND Semester


the employer or client. Some firms centralize operations research in one department; others
use operations research in each division. Some organizations contract operations research
services with a consulting firm. Economists, systems analysts, mathematicians, industrial
engineers, and others may apply operations research techniques to address problems in
their respective fields. Operations research analysts may also work closely with senior
managers to identify and solve a variety of problems. Regardless of the type or structure of
the client organization, operations research in its classical role of carrying out analysis to
support management’s quest for performance improvement entails a similar set of
procedures. Managers begin the process by describing the symptoms of a problem to the
analyst, who then formally defines the problem. For example, an operations research
analyst for an auto manufacturer may be asked to determine the best inventory level for
each of the parts needed on a production line and to determine the number of windshields
to be kept in inventory. Too many windshields would be wasteful and expensive, while too
few could result in an unintended halt in production. Operations research analysts study
such problems, then break them into their component parts. Analysts then gather
information about each of these parts from a variety of sources. To determine the most
efficient amount of inventory to be kept on hand, for example, OR analysts might talk with
engineers about production levels, discuss purchasing arrangements with buyers, and
examine data on storage costs provided by the accounting department. The procedures of
operations research were first formalized by the military. They have been used in wartime
to effectively deploy radar, search for enemy submarines, and get supplies to where they
are most needed. In peacetime and in private enterprises, operations research is used in
planning business ventures and analyzing options by using statistical analysis, data and
computer modeling, linear programming, and other mathematical techniques. Large
organizations are very complex. They must effectively manage money, materials,
equipment, and people. Operations research analysts find better ways to coordinate these
elements by applying analytical methods from mathematics, science, and engineering.
Analysts often find many possible solutions for meeting the goals of a project. These
potential solutions are presented to managers, who choose the course of action that they
think best. Operations research analysts are often involved in top-level strategizing,
planning, and forecasting. They help to allocate resources, measure performance, schedule,
design production facilities and systems, manage the supply chain, set prices, coordinate

Chandan Kumar Sah Page 111 of 152 MBA 2ND Semester


transportation and distribution, or analyze large databases. The duties of the operations
research analyst vary according to the structure and management of the organization they
are assisting. Some firms centralize operations research in one department; others use
operations research in each division. Operations research analysts also may work closely
with senior managers to identify and solve a variety of problems. Analysts often have one
area of specialization, such as working in the transportation or the financial services
industry. Operations research analysts start a project by listening to managers describe a
problem. Then, analysts ask questions and formally define the problem. For example, an
operations research analyst for an auto manufacturer may be asked to determine the best
inventory level for each of the parts needed on a production line and to ascertain the
optimal number of windshields to be kept in stock. Too many windshields would be
wasteful and expensive, whereas too few could halt production. Analysts would study the
problem, breaking it into its components. Then they would gather information from a
variety of sources. To determine the optimal inventory, operations research analysts might
talk with engineers about production levels, discuss purchasing arrangements with buyers,
and examine storage-cost data provided by the accounting department. Relevant
information in hand, the analysts determine the most appropriate analytical technique.
Techniques used may include a Monte Carlo simulation, linear and nonlinear
programming, dynamic programming, queuing and other stochastic-process models,
Markov decision processes, econometric methods, data envelopment analysis, neural
networks, expert systems, decision analysis, and the analytic hierarchy process. Nearly all
of these techniques involve the construction of a mathematical model that attempts to
describe the system being studied. So, the problem of the windshields, for example, would
be described as a set of equations that try to model real-world conditions. The use of
models enables the analyst to explicitly describe the different components and clarify the
relationships among them. The descriptions can be altered to examine what may happen to
the system under different circumstances. In most cases, a computer program is developed
to numerically evaluate the model. Usually the model chosen is modified and run
repeatedly to obtain different solutions. A model for airline flight scheduling, for example,
might stipulate such things as connecting cities, the amount of fuel required to fly the
routes, projected levels of passenger demand, varying ticket and fuel prices, pilot
scheduling, and maintenance costs. By assessing different possible schedules, the analyst is

Chandan Kumar Sah Page 112 of 152 MBA 2ND Semester


able to determine the best flight schedule consistent with particular assumptions. Based on
the results of the analysis, the operations research analyst presents recommendations to
managers. The analyst may need to modify and rerun the computer program to consider
different assumptions before presenting the final recommendation. Once managers reach a
decision, the analyst usually works with others in the organization to ensure the plan’s
successful implementation.

Q3. Explain how the profit maximization transportation problem can be converted to an
equivalent cost minimization transportation problem.
Ans: How to convert profit maximization transportation problem to an equivalent cost
minimization transportation problem can be understood by following Illustration as:
A firm has three factories located in city A, B & C and supplies goods to four dealers,
dealer 1, 2, 3 & 4, spread all over the country. The production capacities of these factories
are 1000, 700 & 900 units per month respectively. The monthly orders from the dealers are
900, 800, 500 & 400 units respectively. Per unit return (excluding transportation costs) are
Rs. 8, 7 & 9 at the three factories. Unit transportation costs from the dealers are given
below:

Factory Dealers
1 2 3 4
City - A 2 2 2 4
City - B 3 5 3 2
City - C 4 3 2 1

Optimal distribution system to maximize the total return to be determined.

From the given data, we compute a matrix of net returns as done in table below;

Chandan Kumar Sah Page 113 of 152 MBA 2ND Semester


(Transportation matrix (Net return) for the Maximization problem)

Factory Dealers Factory


1 2 3 4 capacity
City - A 6 6 6 4 1000
City - B 4 2 4 5 700
City - C 5 6 7 8 900

To convert the given maximization problem to an equivalent minimization problem, we


identify the cell (element) which has the highest contribution per unit (in this problem C-4
has highest per unit contribution, Rs.8), and subtract all elements from this highest element.
The resultant matrix is a transportation problem with minimizing objective function. This
has been given in the following table.

(Transportation matrix for the Minimization problem)

Factory Dealers Factory


1 2 3 4 capacity
City - A 2 2 2 4 1000
City - B 4 6 4 3 700
City - C 3 2 1 0 900
Dealer 900 80 50 400 2600
requirement 0 0

The minimization problem is solved as a usual transportation problem. The resulting


optimal solution is also the optimal solution to the original (maximization) problem. The
value of the objective function is computed by referring the matrix of the maximization
problem. It should be noted that the converted minimization problem will have at least one
element with zero value.

Q4. Write the difference in the simplex solution procedure for a maximization problem
and a minimization problem of linear programming.

Chandan Kumar Sah Page 114 of 152 MBA 2ND Semester


Ans: The difference in the simplex solution procedure for a maximization problem and a
minimization problem of linear programming can be explained by the steps followed to
solve the minimization/ minimization problem as follows;
1. Introduce stack variables (Si’s) for “£” type of constraint.
2. Introduce surplus variables (Si’s) and artificial variables (Ai) for “³” type of constraint.
3. Introduce only Artificial variable for “=” type of constraint.
4. Cost (Cj) of slack and surplus variables will be zero and that of artificial variable will be
“M”
5. Find Zj – Cj for each variable.
6. Slack and artificial variables will form basic variable for the first simplex table. Surplus
variable will never become basic variable for the first simplex table.
7. Zj = sum of [cost of variable x its coefficients in the constraints – Profit or cost
coefficient of the variable].
8. Select the most negative value of Zj – Cj. That column is called key column. The
variable corresponding to the column will become basic variable for the next table.
9. Divide the quantities by the corresponding values of the key column to get ratios; select
the minimum ratio. This becomes the key row. The basic variable corresponding to this
row will be replaced by the variable found in step 6.
10. The element that lies both on key column and key row is called Pivotal element.
11. Ratios with negative and “a” value are not considered for determining key row.
12. Once an artificial variable is removed as basic variable, its column will be deleted from
next iteration.
13. For maximization problems, decision variables coefficient will be same as in the
objective function. For minimization problems, decision variables coefficients will have
opposite signs as compared to objective function.
14. Values of artificial variables will always is – M for both maximization and
minimization problems.
15. The process is continued till all Zj – Cj ³ 0.

Q5. What do you mean by the two-phase method for solving a given LPP? Why is it used?

Chandan Kumar Sah Page 115 of 152 MBA 2ND Semester


Ans: The drawback of the penalty cost method is the possible computational error resulting from
assigning a very large value to the constant M. To overcome this difficulty, a new method
is considered, where the use of M is eliminated by solving the problem in two phases.
Phase I: Formulate the new problem. Start by eliminating the original objective function
by the sum of the artificial variables for a minimization problem and the negative of the
sum of the artificial variables for a maximization problem. The Simplex method optimizes
the ensuing objective with the constraints of the original problem. If a feasible solution is
arrived the optimal value of the new objective function is zero. Subsequently proceed to
phase II. If the optimal value of the new objective function is non-zero, it means there is no
solution to the problem and the method terminates.
Phase II: Start phase II using the optimum solution of phase I as the base. Then take the
objective function without the artificial variables and solve the problem using the Simplex
method.

Q6. Indicate any four shortcomings of taking a simulation approach to solve an O.R.
problem.

Ans: Shortcomings of taking a simulation approach to solve an O.R. problem


The range of application of simulation in business is extremely wide. Unlike other
mathematical models, simulation can be easily understood by the users and thereby
facilitates their active involvement. This makes the results more reliable and also ensures
easy acceptance for implementation. The degree to which a simulation model can be made
close to reality is dependent upon the ingenuity of the OR team who identifies the relevant
variables as well as their behavior. In case of other OR models, simulation helps the
manager to strike a balance between opposing costs of providing facilities (usually
meaning long term commitment of funds) and the opportunity and costs of not providing
them. The simulation approach is recognized as a powerful tool for management decision-
making. Shortcoming of taking a simulation approach to solve an O. R. problems are as
follows;
1. It does not produce optimal results. Solutions are approximate, and it is some less than
formal but ‘satisfactory’ approach to problem-solving only.

Chandan Kumar Sah Page 116 of 152 MBA 2ND Semester


2. To be able to simulate systems, a fairly good knowledge of the parts or components of
the system and their characteristics is required. The desire is to understand, explain and
predict the dynamic behavior of the system or the sum total of these parts. Adequate
knowledge of the system behavior.
3. Each simulation run like a single experiment conducted under a given set of conditions
as defined by a set of values for the input solution. A number of simulation runs will be
necessary and thus can be time consuming. As the number of variables increases in
terms of input, the difficulty in finding the optimum values increases considerably.
4. Since simulation involves repetitions of the experiment, it is a time consuming task
when manually done.
5. As a number of parameters, increase, the difficulty in finding the optimum values
increases to a considerable extent.
6. Because of the simplicity in adoption of simulation process, one may develop to rely on
this technique too often, although mathematical model is more suitable to the situation.
7. One should not ignore the cost associated with a simulation study for data collection,
formation of the model. A good simulation model may be very expensive. Often it
takes years to develop a usable corporate planning model.
8. The computer time as it is fairly significant.
9. A simulation application is based on the premise that the behaviour pattern of relevant
variables is known, and this very premise sometimes becomes questionable.
10. Not always can the probabilities be estimated with ease or desired reliability. The
results of simulation should always be compared with solutions obtained by other
methods wherever possible, and “tempered” with managerial judgment

Chandan Kumar Sah Page 117 of 152 MBA 2ND Semester


Assignment Set – 2

Q1. Explain how to transform an unbalanced transportation problem into a balanced


transportation problem where the demand of warehouses is satisfied by the supply of
factories.
Ans: If the total supply is more than the total demand, we introduce an additional column, which
will indicate the surplus supply with transportation cost zero. Similarly, if the total demand
is more than the total supply, an additional row is introduced in the table, which represents
unsatisfied demand with transportation cost zero. The balancing of an unbalanced
transportation problem is illustrated in the following example.

Example

Warehouse
Plant Supply
W1 W2 W3
A 28 17 26 500
B 19 12 16 300
Demand 250 250 500

Solution:

The total demand is 1000, whereas the total supply is 800.

Si < Dj
Total supply < total demand.
To solve the problem, we introduce an additional row with transportation cost zero
indicating the unsatisfied demand.

Warehouse
Plant Supply
W1 W2 W3
A 28 17 26 500
B 19 12 16 300
Unsatisfied
0 0 0 200
demand
Demand 250 250 500 1000

Chandan Kumar Sah Page 118 of 152 MBA 2ND Semester


Using matrix minimum method, we get the following allocations.

Warehouse
Plant Supply
W1 W2 W3
A 17 500

B 19 300
Unsatisfied
0 0 200
demand
Demand 250 250 500 1000

Initial basic feasible solution

50 X 28 + 450 X 26 + 250 X 12 + 50 X 16 + 200 X 0 = 16900.

Q2. Explain how the profit maximization transportation problem into a balanced
transportation problem where the demand of warehoused is satisfied by the supply of
factories.
Ans: A fictive corporation A has a contract to supply motors for all tractors produced by a fictive
corporation B. Corporation B manufactures the tractors at four locations around Central
Europe: Prague, Warsaw, Budapest and Vienna. Plans call for the following numbers of
tractors to be produced at each location:
Prague 9 000
Warsaw 12 000
Budapest 9 000
Corporation A has three plants that can produce the motors. The plants and production
capacities are
Hamburg 8 000
Munich 7 000
Leipzig 10 000
Dresden 5 000
Due to varying production and transportation costs, the profit earns on each motor depends
on where they were produced and where they were shipped. The following transportation

Chandan Kumar Sah Page 119 of 152 MBA 2ND Semester


table (Table 7.9) gives the accounting department estimates of the euro profit per unit
(motor).

"Highest - Profit Assignment"


Applying the Stepping Stone method (modified for maximization purposes) to the initial
solution we can see that no other transportation schedule can increase the profit and so the
Highest – Profit initial allocation is also an optimal solution of this transportation problem.
Stepping Stone Method
Step 1: Pick any empty cell and identify the closed path leading to that cell. A closed path
consists of horizontal and vertical lines leading from an empty cell back to itself (If
assignments have been made correctly, the matrix has only one closed path for each empty
cell.) In the closed path there can only be one empty cell that we are examining. The 90-
degree turns must therefore occur at those places that meet this requirement. Two closed
paths are identified. Closed path a is required to evaluate empty cell A-E; closed path b is
required to evaluate empty cell A-F
Step 2: Move one unit into the empty cell from a filled cell at a corner of the closed path
and modify the remaining filled cells at the other comers of the closed path to reflect this
move. (More than one unit could be used to test the desirability of a shift. However, since
the problem is linear, if it is desirable to shift one unit, it is desirable to shift more than one,
and vice versa.) Modifying entails adding to and subtracting from filled cells in such a way
that supply and demand constraints are not violated. This requires that one unit always be
subtracted in a given row or column for each unit added to that row or column. Thus, the
following additions and subtractions would be required:
For the path a:

Chandan Kumar Sah Page 120 of 152 MBA 2ND Semester


• Add one unit to A-E (the empty cell).
• Subtract one unit from A-H.
• Add one unit to C-H.
• Subtract one unit from C-E.
For the path b,
• Add one unit to A-F (the empty cell).
• Subtract one unit from A-H.
• Add one unit to C-H.
• Subtract one unit from C-G.
• Add one unit to D-G.
• Subtract one unit from D-F.
Step 3: Determine desirability of the move. This is easily done by (1) summing the cost
values for the cell to which a unit has been added, (2) summing the cost values of the cells
from which a unit has been subtracted, and (3) taking the difference between the two sums
to determine if there is a cost reduction. If the cost is reduced by making the move, as many
units as possible should be shifted out of the evaluated filled cells into the empty cell. If the
cost is increased, no move should be made and the empty cell should be crossed.

Q3. Illustrate graphically the following special cases of Linear programming problems:
i) Multiple optimal solutions, ii) No feasible solution, iii) Unbounded problem
Ans: Solving a LPP with 2 decision variables x1 and x2 through graphical representation is easy.
Consider x1 x2 – the plane, where you plot the solution space enclosed by the constraints.
The solution space is a convex set bounded by a polygon; since a linear function attains
extreme (maximum or minimum) values only on the boundary of the region. You can
consider the vertices of the polygon and find the value of the objective function in these
vertices. Compare the vertices of the objective function at these vertices to obtain the
optimal solution of the problem.

Working rule
The method of solving a LPP on the basis of the above analysis is known as the graphical
method. The working rule for the method is as follows.

Chandan Kumar Sah Page 121 of 152 MBA 2ND Semester


Step 1: Write down the equations by replacing the inequality symbols by the equality
symbols in the given constraints.
Step 2: Plot the straight lines represented by the equations obtained in step I.
Step 3: Identify the convex polygon region relevant to the problem. Decide on which side
of the line, the half-plane is located.
Step 4: Determine the vertices of the polygon and find the values of the given objective
function Z at each of these vertices. Identify the greatest and least of these values. These
are respectively the maximum and minimum value of Z.
Step 5: Identify the values of (x1, x2) which correspond to the desired extreme value of Z.
This is an optimal solution of the problem
We can analyse linear programming with 2 decision variables graphically.
Example
Let’s look at the following illustration.
Maximise Z = 700 x1+500 x2
Subject to 4x1+3x2  210
2x1+x2  90
and x1  0, x2  0
Let the horizontal axis represent x1 and the vertical axis x2. First, draw the line 4x1 + 3x2
= 210, (by replacing the inequality symbols by the equality) which meets the x1-axis at the
point A (52.50, 0) (put x2 = 0 and solve for x1 in 4x1 + 3x2 = 210) and the x2 – axis at the
point B (0, 70) (put x1 = 0 in 4x1 + 3x2 = 210 and solve for x2).

Suppose a linear programming problem has an unbounded feasible solution space. If the
set of all values of the objective function at different feasible solutions is not bounded
above (respectively, bounded below), and if the problem is a maximisation (respectively,

Chandan Kumar Sah Page 122 of 152 MBA 2ND Semester


minimisation) problem, then we say that the given problem has an unbounded solution.

Q4. What would you deal with the Assignment problems, where a) the objective function
is to be maximized? b) Some Assignments are prohibited?
Ans: Let’s say there are “n” jobs in a factory having “n” machines to process the jobs. A job i
(=1… n), when processed by machine j (=1… n) is assumed to incur a cost Cij. The
assignment is to be made in such a way that each job can associate with one and only one
machine. You can then determine an assignment of jobs to the machines to minimise the
overall cost. The cost data is given as a matrix where rows correspond to jobs and columns
to machines and there are as many rows as the number of columns. The number of jobs and
number of machines should be equal. Assignment becomes a problem because each job
requires different skills and the capacity or efficiency of each person with respect to these
jobs can be different. This gives rise to cost differences. If each person is able to do all jobs
with same efficiency then all costs will be the same and each job can be assigned to any
person. When assignment is a problem it becomes a typical optimization problem.
Therefore, you can compare an assignment problem to a transportation problem. The cost
element is given and is a square matrix and requirement at each destination is one and
availability at each origin is also one. Additionally, you have number of origins, Which
equal the number of destinations. Therefore, the total demand is equal to the total supply.
There is only one assignment in each row and each column. However if you compare this
to a transportation problem, you will find that a general transportation problem does not
have the above mentioned limitations. These limitations are peculiar to assignment
problems only. An assignment problem can be either balanced or unbalanced. Let’s first
focus on a balanced assignment problem. A balanced assignment problem is one where the
number of rows = the number of columns (comparable to a balanced transportation
problem where total demand =total supply).

Balanced assignment problem: No. of rows = No. of columns

Chandan Kumar Sah Page 123 of 152 MBA 2ND Semester


A:- the objective function is to be maximized :-
Some assignment problems are phrased in terms of maximizing the profit or effectiveness
or payoff of an assignment of people to tasks or of jobs to machines. You cannot apply the
Hungarian method to such maximization problems. Therefore, you need to reduce it to a
minimization problem. It is easy to obtain an equivalent minimization problem by

Chandan Kumar Sah Page 124 of 152 MBA 2ND Semester


converting every number in the table to an opportunity loss. To do so, you need to subtract
every value from the highest value of the matrix and then proceed as usual. You will notice
that minimizing the opportunity loss produces the same assignment solution as the original
maximization problem.

B:- b Some Assignments are prohibited


Infeasible assignments
It is sometimes possible that a particular person is incapable of doing certain work or a
specific a specific job cannot be performed on a particular machine. The solution of the
assignment problem should take into account these restrictions so that the infeasible
assignment can be avoided. This can be achieved by assigning a very high cost (say ∞ or
M) to the cells where assignments are prohibited, thereby, restricting the entry of this pair
of job – machine or resource – activity into the final solution. After inserting a high value
 at the cell we need to apply Hungarian method to solve the problem.

Chandan Kumar Sah Page 125 of 152 MBA 2ND Semester


Q5. “Simulation is an especially valuable tool in a situation where the mathematics needed
to describe a system realistically is too complex to yield analytical solutions”.
Elucidate.
Ans: Simulation is an especially valuable tool in a situation where the mathematics needed to
describe a system realistically is too complex to yield analytical solutions”. Elucidate.
Simulation is the imitation of some real thing, state of affairs, or process. The act of
simulating something generally entails representing certain key characteristics or behaviors
of a selected physical or abstract system. Simulation is used in many contexts, such as
simulation of technology for performance optimization, safety engineering, testing,
training, education, and video games. Training simulators include flight simulators for

Chandan Kumar Sah Page 126 of 152 MBA 2ND Semester


training aircraft pilots. Simulation is also used for scientific modeling of natural systems or
human systems in order to gain insight into their functioning. Simulation can be used to
show the eventual real effects of alternative conditions and courses of action. Simulation is
also used when the real system cannot be engaged, because it may not be accessible, or it
may be dangerous or unacceptable to engage, or it is being designed but not yet built, or it
may simply not exist. Simulation can be used when the problem is too complex for
analytical solution and too dangerous for actual experimentation. Key issues in simulation
include acquisition of valid source information about the relevant selection of key
characteristics and behaviors, the use of simplifying approximations and assumptions
within the simulation, and fidelity and validity of the simulation outcomes.

Q6. Describe Gomory’s method of solving an all-integer programming problem.

Ans: An optimum solution to an IPP is obtained by using the simplex method, ignoring the
restriction of integral values. In the optimum solution, if all the variables have integer
values, the current solution will be the required optimum integer solution. Otherwise, the
given IPP is modified by inserting a new constraint called Gomory’s constraint or
secondary constraint. This constraint represents necessary conditions for inerrability and
eliminates some non-integer solution without losing any integral solution. On addition of
the secondary constraint, the problem is solved using dual simplex method to obtain an
optimum integral solution. If all the values of the variables in the solution are integers, then
an optimum inter-solution is obtained, or else a new constraint is added to the modified
LPP and the procedure is repeated till the optimum solution is derived. An optimum integer
solution will be reached eventually after introducing enough new constraints to eliminate
all the superior non-integer solutions. The construction of additional constraints, called
secondary or Gomory’s constraints is important and needs special attention.
Construction of Gomory’s constraints
Consider a LPP or an optimum non–integer basic feasible solution. With the usual
notations, let the solution be displayed in the following simplex.

Chandan Kumar Sah Page 127 of 152 MBA 2ND Semester


The optimum basic feasible solution is given by
xB = [x2, x3 ] = [y10, y20]; max z = y00
Since xB is a non-integer solution, we can assume that y10 is fractional. The constraint
equation is
y10 = y11 x1 + y12 x2 + y13 x3+ y14 x4  0xx4 1  
It reduces to
y10 = y11 x1 + x2 + y14 x4 _____ (1) 0xx4 1  
Because x2 and x3 are basic variables (which implies that y12 = 1 and y13 = 0). The above
equation can be rewritten as
x2 = y10 - y11 x1 - y14 x4
This is a linear combination of non-basic variables. Now, since y10 0 the fractional part of
y10 must also be non-negative. You can split each of yij in (1) into an integral part Iij , and
a non-negative fractional part, f1j for j = 0,1,2,3,4. After the breakup (1) above, you can
write it as:
I10 + f10 = (I11 + f11) x2 + (I14 + f14) x4
Or
f10 - f11 x2 - f14 x4 = x2 + I11 x1 + I14x4 - I10 _
If you compare (1) and (2), you will see that if you add an additional constraint in such a
way that the left-hand side of (2) is an integer; then you will be forcing the non-integer y10
towards an integer.
The desired Gomory’s constraint is
f10 – f11 x1 – f11 x4 ≤ 0
It is possible to have f10 – f11 x1 – f11 x4 = h where h > 0 is an integer. Then f10 = h +
f11 x1 + f14 x4 is greater than one. This contradicts that 0 < fij < 1 for j = 0, 1, 2, 3, 4.
Thus Gomory’s constraint is
10 sla 4 ,1j j ij 4 , 1j 10 j ij 4 , 1j j ig f ) 1(G x f or f x . f or f x f 10 Where Gsla (1) is a
slack variable in the above first Gomory’s constraint

Chandan Kumar Sah Page 128 of 152 MBA 2ND Semester


The additional constraint to be included in the given LPP is towards obtaining an optimum
all integer solution. After adding the constraint, the optimum simplex table looks like given
below.

Since – f10 is negative, the optimal solution is unfeasible. Thus the dual simplex method is
to be applied for obtaining an optimum feasible solution. After obtaining this solution, the
above referred procedure is applied for constructing second Gomory’s constraint. The
process is to be continued till all the integer solution has been obtained.

Chandan Kumar Sah Page 129 of 152 MBA 2ND Semester


Chandan Kumar Sah Page 130 of 152 MBA 2ND Semester
Assignment Set – 1

Q1. Comment on the following


a. Importance of DMAIS in project management cycle
b. Knowledge areas of project management
Ans: a) DMAIS: Project management considers the five steps - DMAIS – as generic for any
system of a journey towards excellence. DMAIs is highly relevant in Project Management
for the simple reason that each step gives out in detail the actions to be taken to ensure
readiness for the next step. Verification of DMAIS implementation is possible with
checklists which can be prepared and used by employees at all levels. The five steps are: -
i) Define:- This step requires that what is sought to be achieved is identified in all its
detail. The following inputs which will define what we are going to make: a)
Benchmark: It refers to the standards achieved by the best in the industry. A
company’s product is set to meet them. b) Customer Requirement: It refers to the
documentation of customer requirements. Proper understanding of customer
requirement is of utmost importance. c) Process Flow Map: It shows the activities that
take place to result in the product at the end of them. d) Quality Function
Deployment: This tool compares the quality characteristics in a company’s product
with those in their competitors’ and their relative importance to the customer. e)
Project Management Plan: This includes the materials, men, activities, schedules,
milestones and so on.
ii) Measure:- In this step, we measure the outcome of the activities. This is done using the
following methods. a) Data collection: We need to collect the data about the work that
is done and compare as to how it corresponds with what is required. b) Defect Metrics:
We need to capture the deviations that are in the effective potion of the work in defect
metrics. Then we need to decide whether they are acceptable or need rectification. c)
Sampling: If the volumes are high, we need to select a few of them and inspect them to
see whether the entire batch is acceptable.
iii) Analyze:- In this step, we have to analyze the data received from the preceding step by
using the following tools: a) Cause and Effect Diagrams – also called Fishbone
Diagrams, b) Failure Mode and Effect Analysis FMEA, c) Root Cause Analysis, d)
Reliability Analysis

Chandan Kumar Sah Page 131 of 152 MBA 2ND Semester


iv) Improve:- In this step, we have to implement the measures to remove the defects found
earlier for improving the process. This can be done using the following measures. a)
Design of Experiments: The effect of changing values of parameters is done in a
controlled way. This allows us to experimentally determine the effect of variations
determined. We can use the results for optimizing the process. b) Robust Design: The
equipment design is made robust to reduce the variations. c) Tolerances: The permitted
deviations are made closer, so that the capability of process is increased.
v) Standardize:- When improvements have become consistent, the methods adopted are
standardized.

b) Knowledge areas of project management cycle: There are nine knowledge areas
consisting of :
1. Project Integration Management: Deals with processes that integrate different
aspects of project management.
2. Project Scope Management: Deals with processes that are responsible for
controlling project scope.
3. Project Time Management: Deals with Processes concerning the time constraints
of the project.
4. Project Cost Management: Deals with processes concerning the cost constraints of
the project.
5. Project Quality Management: Deals with processes that assure that the project
meets its quality obligations.
6. Project Human Resources Management: Deals with the processes related to
obtaining and managing the project team.
7. Project Communication Management: Deals with the processes concerning
communication mechanisms of a project.
8. Project Risk Management: Deals with the processes concerned with project risk
management.
9. Project Procurement Management: Deals with processes related to obtaining
products and services needed to complete a project.

Q2. Write few words on:

Chandan Kumar Sah Page 132 of 152 MBA 2ND Semester


a. Project Characteristics
b. WBS
c. PMIS
d. Project Management strategies - Internal & External
Ans: a) Project Characteristics: A project plan can be considered to have five key
characteristics that have to be managed:
• Scope: defines what will be covered in a project.
• Resource: what can be used to meet the scope.
• Time: what tasks are to be undertaken and when.
• Quality: the spread or deviation allowed from a desired standard.
• Risk: defines in advance what may happen to drive the plan off course, and what will be
done to recover the situation.

b) WBS: WBS stands for Work Breakdown Structure. The entire process of a project may
be considered to be made up on number of sub process placed in different stage called
Work Breakdown Structure. WBS is the technique to analyze the content of work and cost
by breaking it down into its component parts. Project key stages from the highest level of
the WBS, which is then used to show the details at the lower levels of the project. Each key
stage comprises many tasks identified at the start of planning and later this list will have to
be validated. WBS is produced by identifying the key elements, breaking each element
down into component parts and continuing to breakdown until manageable work packages
have been identified. These can then be allocated to the appropriate person. The WBS does
not show dependencies other than a grouping under the key stages. It is not time based –
there is no timescale on the drawing.

c) PMIS: PMIS stands for Project Management Information System. An information


system dealing with project management tasks is the project management information
system. It helps in decision making in arriving at optimum allocation of resources. The
information system is based on a database of the organization. A project management
information system also holds schedule, scope changes, risk assessment and actual results.
The information is communicated to managers at different levels of the organization
depending upon the need. The four major aspects of PMIS are: -

Chandan Kumar Sah Page 133 of 152 MBA 2ND Semester


1. Providing information to the major stakeholders

2. Assisting the team members, stakeholders, managers with necessary information and
summary of the information shared to the higher level managers

3. Assisting the managers in doing what of analyses about project staffing proposed
staffing changes and total allocation of resources

4. Helping organizational learning by helping the members of the organization learn


about project management

d) Project Management Strategies – Internal & external


Internal Project Management Strategies: Project fail for many internal reasons, some of
them technical, some of them managerial. However, even the technical failures can often
be traced back to a failure on the part of the project’s executive management to recognize
and deal with these inherent managerial risks. The project manager has a vital role to play
in achieving project success and should therefore insist on the following:
a) The executive must clearly demonstrate support for the project concept by the
active sponsorship and control.

b) The project manager and his team members must be competent. Other functional
personnel assigned to the project must also be competent.

c) The project manager should have a say in the assembly of his project team, which
will help to obtain their personal commitment, support and required quality of
service.

d) Effective project management information and control systems must be in place.

External Project Management Strategies: On some projects, events external to the


project sometimes come as a surprise to the project manager and his team and are therefore
seem as obstacles to progress. However, projects generally exist only because of that
external environment and so it is essential for the project team to recognize that they must
also be responsive to it. Clearly, the environment will not be the same for every project. If
fact, it is likely to be determined principally by three considerations namely:
a) The product or service resulting from the project

Chandan Kumar Sah Page 134 of 152 MBA 2ND Semester


b) The technology and the manner of its application, and

c) Its physical location

To identify potential difficulties stemming from the project’s stakeholders, to assess their
probability of occurrence, and to try to be head them off in advance, the project team must
learn to interact frequently with those individuals and institutions which constitute the most
important elements of the project’s external environment. Together with the project’s
sponsors, owners and users, these people constitute the project’s direct and indirect
stakeholders.

Q3. What are the various SCMo software available in project management? Explain each
in brief.
Ans: SCMo:- Supply Chain Monitoring can be an add-on to existing ERP systems. SCMo makes
it more efficient to master difficult to manage supply networks. The various Soft wares
available in project management are:
1. ARROW: It is a consortia of institutional repository solution, combining open source
and proprietary Software. Arrow is preferred support software because it provides a
platform for promoting research output in the ARROW context, Sefeguards digital
information, Gathers an institution’s research output into one place, Provides consistent
ways of finding similar objects, Allows information to be preserved over the long term,
Allows information from many repositories to be gathered and searched in one step,
Enables resources to be shared, while respecting access constraints, Enables effective
communication and collaboration between researchers. The ARROW project will
identify and test software or solutions to support best practice institutional digital
repositories comprising e-prints, digital theses and electronic publishing.
2. FEDORA: ARROW wanted a robust, well architected underlying platform and a
flexible object-oriented data model to be able to have persistent identifiers down to the
level of individual data streams. It accommodates the content model to be able to be
version independent. Since the beginning of the project ARROW has worked activity
and closely with FEDORA and the FEDORA Community. The ARROW project’s
technical architect is a member of FEDORA Advisory Board and sits on FEDORA
Development Group.

Chandan Kumar Sah Page 135 of 152 MBA 2ND Semester


3. VITAL: VITAL refers to ARROW specified software created and fully supported by
VTLS Inc. built on top of FEDORA. IT currently provides: VITAL Manager, VITAL
Portal, VITAL Access Portal, VALET - Web Self-Submission Tool, Batch Loader
Tool, Handles Server, Google Indexing and Exposure, SRU/SRW Support, VITAL
architecture overview. VITAL is part of creative development of ARROW institutional
repositories. VITAL has the following features:
a) Inclusion of multimedia and creative works produced in Australian universities
b) Limited exposure nationally of internationally
c) Addition of annotation capability
d) Inclusion of datasets and other research output not easily provided in any other
publishing channel
e) Being developed in conjunction with the DART Project
f) Exploration of the research-teaching nexus tools that will allow value added
services for repositories
g) Integration with or development of new tools that will allow value added services
for repositories
4. PILIN: Persistent Identifiers and Linking Infrastructure: There has been a growing
realization that sustainable identifier infrastructure is required to deal with the vast
amount of digital assets being produced and stored within universities. PILIN is a
particual challenge for e-research communities where massive amounts of data are
being generated without any means of managing this data over any length of time. The
board objectives are to:
a) Support adoption and use of persistent identifiers and shared persistent identifier
management services by the project stakeholders
b) Plan for a sustainable, shared identifier management infrastructure that enables
persistence of identifiers and associated services over archival lengths of time.
c) Deploy a Worldwide Site Consolidation Solution for Exchange Server 2003 at
Microsoft
d) Add Picture
e) Use Microsoft Exchange Server 2003 to consolidate more than 70 messaging sites
worldwide into seven physical locations
5. MS EXCHANGE SERVER 2003:

Chandan Kumar Sah Page 136 of 152 MBA 2ND Semester


Q4. List the various steps for Risk management. Also explain GDM and its key features.
Ans: Risk Management: Risks are those events or conditions that may occur and whose
occurrence has a harmful or negative impact on a project. Risk Management aims to
identify the risks and then take actions to minimize their effect on the project. Risk
management entails additional cost. Hence risk management can be considered cost-
effective only of the cost of risk management is considerably less than the cost incurred if
the risk materializes. The following are the simple steps which can be followed:
a) Determine scope of the risk session.
b) Select the team and moderator.
c) The moderator explains the risk process to new team members.
d) Identify risks.
e) Brainstorm areas of risk.
f) Identify things that have caused problems in the past, such as loss of key staff, missed
deadlines, or error-prone software.
g) Remove invalid or irrelevant stuff.
GDM: GDM stands for Global Delivery Model. The term GDM is typically associated
with companies engaged in IT consulting and services delivery business. The GDM enables
an industry or business to plan, design and deliver products and services to any customer
worldwide with speed, accuracy, economy and reliability. GDM enables its customer to
leverage varied locations across the globe that provides optimized value for every
component of delivery. The key features of GDM are:
a) Standardization: It includes ingenious design and development of components and
features which are like to be accepted by 90% of world-wide customers. GDM heavily
depends on Global Standards of Design focusing on highly standardized methods and
processes of manufacture or development. It adopts plug-and-socket concepts with
minimum adaptable joints or connections.
b) Modularization: GDM requires product or solution to be split up into smallest possible
individual identifiable entities. These entities will have limited individual functioning
capability but they can become powerful and robust in combination with other modules.
c) Minimum Customization: GDM mandates only minimum changes or modifications to
suit individual customers.

Chandan Kumar Sah Page 137 of 152 MBA 2ND Semester


d) Maximum Micro Structuring: GDM encourages splitting of the Product Modules
future into much smaller entity identifiable more through characteristics rather than
application features. These Microbial Entities are standardized even across Multiple
Modules. Application of these Microbial Entities rest within multiple projects of
Products or even as add-ons to suit customer needs later.

Q5. Answer the two parts:


a. Importance of data management in project management-Comment.
b. What is the significance of reviewing ROI?
Ans: a) Importance of data management in project management: Data Management consists of
conducting activities which facilitate acquiring data, processing it and distributing it.
Acquisition of data is the primary function. Data should have three important
characteristics – timeliness, sufficiency and relevancy. Management of acquisition lies in
ensuring that these are satisfied before they are stored for processing and decisions taken
on the analysis. There should be data about customers, suppliers, market conditions, new
technology, opportunities, human resources, economic activities, government regulations,
political upheavals, all of which affect the way we function. Most of the data go on
changing because the aforesaid sources have uncertainty inherent in them. So updating data
is a very important aspect of their management. Storing what is relevant in a form that is
available to concerned persons is also important. When a project is underway dataflow
from all members of the team will be flowing with the progress of activities. The data may
be about some shortfalls for which the member is seeking instructions. A project manager
will have to analyze them, discover further data from other sources and see how he can use
them and take decisions. Many times he will have to inform and seek sanction from top
management. The management will have to study the impact on the overall organizational
goals and strategies and convey their decisions to the manager for implementation. To
ascertain availability at some future point of time, information about orders placed,
backlogs, lead times are important for all the members. A proper MIS will take care of all
these aspects. ERP packages too help in integrating data from all sources and present them
to individual members in the way they require. When all these are done efficiently the
project will have no hold ups an assure success.

Chandan Kumar Sah Page 138 of 152 MBA 2ND Semester


b) Significance of reviewing ROI: ROI stands of Return on Investment. ROI is the
calculated benefit that an organisation is projected to receive in return for investing money,
time and resources in a project. Within the context of the review process, the investment
would be in an information system development or enhancement project. ROI information
is used to assess the status of the business viability of the project at key checkpoints
throughout the project’s life-cycle. ROI may include the benefits associated with improved
mission performance, reduced cost, increased quality, speed, or flexibility, and increased
customer and employee satisfaction. ROI should reflect such risk factors as the project’s
technical complexity, the agency’s management capacity, the likelihood of cost overruns,
and the consequences of under or non-performance. Where appropriate, ROI should reflect
actual returns observed through pilot projects and prototypes. ROI should be quantified in
terms of the break-even point, which is the time when the investment begins to generate a
positive return. ROI should be re-calculated at every major checkpoint of a project to see if
the BEP is still on schedule, based on project spending and accomplishments to date.

Q6. XYS Company implements CMMI level-03. To make further changes it decides on
starting a new division in the organization. It decides to advance the existing project
management. What are the steps to be followed by the organization to drive project
management to a new horizon?
Ans: The steps to be followed by XYZ Company to drive project management to a new horizon
are as under:-
a) Believing in discontinuity and not in continuity with incremental improvements:
Continuity or status quo is a function of quantum of changes. Incremental
improvements are valid only when the rate of change is not excessive. Modern domains
and development processes are highly dynamic. Project management today demands
discontinuity and greater flexibility, with no hurdles to move into future. An open mind
towards the unknown is the key. Members are not expected to be rattled by sweeping
changes demanded by the dynamically changing objectives.
b) Owning the problems and sharing the solutions: Experience is gained through keeping
an open mind to problems. Experience is now not judged by number of years of service
but by the number and complexity of problems owned and solved. The fixed mould

Chandan Kumar Sah Page 139 of 152 MBA 2ND Semester


mentality of passing the buck and expecting credit for discoveries is not appreciated
any more.
c) Breaking the status quo mentality: The implies that future is not an extension of the
present and therefore it needs to be tamed. Excessive focusing on future may not
however be appreciated. A proper balance is a must. In most cases, a conflict between a
present – centric manager and a future centric project leader can only be solved by the
innovative predictive approach of a member – it is usually the member who is more
aware of future bugs and client centric problems.
d) Stepping out of comfortable zone: Momentarily, a member might feel more
comfortable to stay in the comfort zone and refrain from risks. However, he or she
should remember that a team leader has emerged by stepping out of the comfort zone.
On the other hand, if a member decides to step out of the comfort zone and be
innovative and contribute then this would amount to emergence of a future leader.
Promotions are not time bound any more.
e) Human capital bypassing financial: Human capital has left financial capital far
behind. A member should therefore appreciate and maintain self realization of his
importance in the organisation. However, to sustain credibility, the member should
remember that his or her value is not related to the level or the salary but on the
quantum of output, again not on the volume of coding but on the value of the work
done to the product. Combining these two concepts of human capital and value, the
concept of return on the time invested has replaced return on investment when it comes
to valuation.
f)Transforming work culture from 5 to 7 dimensions: Conventionally we live among the
five dimensions – X, Y, Z, time and mind. We need to add passion and joy if we aim
any substantial progress. It is important for a member to decide to do what he or she
wants to do Then the gap between wish and reality is narrowed and the results improve.
A project could well become a work of art.
g) Real number of encounters replacing number of years of experience:
The experience level is now judged by the number of encounters of major problems
solved rather than the number of years. Experience is then countered as wisdom. The
secret formula for a member is to build an inventory of encounters meaningful to his or
her own dream or passion profile.

Chandan Kumar Sah Page 140 of 152 MBA 2ND Semester


h) Seeking meaning out of change: Change could be a threat or an opportunity. If change
is resisted, it is essential for the entire team, through extraordinary team effort and brain
storming, to weigh consequences before drafting of a plan to match proposals for
changes. The key lies in extraordinary inter personal skills and communication.
i) Detachment from the fruits of results: To act is within one’s control. To get a reaction or
a reward is not within his or her purview. Too much attention to these factors would
result in drop in productivity and further hinder personal progress. The team members
should rather analyse and discuss among themselves and let results speak for
themselves.

Chandan Kumar Sah Page 141 of 152 MBA 2ND Semester


Assignment Set – 2

Q1. Providing adequate resource is key to productivity- Comment.

Ans: Key elements of a Productivity Improvement Program:


1. Obtain Upper Management Support. Without top management support, experience
shows a PIP likely will fail. The Chief Executive Officer should issue a clear,
comprehensive policy statement. The statement should be communicated to everyone in the
company. Top management also must be willing to allocate adequate resources to permit
success.
2. Create New Organizational Components. A Steering Committee to oversee the PIP and
Productivity Managers to implement it are essential. The Committee should be staffed by
top departmental executives with the responsibilities of goal setting, guidance, advice, and
general control. The Productivity Managers are responsible for the day-to-day activities of
measurement and analysis. The responsibilities of all organizational components must be
clear and well established.
3. Plan Systematically. Success doesn't just happen. Goals and objectives should be set,
problems targeted and rank ordered, reporting and monitoring requirements developed, and
feedback channels established.
4. Open Communications. Increasing productivity means changing the way things are
done. Desired changes must be communicated. Communication should flow up and down
the business organization. Through publications, meetings, and films, employees must be
told what is going on and how they will benefit.
5. Involve Employees. This is a very broad element encompassing the quality of work life,
worker motivation, training, worker attitudes, job enrichment, quality circles, incentive
systems and much more. Studies show a characteristic of successful, growing businesses is
that they develop a "corporate culture" where employees strongly identify with and are an
important part of company life. This sense of belonging is not easy to engender. Through
basic fairness, employee involvement, and equitable incentives, the corporate culture and
productivity both can grow.
6. Measure and Analyze. This is the technical key to success for a PIP. Productivity must
be defined, formulas and worksheets developed, sources of data identified, benchmark

Chandan Kumar Sah Page 142 of 152 MBA 2ND Semester


studies performed, and personnel assigned. Measuring productivity can be a highly
complex task. The goal, however, is to keep it as simple as possible without distorting and
depreciating the data. Measurement is so critical to success, a more detailed analysis is
helpful.

Q2. Compare the following:


a. Traditional Vs. Projectised Organization.
b. Bottom-up Vs. Top-down estimation
Ans: a) Traditional Vs. Projectised Organisation
Traditional Organisations Projectised Organisations
They have the formal organisation structure, They have teams comprising members
with documents, functions, sections having a who are responsible for completing one
hierarchy of managers and their assistants. entire deliverable products.
All of the managers function on a continuous The teams will have all the resources
basis catering to a series of requirements required to finish the jobs.
issued by the planning department.
An assembly of various units of their They have a time schedule within which
production forms a products and a variety of all the elements of the projects have to
such products make up the business of the be completed.
company.
No particular member or a department or a There is greater accountability among
team is responsible for the completion of any team members and everyone is
particular product. Their creativity and responsible for the delivery.
innovation is in particular respect of their
jobs.
Most of the members do not get exposed to It is found that a sense of ‘ownership’ of
other areas of operations in the organization. the project motivates team members to
They become specialists and insular. be creative, cooperative among them to
achieve high productivity.

b) Bottom-up Vs. Top-down estimation:


Bottom-up estimation: The bottom up approach consists of the following steps.
1) Project manager first divides the product under development into major modules.
2) Each module is subdivided into smaller units.

Chandan Kumar Sah Page 143 of 152 MBA 2ND Semester


3) Project manager defines a standard for manufacturing and self-testing by Identifying
modules in the system and classifying them as simple, medium or complex, Using
either the provided standard definitions or definitions from past projects as much as
possible, Getting the average build effort for simple/medium/complex programs from
the baseline if a project specific baseline exists.
Top-down estimation: The top down approach consists of the following steps:
1) Getting the estimate of the total size of the product in function points
2) Fixing the productivity level for the project using the productivity data from the project
specific capability baseline from the general process capability baseline, or from similar
projects
3) Obtaining the overall effort estimate from the productivity and size estimates
4) Using effort distribution data from the process capability baselines or similar projects to
estimate the effort for the various phases
5) Refining the estimates taking project specific factors into consideration

Q3. List out the macro issues in project management and explain each.
Ans: The macro issues in project management are as under:-
a. Evolving Key Success Factors (KFS) Upfront: In order to provide complete stability to
fulfillment of goals, a project manager needs to constantly evaluate the key success
factors from time to time. While doing so, he needs to keep the following aspects of
KSFs in mind:
* The KFS should be evolved based on a basic consensus document.
* KSF will also provide an input to effective exit strategy (EES). Exit here does not
mean exit from the project but from any of the drilled down elemental activities
which may prove to be hurdles rather than contributors.
* Broad level of KSF should be available at the conceptual stage and should be
firmed up and detailed out during the planning stage. The easiest way would be
for the team to evaluate each step for chances of success on a scale of ten.
* KSF should be available to the management – duly approved by the project
manager – before execution and control stages.

Chandan Kumar Sah Page 144 of 152 MBA 2ND Semester


* KSF rides above normal consideration of time and cost – at the levels
encompassing client expectation and management perception – time and cost
come into play as subservient to these major goals.
b. Empowerment Title (ET): ET reflects the relative importance of members of the
organisation at three levels:
i) Team members are empowered to work within limits of their respective allocated
responsibilities. The major change from bureaucratic systems is an expectation from
these members to innovate and contribute to time and cost.
ii) Group leaders are empowered additionally to act independently towards client
expectation and are also vested with some limited financial powers.
iii) Managers are empowered further to act independently but to maintain a scientific
balance amount time, cost, expectation and perception, apart from being a virtual
advisor to the top management.
c.Partnering Decision Making (PDM): PDM is a substitute to monitoring and control. A
senior with a better decision making process will work closely with the project
managers as well as members to plan what best can be done to manage the future better
from past experience. The key here is the active participation of members in the
decision making process. The ownership is distributed among all irrespective of levels
– the term equally should be avoided here since ownership is not quantifiable. The right
feeling of ownership is important.
d. Management by Exception (MBE): “No news is good news”. If a member wants help
he or she locates a source and proposes to the manager only of such help is not
accessible for fee. Similarly, a member should believe that a team leader’s silence is a
sign of approval and should not provoke comments through excessive seeking of
opinions. In short leave people alone and let situation perform the demanding act. The
bend limit of MBE can be evolved depending on the sensitivity of the nature and size of
the project. MBE provides and facilitates better implementation of effectiveness of
empowerment titles. MBE is more important since organizations are moving toward
multi-skilled functioning even at junior most levels.

Q4. Describe the traits of a professional manager in details?

Chandan Kumar Sah Page 145 of 152 MBA 2ND Semester


Ans: The project manager is an important team member who often makes the difference between
project success and failure. A project manager should have the skills to unite diverse
individuals and have them function as on cohesive team. The traits of a professional
manager are as under:
a) Leadership: These managers lead by exhibiting the characteristics of leadership. They
know what they should do, know why they are doing it, how to do it and have the
courage and vision to execute it. They have the power of taking along others with them.
They lead by making themselves as an example for the entire team. He is about to trust
his team members and assign the right job to the right person.
b) People Relationships: Any leader without followers cannot be successful. They need to
have excellent human relationship skills. The manager should build up his team based
on the core values of sincerity, objectivity, dedication and ethics, ensure that his
subordinates get opportunities for growth based on performance, make his subordinates
a part of the decision making process, thus ensuring cooperation and commitment
during implementation, delegate freely and support them, give enough room for
expression of thoughts and also make things challenging for team members, Keep
aspirations of the team members into considerations.
c) Integrity: Highest levels of trust, fairness and honesty are expected while dealing with
people both within and outside the organisation. This includes the customers,
shareholders, dealers, employees, the government and society at large.
d) Quality: The quality philosophy should not cover only the product quality, but every
process that has gone into making it. Economy of words when instructions are given,
acknowledging compliance, arriving on time, remembering the promises and above all
a keen eye for details and patience to make others know what they want are
components of quality.
e) Customer Orientation: It is now recognized that every organisation has two sets of
customers - internal customers and external customers. Internal customers are people
in the organisation – employees, directors and team members. External customers are
clients and all members of society the company comes in contact in connection with the
business. Both the customers need the company’s solutions for their problems. So, the
manager’s thinking about any problem is – what can I do for him – and all actions will
be in that direction. He should constantly thrive for customer satisfaction.

Chandan Kumar Sah Page 146 of 152 MBA 2ND Semester


f) Innovation and creativity: Professional managers think beyond the obvious. They
exhibit a keenness to go behind a problem and attempt to find the root cause of the
problem. They will draw from their experience from diverse fields, seek further
information and consider all possible alternatives and come out with some new and
unique solution. This happens when they have open minds.
g) Performance Management: The professional manager not only ensures that his
performance is at peak all times, but motivates his entire team to perform the same.
This comes by appreciation and encouragement. In case of shortfalls, he arranges
training for them so that their performance improves. Thus the team members know
that they are expected to perform, that they get help to do so and their effort is
recognized and rewarded too. This is the simple path of performance management. The
managers can follow a seven step performance management model 1) Set
Objectives/Performance Standards, 2) Communicate these to the employees, 3)
Review/monitor, 4) Check actual performance Vs. Standards set, 5) Identify gaps, 6)
Jointly decide on corrective action, if needed, 7) Reset objectives for next period.
h) Identification with the organisation: A sense of pride and belonging goes with the
ownership of the job, the project, team members and organisation. This is brought
about by the culture and communication system in the organisation. Information
sharing brings in trust and promotes belongingness. The tendency seen is that most
managers strongly identify with their own departments, units or divisions and they lack
a sense of organisation.
i) Empowering employees: The professional manager should possess the ability to
empower his employees down the line. Many managers are not even ready to delegate
their authority to subordinates and end up only delegating responsibility. Empowerment
is the process by which employees are encouraged to take decisions pertaining to their
area of work. This leads employees developing a sense of pride in their jobs.
j) Coping with changes: There is a saying – The only constant in this world is change. A
professional manager has the ability and capacity to cope with change. He accepts the
fact that change is inevitable and is ready to implement change at the workplace. To
implement change successfully, it is essential that employees are involved in the
implementation of change. Moreover, the positive and negative consequences of
change need to be discussed and understood before implementation. Thus a

Chandan Kumar Sah Page 147 of 152 MBA 2ND Semester


professional manager has the attitude to accept change as a way of life and takes it in
his stride.

Q5. List the major participants of project review process. Also highlight roles and
responsibilities of each.
Ans: The main particulars of project review process are:
i) Establishing a training plan to acquaint the project team members with the
methodologies, technologies and business areas under study
ii) Updating the project schedule to accommodate scheduled training activities
iii) Identifying the needs for review and reviewing the project scope
iv) Reviewing a project with respect to its stages and progress by preparing a plan for the
review, fixing an agenda to review the project progress and keeping the reports ready
for discussion about stage performance
v) Reviewing the project scope, the objective statement, the non-conformances in the
project stages and identifying the need to use the project plan
vi) Preparing a proper project plan indicating all the requirements from start to finish of the
project and also at every stage of the project
vii) Preparing a checklist of items to be monitored and controlled during the course of
execution of the project.

Roles and responsibilities: The following are the roles and responsibilities:
a) Chief Information Officers (CIO): Conducts Project Senior Management Reviews,
monitors project progress, facilities resolution of related projected issues.
b) Chief Financial Officers (CFO): Approves investments in corporate/major
information systems projects.
c) Systems Owners: Develop or approve project deliverables, present project statuses, and
facilitate resolution of project issues.
d) Project Managers: Perform day to day project management, develop project
deliverables, prepare project management review and senior management review
presentations, present project status and manage resolution of project issues.

Chandan Kumar Sah Page 148 of 152 MBA 2ND Semester


e) Program Managers: Develop or approve project deliverables, approve changes to
project scope, ensure project reporting, present project status, conduct project
management reviews, manage project funding and authorize work activities.
f)Corporate Management Investment Process Program Staff: Evaluate major information
systems which receive CMIP funding and prepare report for the top management.
g) Delivery Manager: Reviews and comments on project deliverables and work products,
schedules and supports the review meetings, provides support to systems owners and
project managers, advises the CIO and associate CIOs
h) Key Project Stake Holders and other invited participants: Attend the review meeting,
participate in discussion and provide inputs as appropriate.

Q6. ABC organization has been in software business since last 20 years. The senior
management feels that although they are making profits, but the profit on an average
is the same each year. They decide that they would make some additions to the
business and decided to go ahead with development of some high technology for
better profits. Can you suggest some guidelines, which the management should follow
in this venture?
Ans: Every business aims to commence its activities in the foreign market. The foreign market
provides with both opportunities and risks. Therefore some prefer to enter in to strategic
relationships and one such is the Joint Ventures. A Joint Venture is an entity formed
between two or more parties to undertake economic activity together. The JV parties agree
to create, for a finite time, a new entity and new assets by contributing equity. They then
share in the revenues, expenses, and assets and the control of the enterprise. Therefore the
basic characteristics of joint venture can be summed up as:
1) Based on a Contractual Agreement.
2) Specific limited purpose and duration.
3) Joint Property Interest
4) Common Financial and Intangible goals and objectives.
5) Shared profits, losses, management and control.
Reasons for setting Joint Ventures abroad. The reasons for setting up joint ventures can be
contributed to three main factors and they are:
1. Internal Reasons.

Chandan Kumar Sah Page 149 of 152 MBA 2ND Semester


2. Competitive Goals.
3. Strategic Goals.

1. The Internal reasons are as follows:


• Building on company’s strength.
• Spreading on costs and risks.
• Improving access to financial resources.
• Economies of scale and advantages of size.
• Access to new technologies and customers.
• Access to innovative managerial practices.

2. The Competitive Goals are as follows:


• Influencing structural evolution of the industry.
• Defensive response to blurring industry boundaries.
• Creation of stronger competitive units.
• Speed to market.
• Improved Agility.

3. The Strategic Goals are as follows:


• Diversification
• Synergies.
• Transfer of technology/skill

Indian Joint Ventures Abroad


India started opening its economy a decade ago to integrate with global economy. The
business ventures abroad are not a new phenomenon in the independent India. The
initiatives were taken way back in the 1960s with the first ventures of Birlas in Ethopia in
the year 1964. However, it has assumed specific significance after the Indian government
started economic reforms in the year 1991, making globalization of Indian business an
integral part of economic reforms.

Chandan Kumar Sah Page 150 of 152 MBA 2ND Semester


Significance of Indian Joint Ventures Abroad
International trade is considered to be imperative for economic development. Economic
borders of various countries have been opened on this premise under the aegis of world
trade organization. In countries, whose economy has moved from the level of necessity to
comforts and luxuries levels, there are increasing pressures for newer, better and superior
products with consistent quality, high reliability and attractive finish etc. Further, with the
labour becoming increasingly costly, the firms have to go for development of capital
intensive technologies. The huge investments in new product and technology development
demands higher levels of production to ensure operations of the firms above the breakeven
point. The scale of operations required over a period of time reaches a level that is well
above the entire domestic demand in most of the developed countries, which generally
have small population. The firms thus face the problem of searching new markets and
cheaper sources of raw material, labour and other resources. Their growth and
development, thus, depends upon internationalization of the business.

Advantages and Disadvantages


A business while deciding upon whether to go for a joint venture should make a thorough
analysis on its business goals.

Advantages
• Financial resources can be shared.
• Allows for Investor diversification.
• Reduces local Friction.
• Reduce Fixed costs per product.
• Direct management of business activities.
• Competitive strengths of two parties can be combined.
• A local JV partner knows the market.
• Economic incentives add value to JVs.

Disadvantages
• JV profits are shared.

Chandan Kumar Sah Page 151 of 152 MBA 2ND Semester


• Shared technologies can be used beyond JV.
• Local Management of a JV can be unknown

Broadly there are two schemes under which an Indian Party can set up a JV abroad, namely
the Automatic Route and the Normal Route/Approval Route.

Automatic Route
Under the Automatic Route, an Indian Party does not require any prior approval from the
Reserve Bank for setting up a JV abroad (in case of investment in the financial sector,
however, prior approval is required from the concerned regulatory authority both in India
and abroad).

The criteria for direct investment under the Automatic Route are as under:
• The total µfinancial commitment of the Indian Party in JVs in any country other
than Nepal, Bhutan and Pakistan is up to 100% of its net worth and the investment is in
a lawful activity permitted by the host country
• The Indian Party is not on the Reserve Banks exporters caution list / list of
defaulters to the banking system published/ circulated by the Credit Information Bureau
of India Ltd. (CIBIL)/RBI or under investigation by the Enforcement Directorate or any
investigative agency or regulatory authority;
• The Indian Party routes all the transactions relating to the investment in a JV
through only one branch of an authorized dealer to be designated by it.

Normal Route
Proposals not covered by the conditions under the automatic route require the prior
clearance of the Reserve Bank for which a specific application in form ODI with the
documents prescribed therein is required to be made to RBI. Requests under the normal
route are considered by taking into account inter alias the prima facie viability of the
proposal, business track record of the promoters, experience and expertise of the promoters,
benefits to the country, etc.

Chandan Kumar Sah Page 152 of 152 MBA 2ND Semester

Anda mungkin juga menyukai